Study Kit Brochure for SSC Combined Graduate Level...

105
Study Kit Brochure for SSC Combined Graduate Level Examination - 2014 For Any Query Call us @ +91 8800734161, 011- 65023618

Transcript of Study Kit Brochure for SSC Combined Graduate Level...

Page 1: Study Kit Brochure for SSC Combined Graduate Level ...static.upscportal.com/files/study-kit/ssc-cgl/SSC-CGL-2013... · Brochure for SSC Combined Graduate Level Examination - 2014

Study KitBrochure for SSC

Combined Graduate LevelExamination - 2014

For Any Query Call us@

+91 8800734161, 011- 65023618

Page 2: Study Kit Brochure for SSC Combined Graduate Level ...static.upscportal.com/files/study-kit/ssc-cgl/SSC-CGL-2013... · Brochure for SSC Combined Graduate Level Examination - 2014

Click Here for SSC CGL Online Coaching: http://sscportal.in/community/courses/ssc-cgl-tier-1

[Click Here to Buy This Study Kit in Hard Copy: http://sscportal.in/community/study-kit/cgl ] 1

1. Study Kit Details

• Our Objectives • Our Strategy • Implementation Aspects • Contents of the Kit • Price of the Kit • Payment Options • Contact Details

2. Contents Details of Study Kit for Each Sections

(I) Contents of General Intelligence and Reasoning

• Contents of Verbal • Contents of Non Verbal Reasoning

(II) Contents of General Knowledge and Awareness

• Contents of Conventional Parts (Indian Polity, Geography, Economy, General Science & Indian History)

• Contents of Current Affairs

(III) Contents of Numerical Aptitude

• Contents of Basic Mathematics • Contents of Data Interpretation & Data Sufficiency

3. Sample Study Materials of Our Study Kit

Contents of the Brochure

Page 3: Study Kit Brochure for SSC Combined Graduate Level ...static.upscportal.com/files/study-kit/ssc-cgl/SSC-CGL-2013... · Brochure for SSC Combined Graduate Level Examination - 2014

Click Here for SSC CGL Online Coaching: http://sscportal.in/community/courses/ssc-cgl-tier-1

[Click Here to Buy This Study Kit in Hard Copy: http://sscportal.in/community/study-kit/cgl ] 2

Dear Candidates, The Complete Study Kit in Hard copy of for Tier -1 examination is now available.

Our Objectives:

• Firstly to cover 100% SSC Combined Graduate Level (CGL) examination syllabus. • Secondly to compile all the required study materials in a single place, So to save the precious time of

the aspirants. Further, in this study kit we have incorporated all the available study material in the market, we have also include the material of standard Books meant for SSC CGL preparation; like REASONING BY RS AGGARWAL, QUANTITIVE APTITUTE BY RS AGRRAWAL, MAGICAL BOOKS ON QUICKER MATHS BY M TYRA, GENERAL STUDIES SECTION based on MANORAMA YEAR BOOK etc. It is in the interest of candidate so that they need not to study separate books for every section

Our Strategy:

• Content of every section of the syllabus is developed after a thorough research of last year Question in Papers.

• Every section is covered with practice set. • Full length Mock Test Papers will be sent to the candidates during the course of the preparation.

Implementation Aspects:

• You will Get 5 booklets of (i). General Intelligence and Reasoning, (ii) General Knowledge and Awareness (iii) Numerical Aptitude, (iv) Numerical Aptitude (Data Interpretation) and (v) English Comprehension

• Study Kit will be delivered to your postal Address after payment confirmation. • After dispatching your kit we will provide you courier tracking details. • For any help we will provide Telephonic & Email Support to the candidates.

Contents of the Kit: Booklet No Subjects Pages MCQs

1 General Intelligence & Reasoning 176 1347+

2 General Knowledge & Awareness 332 950+

3 4

Numerical Aptitude Numerical Aptitude (Data Interpretation)

423 157 2496+

5 English Comprehension 147 860+

Total Booklets - 5

Total Pages: 1300+

Total MCQs: 5653+

Study Kit Details

Page 4: Study Kit Brochure for SSC Combined Graduate Level ...static.upscportal.com/files/study-kit/ssc-cgl/SSC-CGL-2013... · Brochure for SSC Combined Graduate Level Examination - 2014

Click Here for SSC CGL Online Coaching: http://sscportal.in/community/courses/ssc-cgl-tier-1

[Click Here to Buy This Study Kit in Hard Copy: http://sscportal.in/community/study-kit/cgl ] 3

Price of the Kit:

• The price of our Study Kit is Rs. 3500, including courier charges and full length Mock Test Series which you will get during your course of preparation.

Payment Options: First option: Cash Deposit You can deposit the amount at any ICICI Branch in these following Account Numbers.

Second option: Online Electronic Transfer You can transfer the amount from any Bank Account to our Account No. by using NEFT. Our Account Numbers are

Our Accounts Detail

Third Option: By Hand

Our Office Address:

After Your Payment Click the Given Link for further Process: http://sscportal.in/community/study-kit/payment-details

Page 5: Study Kit Brochure for SSC Combined Graduate Level ...static.upscportal.com/files/study-kit/ssc-cgl/SSC-CGL-2013... · Brochure for SSC Combined Graduate Level Examination - 2014

Click Here for SSC CGL Online Coaching: http://sscportal.in/community/courses/ssc-cgl-tier-1

[Click Here to Buy This Study Kit in Hard Copy: http://sscportal.in/community/study-kit/cgl ] 4

Section 1: General Intelligence and Reasoning

• Analogy • Classification • Series • Coding- Decoding • Blood relations • Direction sense test • Logical venn diagram • Alphabet Test • Sitting Arrangement • Mathematical Operations • Arithmetical Reasoning • Asserting the Missing Characters • Number, Ranking and Time Sequence • Syllogism • Logical Sequence of Words • Alpha- numeric of Words • Word Arrangement • Ranking test • Puzzle test • Cube and Cuboid • Dice • Diagram Intrepretation • Mirror Image • Water Image • Test of Equality • Matrix • Time and Clock • Calendar

Section 2: General Knowledge and Awareness

1. Conventional section

• Indian Polity • Geography • Indian Economy • Indian History • General Science

2. Current Affairs

• Weekly events of national and international events • Last one year of focused current affairs for SSC exam

Section 3: Numerical Aptitude

1. Section I

Page 6: Study Kit Brochure for SSC Combined Graduate Level ...static.upscportal.com/files/study-kit/ssc-cgl/SSC-CGL-2013... · Brochure for SSC Combined Graduate Level Examination - 2014

Click Here for SSC CGL Online Coaching: http://sscportal.in/community/courses/ssc-cgl-tier-1

[Click Here to Buy This Study Kit in Hard Copy: http://sscportal.in/community/study-kit/cgl ] 5

• Number system • Fractions & Decimal Fractions • Indices and dices • Square Root & Cube Root • Simplification • HCF & LCM • Problems on Ages • Percentage • Profit & Loss • Ration & Proportion • Partnership • Time & Work • Problems on Train • Boats & Streams • Mixture • Simple Interest • Compound Interest • Area • Volume & Surface Area • Height & Distance • Trigonometry • Basic Algebra • Clocks and Calendar

2. Section II

• Tabulation • Bar Graph • Pie Charts • Line Graphs • Data Sufficiency

Section 4: English Language & Comprehension

1. Part I

• Parts of Speech/ Sentence • Spotting Error/Common Error • One Word Substitution • Idioms & Phrases • Sentence Correction • Antonyms & Synonyms • Cloze test or Numbered Gaps • Sentence Reconstruction/ Jumbled Sentence • Spelling Test/ Commonly Misspelled Word

2. Part II : Comprehension

Page 7: Study Kit Brochure for SSC Combined Graduate Level ...static.upscportal.com/files/study-kit/ssc-cgl/SSC-CGL-2013... · Brochure for SSC Combined Graduate Level Examination - 2014

Click Here for SSC CGL Online Coaching: http://sscportal.in/community/courses/ssc-cgl-tier-1

[Click Here to Buy This Study Kit in Hard Copy: http://sscportal.in/community/study-kit/cgl ] 6

SAMPLE CHAPTERS OF THE STUDY KIT

Section 1 : General Intelligence and Reasoning

Coding-Decoding

What is Coding-Decoding? For transmitting secret messages from one place to another, especially in defense services, Coding is

used. Decoding is the ability to break the secret code. The codes are based on various principles or patterns. In CSAT, questions based on coding-decoding could be given to judge the intelligence and mental ability of the candidates.

The coded word itself does not make any sense unless we know the coding principle. For coding-decoding, following basic knowledge is required.

1. Forward Order of Letters A B C D E F G H I 1 2 3 4 5 6 7 8 9 J K L M N 0 P Q R 10 11 12 13 14 15 16 17 18 S T U V W X Y Z 19 20 21 22 23 24 25 26

2. Reverse Order of Letters A B C D E F G H I 26 25 24 23 22 21 20 19 18 J K L M N 0 P Q R 17 16 15 14 13 12 11 10 9 S T U V W X Y Z 8 7 6 5 4 3 2 1

3. opposite Letters A-Z, H-S, B-Y, I-R, C-X, J-Q, D-W, K-P, E-V, L-0, F-U, M-N, G-T.

The questions based on coding-decoding are broadly categorised into following categories

Type 1 Coding with Alphabet Letters: In such questions, the letters do not stand for themselves but are

allotted some other values based on some logical patterns or analogies. By deciphering the principles or pattern involved the candidates are required to decode the coded word.

Example l: As ‘COUNSEL’ is to BITIRAK’ So ‘GUIDANCE’ is to (a) FOHYZJBB (b) HOHYBJBA (c) FPHZZKAB (d) FORHYZJBB

Solution. (a) The pattern is C – 1 = B O – 6 = I

Page 8: Study Kit Brochure for SSC Combined Graduate Level ...static.upscportal.com/files/study-kit/ssc-cgl/SSC-CGL-2013... · Brochure for SSC Combined Graduate Level Examination - 2014

Click Here for SSC CGL Online Coaching: http://sscportal.in/community/courses/ssc-cgl-tier-1

[Click Here to Buy This Study Kit in Hard Copy: http://sscportal.in/community/study-kit/cgl ] 7

U – 1 = T N – 5 = I S – 1 = R E – 4 = A L – 1 = K

G

U I D A N C E

F O H Y Z J B B

-1 -6 -5 -4 -3-1 -1 -1

Example 2: In a certain code, TERMINAL is written as SDQLJOBM. How is CREDIBLI written in that code? (a) BQDCJCMF (b) BQDCHAKD (c) DSFEJCMF (d) DSFEHAKD

Solution. (a)

T S

E D

R Q

M L

I J

N O

A B

L M

-1

-1

-1

-1

+1

+1+1

+1

On the basis of this rule CREDIBLE will be written as BQDCJCMF.

Type 2 Coding with Numbers: The principle of coding with numbers is similar to that of coding with letters

except the use of digits. Example 3: If in a certain language, E is coded as 2, H is coded as 1, N is coded as 7, P is codec as 5, A

is coded as 0, T is coded as 8 and L is coded as 3, how is ELEPHANT coded in that language ? (a) 23251078 (b) 32210587 (c) 23527801 (d) 22315078

Solution. (a) E = 2, L = 3, E = 2, P = 5, H = 1, A = 0, N = 7, T = 8 Hence, only (a) is valid.

Example 4: In a certain language, 1 is coded as C, 6 is coded as E, 0 is coded as L, 2 is coded a; G and 3 is coded as O. How is 1300626 coded in that language ? (a) OLCGELE (b) EGEOLEL (c) COLLGEE (d) COLLEGE

Solution. (d) 1 = C, 3 = 0, 0 = L 0 = L, 6 = E, 2 = G, 6 = E Hence, only (d) is valid.

Page 9: Study Kit Brochure for SSC Combined Graduate Level ...static.upscportal.com/files/study-kit/ssc-cgl/SSC-CGL-2013... · Brochure for SSC Combined Graduate Level Examination - 2014

Click Here for SSC CGL Online Coaching: http://sscportal.in/community/courses/ssc-cgl-tier-1

[Click Here to Buy This Study Kit in Hard Copy: http://sscportal.in/community/study-kit/cgl ] 8

Type 3 Mixed coding (Alphabets + Digits): In such questions, both letters and numbers are assigned as

code The candidates are required to identify the analogy given in the questions. Example 5: If ‘INDIA’ stands for ‘I - 14 - D - 9 - A’ and NEPAL’ stands for N - 5 _ P _ 1 _ L, then how

will you code the GIRL ? (a) G9R12 (b) G15R10 (c) G10R9 (d) G9R11

Solution. (a) Assign the alphabetical number to the letters. Example 6: If ‘LOINESS’ is coded as `« ? # @ $ $ $’and MEN’ is coded as ↑φ @, then how will you code

the MISSION’? (a) ↑ # $ $ # ? @ (b) # ↑ $ ? « φ @ (c) ↑ # $ « φ @ (d) ↑ $ # ? « φ @

Solution. (a) The intelligent deciphering of the two codes gives the following M I S O L E N ↑ # $ ? « φ @

So, MISSION will be coded as ↑ # $ $ # ? @ .

Type 4 Miscellaneous types: In such questions, a careful deciphering is required to find out the patterns. Example 7: If FRANCE’ is coded as NCEFRA’ and ‘CANADA’ is coded as ADACAN, then how will you

code ‘MEDICINE’? (a) CNIEMEDI (b) CINEDEMI (c) CINEMEDI (d) CINEDEIM

Solution.(c) The first half part is transferred after the second half part. The coding is made as follows F R A N C E N C E F R A

So, the code of MEDICINE will be coded as ‘CINEMEDI’. Some More Solved Examples Example 8: If A = Z, B = Y, C = X and so on, then what will be the code of BLACK?

(a) OYZXP (b) YOZXP (c) YOZPX (d) YOXZP

Solution. (b) Now, the code of BLACK is YOZXP. Example 9: If FOOD is coded as ENNC, then decode the code of SPEED.

(a) RODDC (b) RDOCC (c) RODCD (d) ROCDC

Solution. (a) Here, each letters is allotted the value of its preceding letter in the sequence. So the code of SPEED is RODDC.

Example 10: If INLAND is coded as DNALNI, how will POSTED be coded? (a) DESTOP (b) DETOPS (c) DETSPO (d) DETSOP

Solution. (a) Now, reverse the letters of the word. So, the code of POSTED will be DETSOP. Example 11: If EXPORT is coded as USPQYF, then how will CAPITAL be coded?

(a) MBJUQBD (b) MBUJQBD (c) BMUJQBD (d) MBUJQDB

Solution. (b) Reverse the word and use next alphabet as code. So, the answer is MBUJQBD.

Page 10: Study Kit Brochure for SSC Combined Graduate Level ...static.upscportal.com/files/study-kit/ssc-cgl/SSC-CGL-2013... · Brochure for SSC Combined Graduate Level Examination - 2014

Click Here for SSC CGL Online Coaching: http://sscportal.in/community/courses/ssc-cgl-tier-1

[Click Here to Buy This Study Kit in Hard Copy: http://sscportal.in/community/study-kit/cgl ] 9

EXERCISE 1.In a certain code LAWN is written as JCUP. How will SLIT be coded in that code? (a) QNGV (b) QJGV (c) QNVG (d) NJGV 2. In a certain code SATELLITE is written as FUBHTLDSHK. How is LAUNCHING written in that

code? (a) DOUBFMGHO (b) OVBMCFMHG (c) OVMBCFMGH (d) DOUBCFMHG 3. In a certain code LOUD is written as JOSF then which of the following English words shall be coded

as PKQG? (a) RISE (b) ROPE (c) ROAD (d) RICE 4. In a certain code BREAKDOWN is written as BFSCJMVNC. How is ORGANISED written in that

code? (a) PSHBMCDRH (b) BHSPMCDRH (c) BHSPOCDRH (d) BHSPNHRDC 5. In a certain code CONCISE is written as FTJBBNM. How is FISHERY written in that code? (a) ZSFIGJT (b) ZSFGIHR (c) ZSFGEHR (d) ZSFEHGR 6. In a certain code '59346' is written as $AD%F and '8173' is written as 'HB#D'. How is '9865' written in

that code? (a) HAF$ (b) AFH$ (c) ADF$ (d) None of these 7. In a certain code PRAISE is written as #@$27% and RESPIRE is written as @%7#2@%. How is

REPAIR written in that code? (a) @%#2$@ (b) @%$#2@ (c) @%#$2@ (d) @%$2#@ 8. In a certain code LONG is written as 5123 and GEAR is written as 3748. How is LANE written in

that code? (a) 5427 (b) 5247 (c) 5847 (d) 5237 9. In a certain code READ is written as #5%6 and PAID is written as $%46. How is RIPE written in that

code? (a) #4$5 (b) #6$5 (c) $4#5 (d) $4#6 10. In a certain code GEAR is written as '5934' and RIPE is written as '4869'. How is PAGE written in

that code? (a) 6359 (b) 6539 (c) 4359 (d) 6459 Directions (Q. No. 11-20) : In each of these questions a group of letters is given followed by four combinations of digit/symbol numbered (i), (ii), (iii) and (iv). Letters are to be coded as per the scheme and conditions given below. You have to find out the serial number of the combination, which represents the letter group. Letter of that combination is your answer. If none of the combinations is correct, your answer is (5) i.e. 'None of these'. Letters: F G A K M E S P L Q B U R I T Digit/Symbol Codes: 9 % 2 $ * 7 @ 8 1 6 5 Ó 4 # 3 Conditions: (i) If the first letter is a vowel and the last a consonant, both are to be coded as the code for the vowel. (ii) If the first letter is a consonant and the last a vowel, the codes for the first and the last letters are to

be interchanged,

Page 11: Study Kit Brochure for SSC Combined Graduate Level ...static.upscportal.com/files/study-kit/ssc-cgl/SSC-CGL-2013... · Brochure for SSC Combined Graduate Level Examination - 2014

Click Here for SSC CGL Online Coaching: http://sscportal.in/community/courses/ssc-cgl-tier-1

[Click Here to Buy This Study Kit in Hard Copy: http://sscportal.in/community/study-kit/cgl ] 10

(iii) If both the first and the last letters are consonants both are to be coded as 'd' (iv) If more than two vowels are there in the group of letters, all vowels are to be coded as £. 11. TEFSUM (a) δ79@©d (b) 379@©* (c) *79@©3 (d) δ79@δ© (5) None of these 12. BSQEGU (a) 5@67%© (b) δ@67%δ (c) ©@67%© (d) None of these 13. KAGFUB (a) $2%9©5 (b) 52%9©$ (c) $2%©95 (d) δ2%9©δ 14. AQUMTE (a) 76©*32 (b) £6£*3£ (c) 26©*32 (d) 26©*23 15. IPKUSR (a) #8$©@# (b) 48$©@# (c) #8$©@4 (d) #8$4© ANSWERS: 1. (a) 2. (b) 3. (a) 4. (b) 5. (c) 6. (d) 7. (c) 8. (a) 9. (a) 10. (a) 11. (a) 12. (d) 13. (d) 14. (b) 15. (a) EXPLANATIONS: (1) –2 +2 –2 +2 –2 +2 –2 +2 L A W N

J C U P

S L I T

Q N G V

Similarly

(2)

\

+1 S A T E

+1 +1 +1

F U B T

L

L

–1 L I T E

–1 –1 –1

D S H G Similarly +1 L A U N

+1 +1 +1

O V B M

C

C

–1 H I N G

–1 –1 –1

F M H G (3) –2 +2 –2 +2 –2 +2 –2 +2 L O U D

J O S F

R I S E

P K Q G

Similarly

(4) +1 b R E A

+1 +1 +1

B F S C

L

J

–1 D O W N

–1 –1 –1

M V M C

–1

Page 12: Study Kit Brochure for SSC Combined Graduate Level ...static.upscportal.com/files/study-kit/ssc-cgl/SSC-CGL-2013... · Brochure for SSC Combined Graduate Level Examination - 2014

Click Here for SSC CGL Online Coaching: http://sscportal.in/community/courses/ssc-cgl-tier-1

[Click Here to Buy This Study Kit in Hard Copy: http://sscportal.in/community/study-kit/cgl ] 11

Similarly +1 O R G A

+1 +1 +1

B H S P

N

M

–1 I S E D

–1 –1 –1

C D R H 6. Here, Number : 5 9 3 4 6 8 1 7 Code : $ A D % F H B # Therefore, the code for 9895 ⇒ AHF$ 7. P R A I S E # @ $ 2 7 % Similarly R E P A I R @ % # $ 2 @ 8. L O N G E A R 5 1 2 3 7 4 8 Similarly, L A N E 5 4 2 7

9. R E A D P I # 5 % 6 $ 4 So, RIPE → $4$5 11. T E F S U M δ 7 9 @ © δ [condition iii]

12. B S Q E G U © @ 6 7 % 5 [condition ii]

13. K A G F U B δ 2 % 9 © δ [condition iii]

14. A Q U M T E £ 6 £ * 3 £ [condition iv]

15. I P K U S R

# 8 $ © @ # [condition i]

Syllogism

The word ‘Syllogism’ is also referred to ‘Logic’. Syllogism is an important section of logical reasoning and hence, a working knowledge of its rules is required on the part of the candidate. Hence, it can be expressed as the ‘Science of thought as expressed in language’. The questions based on syllogism can be solved by using Venn diagrams and some rules devised with the help of analytical ability.

Page 13: Study Kit Brochure for SSC Combined Graduate Level ...static.upscportal.com/files/study-kit/ssc-cgl/SSC-CGL-2013... · Brochure for SSC Combined Graduate Level Examination - 2014

Click Here for SSC CGL Online Coaching: http://sscportal.in/community/courses/ssc-cgl-tier-1

[Click Here to Buy This Study Kit in Hard Copy: http://sscportal.in/community/study-kit/cgl ] 12

With this unique characteristic, this test becomes an instrument of teaching the candidates to follow the rules and work as per the instructions without an error. Here, only the basic concept and rules, which have a bearing on reasoning faculty could alone help. There are some terminology which are used in syllogism.

Proposition It is also referred to as ‘Premises’. It is a sentence which asserts that either a part of, or the whole of,

one sets of objects-the set identified by the subject term in the sentence expressing that sentence either is included in, or is excluded from, another set-the set identified by the predicate term in that sentence.

Types of Proposition Categorical Proposition There is relationship between the subject and the predicate without any

condition. Example : I. All beams are logs. II. No rod is stick.

Hypothetical Proposition: There is relationship between subject and predicate which is asserted conditionally. Example : I. If it rains he will not come. II. If he comes, I will accompany him.

Disjunctive Proposition In a disjunctive proposition the assertion is of alteration. Example : I. Either he is brave or he is strong. II. Either he is happy or he cannot take revenge.

Parts of Proposition It consists of four parts.

1. Quantifier: In quantifier the words, ‘all’, ‘no’ and ‘some’ are used as they express quantity. ‘All’ and ‘no’ are universal quantifiers because they refer to every object in a certain set. And quantifier ‘some’ is a particular quantifier because it refers to at least one existing object in a certain set. 2. Subject: It is the word about which something is said.

3.Predicate: It is the part of proposition which denotes which is affirmed or denied about the subject. 4. Copula: It is the part of proposition which denotes the relation between the subject and predicate. Example : All boys are brilliant ↑ ↑ ↑ ↑ Quantifier Subject Copula Predicate

Hence, the standard form of proposition is Quantifier + Subject + Copula + Predicate Four-fold classification of categorical proposition: On the basis of quality and quantity of

proposition we can classify them in four categories. To draw valid inferences it is necessary to have a clear understanding of the A, E, I, O relationship as given in the table. Symbol Proposition Quantity Quality

A All A are B Universal Affirmative

E No A is B Universal Negative

I Some A are B Particular Affirmative

O Some A are not B Particular Negative

Rules for Deriving the Conclusions from Two Given Premises 1. Universal affirmative or A-type proposition.

Page 14: Study Kit Brochure for SSC Combined Graduate Level ...static.upscportal.com/files/study-kit/ssc-cgl/SSC-CGL-2013... · Brochure for SSC Combined Graduate Level Examination - 2014

Click Here for SSC CGL Online Coaching: http://sscportal.in/community/courses/ssc-cgl-tier-1

[Click Here to Buy This Study Kit in Hard Copy: http://sscportal.in/community/study-kit/cgl ] 13

Dogs

Goats

Take an example : All goats are dogs This is A type proposition: We can see it by graphical representation of the above proposition we observe that goats are distributed in dogs. Hence. we can conclude that in A type proposition only subject is distributed.

2. Universal negative or E-type proposition.

Boy Girl

Take an example : No girl is boy In this type of proposition both subject and predicate are denial of each other. This can also be seen in the diagram representing boy Girl and girl. They have nothing in common. Hence, both subject and predicate are distributed.

3. Particular affirmative or I-type proposition.

Mobile Telephone

Take an example : Some mobiles are telephones. In this type of proposition subject and predicate have something in common. This implies that in I-type neither subject nor Mobiles Telephones predicate is distributed. We can see it graphically as given in figure.

4. Particular negative or O-type proposition.

Boys Students

Take an example : Some boys are not students. In O-type propositions some of the category represented by boys subject is not students, which means that a section of boys is denied with the entire category of students. It is, therefore, deduced that in O-type proposition only predicate is distributed. On account of different logical approach required to be applied for drawing each type of inference, a clear understanding of this difference becomes more important.

Page 15: Study Kit Brochure for SSC Combined Graduate Level ...static.upscportal.com/files/study-kit/ssc-cgl/SSC-CGL-2013... · Brochure for SSC Combined Graduate Level Examination - 2014

Click Here for SSC CGL Online Coaching: http://sscportal.in/community/courses/ssc-cgl-tier-1

[Click Here to Buy This Study Kit in Hard Copy: http://sscportal.in/community/study-kit/cgl ] 14

Rules for Mediate Inference First introduced by Aristotle, a syllogism is a deductive argument in which conclusion has to be drawn

from two propositions referred to as premises. Now consider an example.

Statement: I. Vinay is a boy. II. All boys are honest. Conclusion I. Vinay is honest.

First two sentences I and II are called propositions and the sentence I is called conclusion. This conclusion is drawn from above given two propositions.

Types of Questions Asked in the Examination There are mainly two types of questions which may be asked under this

1. When premises are in specified form Here premise is in specified form. Here mainly two propositions are given. Propositions may be particular to universal; universal to particular; particular to particular; universal to universal.

2. When premises are in jumbled/mixed form Here at least three or more than three proposition are given. Here pair of two propositions out of them follow as same as in specified form.

Type 1 Premises in Specified Forms Case 1: The conclusion does not contain the middle term Middle term is the term common to both the

premises and is denoted by M. Hence, for such case, conclusion does not contain any common term belong to both premises. Example 1 Statement: I. All men are girls. II. Some girls are students. Conclusions I. All girls are men. II. Some girls are not students.

Solution. Since, both the conclusions I and II contain the middle term ‘girls’ so neither of them can follow.

Venn diagram Representation: All possible cases can be drawn by using Venn diagram.

Girls

men

Students

or,

Girls Men Students

By using both representation (a) and (b) it is clear all girls cannot be men as well as (a) shows some

girls are students, here no man is included but at the same time (b) shows some girls are students have some men are also students as all men are girls. Hence, we cannot deduce conclusion II.

So, neither of them can follow. Example 2 Statement: I. All mangoes are chairs. II. Some chairs are tables. Conclusions I. All mangoes are tables. II. Some tables are mangoes. III. No mango is a table.

Page 16: Study Kit Brochure for SSC Combined Graduate Level ...static.upscportal.com/files/study-kit/ssc-cgl/SSC-CGL-2013... · Brochure for SSC Combined Graduate Level Examination - 2014

Click Here for SSC CGL Online Coaching: http://sscportal.in/community/courses/ssc-cgl-tier-1

[Click Here to Buy This Study Kit in Hard Copy: http://sscportal.in/community/study-kit/cgl ] 15

Solution. Here, the term chair is common to both the statement and hence, is the middle term. Statement (I) is A type proposition and in A-type proposition, only subject is distributed, hence, chair being the predicate in the statement (I) is not distributed in the second statement. Thus, none of the conclusions following statement is a valid inference.

Venn diagram representation: All possible cases can be drawn as

Chairs

MangoesTables

or,Chairs

Mangoes

Tables

(i) All mangoes are table-this inference is definitely false neither (a) nor (b) shows this conclusion. (ii) Some tables are mangoes, this inference is uncertain or doubtful. (iii) No mango is a table, this inference is also uncertain or doubtful. Though it can be concluded from the

above discussion that no valid inference can be drawn between mango and table. Case 2: No term can be distributed in the conclusion unless it is distributed in the premises.

If case 1 is compiled with by a pair of statement, it is confirmed that valid mediate inferences can be drawn

from such pair of statement. But every mediate inference drawn cannot be valid. Therefore, case 2 is applied to

check as to the conclusions drawn from a pair of statement in which middle term is distributed, is valid. Example 3 Statement: I. Some boys are students. II. All students are teenagers. Conclusions I. All teenagers are students. II. Some boys are teenagers.

Solution. Statement I is an I-type proposition which distributes neither the subject nor the predicate. Statement II is an A type proposition which distributes the subject ‘students’. Conclusion I is an A-type proposition which distributes the subject ‘teenagers’ only.

Since. the term teenagers is distributed in conclusion I without being distributed in the premises. So, conclusion I cannot follow. In second conclusion, where it is asked that some boys are teenagers. But from statement I it is clear that some students are not students. These students may not be teenagers.

Venn diagram representation: All possible cases can be drawn as follows

Boys Students

Teenagers

.

We have given that all students are teenagers so, its reverse cannot be possible. Hence, conclusion I is false. As we are also given that some boys are students and all students are teenagers. So, some boys which are students must be teenagers. Hence, conclusion II follows.

Page 17: Study Kit Brochure for SSC Combined Graduate Level ...static.upscportal.com/files/study-kit/ssc-cgl/SSC-CGL-2013... · Brochure for SSC Combined Graduate Level Examination - 2014

Click Here for SSC CGL Online Coaching: http://sscportal.in/community/courses/ssc-cgl-tier-1

[Click Here to Buy This Study Kit in Hard Copy: http://sscportal.in/community/study-kit/cgl ] 16

Case 3: If one premises is particular, conclusion is particular. Take an example which explains this case Example 4 Statement: I. Some boys are thieves. II. All thieves are dacoits. Conclusions I. Some boys are dacoits. II. All dacoits are boys.

Solution. Since, one premise is particular, the conclusion must be particular. So, conclusion II cannot follow.

Venn diagram representation: All possible cases can be drawn as follows

Boys Thieves

Dacoits

Here conclusion I follows but the conclusion II cannot follow. Case 4 If the middle term is distributed twice, the conclusion cannot be universal Take an example

which explains such case. Example 5 Statement: I. All Lotus are flowers. II. No Lily is a Lotus. Conclusions I. No Lily is flowers. II. Some Lilies are flowers.

Solution. Here, the first premise is an A proposition and so, the middle term ‘Lotus’ forming the subject is distributed.The second premise is an E proposition and so, the middle term ‘Lotus’ forming the predicate is distributed. Since, the middle term is distributed twice, so the conclusion cannot be universal.

Venn-diagram representation: All possible cases can be drawn as follows

Flowers

Lotus Lily

Lotus

Lotus

Flowers Flowers

Lily

It is clear from the given Venn-diagrams either conclusion I or II must be followed. Case 5 If both the premises are affirmative, the conclusions must be affirmative. Take an example

which follows such case: Example 6 Statement: I. All gardens are schools. II. All schools are colleges. Conclusions I. All gardens are colleges. II. Some gardens are not colleges.

Solution. Since, both the premises are affirmative, the conclusion must be affirmative, so conclusion II cannot follow.

Venn diagram representation: All possible cases can be drawn as follows.

Page 18: Study Kit Brochure for SSC Combined Graduate Level ...static.upscportal.com/files/study-kit/ssc-cgl/SSC-CGL-2013... · Brochure for SSC Combined Graduate Level Examination - 2014

Click Here for SSC CGL Online Coaching: http://sscportal.in/community/courses/ssc-cgl-tier-1

[Click Here to Buy This Study Kit in Hard Copy: http://sscportal.in/community/study-kit/cgl ] 17

Gardens

Schools

Colleges

Now, taking conclusion I, it is clear that all gardens are also colleges. But taking conclusion II, we cannot that derive second conclusion is true. Hence, only the first conclusion must be true.

Case 6 No conclusion follows. There are three types of such cases. (a) If both the premises are particular

Example 7 Statement: I. Some cups are spoons. II. Some spoons are sauccers. Conclusions I. All cups are sauccers. II. Some sauccers are cups.

Solution. Since both the premises are particular, so no definite conclusion follows. Venn diagram representation

CupsSpoons

Sauccers

Cups Sacuccers Spoons

It is clear from both given Venn-diagrams that no conclusion is followed. (b) If both the premises are negative

Example 8 Statement: I. No flower is mango. II. No mango is cherry. Conclusions I. No flower is cherry. II. Some cherries are mangoes.

Solution. Since, both the premises are negative hence, neither conclusion follows. Venn diagram representation: It is clear from both Venn-diagrams that neither conclusion follows. (c) I f the major premise is particular and the minor premise is negative Major premise is the predicate of the conclusion and minor premise is the subject of the conclusion.

Example 9 Statement: I. Some pubs are cows.

II. No kitten are pubs. Conclusions I. No pubs are kitten. II. Some cows are kitten.

Solution. Here, the first premise containing the middle term ‘Kitten’ as the subject is the major premise and the second premise containing the middle term ‘Kitten’ as the predicate is the minor premise. Since, the major premise is particular and the minor premise is negative. So, no conclusion follows.

Page 19: Study Kit Brochure for SSC Combined Graduate Level ...static.upscportal.com/files/study-kit/ssc-cgl/SSC-CGL-2013... · Brochure for SSC Combined Graduate Level Examination - 2014

Click Here for SSC CGL Online Coaching: http://sscportal.in/community/courses/ssc-cgl-tier-1

[Click Here to Buy This Study Kit in Hard Copy: http://sscportal.in/community/study-kit/cgl ] 18

Venn diagram representation: All possible cases are given

PutsCows

Kitten

Pubs Cows Kitten Pubs

Cows

Kitten

(a) (b) (c)

It is clear from the Venn-diagram representation that conclusion I follows.

Complementary pair of conclusions In drawing mediate inferences from given statement, students are required to be more attentive in

selecting complementary pair of conclusion where neither of the conclusions is definitely true but a combination of both makes a complementary pair. As we have already discussed in case number 1 that in the statement where middle term is not distributed, no valid mediate inference can be drawn but there still exists a possibility that a complementary pair of conclusions follows from the statement.

Example 10 Statements I. Some cameras are radios. II. Some statues are cameras. Conclusions I. Some ratios are statues. II. No radio is statue.

Solution. Either ‘some radios are statues’ or ‘No radio is statue’ follows, as I and E-type proposition form a complementary pair.

Venn diagram representation We can draw all possible cases as given below

Radios Cameras

Statues

Radios Cameras Statues

Hence, using both diagrammatical representation we can conclude either some radios are statues or ‘no radio’ is statue. Hence, at least one of the conclusions must be true.

Special Cases

Facts Combinations (Conclusion) Conclusion A + E Either I or II follows E + I Either I or II follows I + O Either I or II follows

Page 20: Study Kit Brochure for SSC Combined Graduate Level ...static.upscportal.com/files/study-kit/ssc-cgl/SSC-CGL-2013... · Brochure for SSC Combined Graduate Level Examination - 2014

Click Here for SSC CGL Online Coaching: http://sscportal.in/community/courses/ssc-cgl-tier-1

[Click Here to Buy This Study Kit in Hard Copy: http://sscportal.in/community/study-kit/cgl ] 19

E + O Either I or II follows

Example 11 Statements I. All vegetables are green. II. Some greens are fruits. Conclusions I. Some fruits are vegetables. II. No fruit is vegetable.

Solution. Here, conclusion I is particular affirmative and conclusion II is universal negative proposition. Hence, either conclusion I or conclusion II follows.

Alternatives

Green

Vegetables

Fruitor

Vegetables

Fruit

Green

Conclusion: If we follow Venn-diagram (a) then we can say no fruit is vegetable. Conclusion II but if we that follow Venn diagram (b) then we can say some fruits are vegetables (conclusion I).

Here, either Venn diagram (a) or Venn-diagram (b) is possible. Hence, conclusion I or conclusion II must be followed.

Minimal Possibilities We can represent statements by keeping in mind our conclusions It’ we follow that our two conclusions

belong to special case, then either one of them is true. We can represent minimum possibilities as given directly in the statement, we don’t need to think

about parameters.

Example 12 Statements I. Some fruits are vegetables. II. Some vegetables are junk food. III. Some junk foods are snacks. Conclusions I. Some junk foods are vegetables. II. Some junk foods are fruits.

Solution. Minimal possibilities

Fruit Vegetables Junk foods Snacks

It is clear from the above diagrams that only conclusion I follows.

Type 2 Mixed/jumbled Problems In such type of problem there is a constitution of mixed problems of universal and particular premises.

Page 21: Study Kit Brochure for SSC Combined Graduate Level ...static.upscportal.com/files/study-kit/ssc-cgl/SSC-CGL-2013... · Brochure for SSC Combined Graduate Level Examination - 2014

Click Here for SSC CGL Online Coaching: http://sscportal.in/community/courses/ssc-cgl-tier-1

[Click Here to Buy This Study Kit in Hard Copy: http://sscportal.in/community/study-kit/cgl ] 20

Directions (Q. Nos. 13 to 16) In each of the questions given below there are three statements followed by three conclusions numbered I, II and III, you have to take the given statements to be true even if they seem to be at variance from commonly known facts. Read all the conclusions and then decide which of the given conclusions logically follows from the given statements disregarding commonly known, facts.

Example 13 Statements All halls are tyres. Some tyres are wheels. All wheels are cars. Conclusions I. Some cars are wheels. II. Some cars are tyres. III. Some wheels are halls. (a) None follows (b) Only I follows (c) Only I and II follow (d) Only III follows

Solution. (c) Conclusions

Tyres

Halls

Cars

Wheels

I. Some cars are wheels. It can be seen from Venn-diagram. II. Also, some cars are tyres (as shown in Venn-diagram). III. It is not clear from the Venn-diagram. Hence, only I and II follow.

Example 14 Statements Some pictures are frames. Some frames are idols. All idols are curtains. Conclusions I. Some curtains are pictures. II. Some curtains are frames. III. Some idols are frames. (a) Only I and II follow (b) Only II and III follow (c) Only I and III follow (d) All follow Solution. (b)

Conclusions

Pictures

frames

idolsCurtains

Page 22: Study Kit Brochure for SSC Combined Graduate Level ...static.upscportal.com/files/study-kit/ssc-cgl/SSC-CGL-2013... · Brochure for SSC Combined Graduate Level Examination - 2014

Click Here for SSC CGL Online Coaching: http://sscportal.in/community/courses/ssc-cgl-tier-1

[Click Here to Buy This Study Kit in Hard Copy: http://sscportal.in/community/study-kit/cgl ] 21

I. Does not follow. II. Follows (clear from the Venn-diagram) III. Follows (clear from the Venn-diagram)

Hence, only II and III follow.

Example 15 Statements Some ice are rings. No ring is paint. Some rings are gold. Conclusions I. No gold is paint. II. No ice is gold. III. Some rings are paints. IV. All golds are rings. (a) Only I and III follow (b) Only I and II follow (c) Only III and IV follow (d) None follows

Solution. (d) Hence, none of the conclusions follows.

Ice Rings

Gold

Paint

Example 16 Statements No candle is bell. Some shoes are bells. All tables are shoes. Conclusions I. Some tables are bells. II. No table is bell. III. Some shoes are candles. IV. No shoes are candles. (a) Only I and IV follow (b) Only I and II follow (c) Only III and IV follow (d) None of these

Solution. (d)

Candles BellsTable

Shoes

Solution. (d) Here conclusion I is particular affirmative and conclusion II is its universal negative. Hence,

either conclusion I or II follows. Also conclusion III is particular affirmative and conclusion IV is its universal negative. Hence, either III or conclusion IV follows. Finally, either conclusion I or II follows and either conclusion III or conclusion IV follows.

EXERCISE: Directions (Q. Nos. 1 to 13) In each question given below are two Statements followed by two conclusions numbered I and II. You have to take the two given Statements to be true even if they

Page 23: Study Kit Brochure for SSC Combined Graduate Level ...static.upscportal.com/files/study-kit/ssc-cgl/SSC-CGL-2013... · Brochure for SSC Combined Graduate Level Examination - 2014

Click Here for SSC CGL Online Coaching: http://sscportal.in/community/courses/ssc-cgl-tier-1

[Click Here to Buy This Study Kit in Hard Copy: http://sscportal.in/community/study-kit/cgl ] 22

seem to be at variance from commonly known facts and decide which of the given conclusions logically follows from the the given two Statements, disregarding commonly known facts. Give answer as (a) If only conclusion I follows (b) If only conclusion II follows (c) If neither I nor II follows (d) If both I and II follow 1. Statement: I. All jungles are tigers. II. Some tigers are horses. Conclusions I. Some tigers are jungles. II. All horses are jungles. 2. Statement: I. All birds are tall. II. Some tall are hens. Conclusions I. Some birds are hens. II. Some hens are tall. 3. Statement: I. All artists are smokers. II. Some smokers are drinkers. Conclusions I. All smokers are artists. II. Some drinkers are not smokers. 4. Statement: I. Some hens are cows. II. All cows are horses. Conclusions I. Some horses are hens. II. Some hens are horses. 5. Statement: I. All buses are cars. II. Some cars are roads. Conclusions I. Some cars are buses. II. Some buses are roads. 6. Statement: I. Some pastries are toffees. II. All toffees are chocolates. Conclusions I. Some chocolates are toffees. II. Some toffees are not pastries.. 7. Statement: I. Some players are singers. II. All singers are tall. Conclusions I. Some players are tall. II. All players are tall. 8. Statement: I. All stones are water. II. Some water are clean. Conclusions I. Some water are stones. II. All clear are water. 9. Statement: I. Some phones are watches. II. All watches are guns. Conclusions I. All guns are watches. II. Some guns are phones. 10. Statement: I. All umbrellas are aeroplanes. II. Some aeroplanes are birds. Conclusions I. Some umbrellas are aeroplanes. II. All birds are umbrellas. 11. Statement: I. Some scooters are trucks.

Page 24: Study Kit Brochure for SSC Combined Graduate Level ...static.upscportal.com/files/study-kit/ssc-cgl/SSC-CGL-2013... · Brochure for SSC Combined Graduate Level Examination - 2014

Click Here for SSC CGL Online Coaching: http://sscportal.in/community/courses/ssc-cgl-tier-1

[Click Here to Buy This Study Kit in Hard Copy: http://sscportal.in/community/study-kit/cgl ] 23

II. All trucks are trains. Conclusions I. Some scooters are trains. II. No truck is a scooter. 12. Statement: I. All tigers are ships. II. Some ships are cupboards. Conclusions I. Some ships are tigers. II. Some cupboards are not ships. 13. Statement: I. All books are pencils. II. Some pencils are cycles. Conclusions I. Some cycles are pencils. II. Some cycles are books.

ANSWERS: 1. (a) 2. (b) 3. (b) 4. (d) 5. (a) 6. (d) 7. (a) 8. (a) 9. (b) 10. (c) 11. (a) 12. (d) 13. (a)

EXPLAINATIONS: (1)

Tigers

Jungles Horses

Hence, only I follows. (2)

Tall

Birds Hens

Hence, from both Venn-diagram conclusion II follows. (3)

Smokers

Aritist Drinkers

Hence, conclusion I follows.

Page 25: Study Kit Brochure for SSC Combined Graduate Level ...static.upscportal.com/files/study-kit/ssc-cgl/SSC-CGL-2013... · Brochure for SSC Combined Graduate Level Examination - 2014

Click Here for SSC CGL Online Coaching: http://sscportal.in/community/courses/ssc-cgl-tier-1

[Click Here to Buy This Study Kit in Hard Copy: http://sscportal.in/community/study-kit/cgl ] 24

(4)

Hens Cows

Horses

Hence, both conclusions are true. (5)

Cars

Buses

Roads

Roads

Hence, only I follows. (6)

Pastries Chocolates

Toffees

Hence, both I and II are true. (7)

Players Singers

Tall

Hence, some players are tall, only I follows. (8)

Water

Stones Clean

Hence, only I follows. (9)

Page 26: Study Kit Brochure for SSC Combined Graduate Level ...static.upscportal.com/files/study-kit/ssc-cgl/SSC-CGL-2013... · Brochure for SSC Combined Graduate Level Examination - 2014

Click Here for SSC CGL Online Coaching: http://sscportal.in/community/courses/ssc-cgl-tier-1

[Click Here to Buy This Study Kit in Hard Copy: http://sscportal.in/community/study-kit/cgl ] 25

Phones Watches

Guns

Hence, only II follows. (10)

Aeroplanes

Umbrellas Birds

Hence, none follows. (11)

Scooters Trucks

Trains

Hence, only I follows. (12)

Ships

Tigers Cupboards

Hence, both follow. (13)

Pencils

Books Cycles

Hence, only I follows.

Page 27: Study Kit Brochure for SSC Combined Graduate Level ...static.upscportal.com/files/study-kit/ssc-cgl/SSC-CGL-2013... · Brochure for SSC Combined Graduate Level Examination - 2014

Click Here for SSC CGL Online Coaching: http://sscportal.in/community/courses/ssc-cgl-tier-1

[Click Here to Buy This Study Kit in Hard Copy: http://sscportal.in/community/study-kit/cgl ] 26

Section 2 : General Knowledge & Awareness

Indian Polity

Preamble

WE, THE PEOPLE OF INDIA, having solemnly resolved to constitute India into a SOVEREIGN SOCIALIST SECULAR DEMOCRATIC REPUBLIC and to se cure to all its citizens:

JUSTICE, social, economic and political;

LIBERTY of thought, expression, belief, faith and worship;

EQUALITY of status and of opportunity;

and to promote among them all

FRATERNITY assuring the dignity of the individual and the unity and integrity of the Nation;

IN OUR CONSTITUENT ASSEMBLY this twenty-sixth day of November, 1949, do HEREBY ADOPT, ENACT AND GIVE TO OURSELVES THIS CONSTITUTION.

EXERCISE: 1. The constitution assembly adopted the constitution on.

(a) Twenty sixth day of January, 1950

(b) Twenty sixth day of November, 1949

(c) Twenty sixth day of January, 1949

(d) None of the above.

2. Who adopted the constitution of India on 26th November 1949.

(a) People of India

(b) Parliament of India

(c) Representative of the people of India in constituent Assembly

(d) Cabinet

3. Consider the following

(i) Socialist

(ii) Democratic

(iii) Sovereign

(iv) Secular

Choose the words in the order that they appear in the Preamble

a) iii, i, iv, ii

b) iii, iv, i, ii

Page 28: Study Kit Brochure for SSC Combined Graduate Level ...static.upscportal.com/files/study-kit/ssc-cgl/SSC-CGL-2013... · Brochure for SSC Combined Graduate Level Examination - 2014

Click Here for SSC CGL Online Coaching: http://sscportal.in/community/courses/ssc-cgl-tier-1

[Click Here to Buy This Study Kit in Hard Copy: http://sscportal.in/community/study-kit/cgl ] 27

c) iii, iv, ii, i

d) iv, i, iii, ii

4. What represents thought, expression, belief faith and worship

(a) Justice

(b) Liberty

(c) Fraternity

(d) Equality

5. Which of the following Ammendment acts amended the Preamble?

(a) 44th Amendment

(b) 42 Amendment

(c) 56th Amendment

(d)It has never been amended

ANSWERS: 1.(b) 2. (a) 3. (a) 4. (a) 5. (c)

The Union and Its Territory

Name and territory of the Union

1. (1) India, that is Bharat, shall be a Union of States. (2) The States and the territories thereof shall be as specified in the First Schedule. (3) The territory of India shall comprise— (a) the territories of the States; (b) the Union territories specified in the First Schedule; and (c) such other territories as may be acquired.

Admission or establishment of new States. 2. Parliament may by law admit into the Union, or establish, new States on such terms and conditions

as it thinks fit. 2A. Sikkim to be associated with the Union. Rep. by the Constitution (Thirty-sixth Amendment) Act,

1975, s. 5 (w.e.f. 26-4-1975).

Formation of new States and alteration of areas, boundaries or names of existing States

3. Parliament may by law— (a) form a new State by separation of territory from any State or by uniting two or more States or parts

of States or by uniting any territory to a part of any State; (b) increase the area of any State; (c) diminish the area of any State; (d) alter the boundaries of any State; (e) alter the name of any State: Provided that no Bill for the purpose shall be introduced in either House of Parliament except on the

recommendation of the President and unless, where the proposal contained in the Bill affects the area,

Page 29: Study Kit Brochure for SSC Combined Graduate Level ...static.upscportal.com/files/study-kit/ssc-cgl/SSC-CGL-2013... · Brochure for SSC Combined Graduate Level Examination - 2014

Click Here for SSC CGL Online Coaching: http://sscportal.in/community/courses/ssc-cgl-tier-1

[Click Here to Buy This Study Kit in Hard Copy: http://sscportal.in/community/study-kit/cgl ] 28

boundaries or name of any of the States, the Bill has been referred by the President to the Legislature of that State for expressing its views thereon within such period as may be specified in the reference or within such further period as the President may allow and the period so specified or allowed has expired.

Explanation I.—In this article, in clauses (a) to (e), "State'' includes a Union territory, but in the proviso, "State'' does not include a Union territory.

Explanation II.—The power conferred on Parliament by clause (a) includes the power to form a new State or Union territory by uniting a part of any State or Union territory to any other State or Union territory.

Laws made under articles 2 and 3 to provide for the amendment of the First and the Fourth Schedules and supplemental, incidental andc consequential matters.

4. (1) Any law referred to in article 2 or article 3 shall contain such provisions for the amendment of the First Schedule and the Fourth Schedule as may be necessary to give effect to the provisions of the law and may also contain such supplemental, incidental and consequential provisions (including provisions as to representation in Parliament and in the Legislature or Legislatures of the State or States affected by such law) as Parliament may deem necessary. (2) No such law as aforesaid shall be deemed to be an amendment of this Constitution for the purposes of

article 368.

EXERCISE: 1. The Union territories are specified in the _____ schedule

(a) 2nd

(b) 1st

(c) 3rd

(d) 4th

2. Sikkim was associated with the Union. Rep. by the Constitutions

(a) 36th Amendment Act

(b) 44th Act

(c) 42nd Act

(d) 56th Act

3. Which of the following are correct regarding the formation of states and its alterations, The parliament by law

(i) Form a new State by separation of territory from any State or by uniting two or more State

(ii) diminish the area of any State;

(iii) alter the boundaries of any State

(iv) alter the name of any State

(a) i, ii and iv

(b) i, ii, iii,

(c) ii and iv

(d) all of the above

4. The Union of India consists of

i) States

ii) Union territories

iii) Acquired territories

(a) i

Page 30: Study Kit Brochure for SSC Combined Graduate Level ...static.upscportal.com/files/study-kit/ssc-cgl/SSC-CGL-2013... · Brochure for SSC Combined Graduate Level Examination - 2014

Click Here for SSC CGL Online Coaching: http://sscportal.in/community/courses/ssc-cgl-tier-1

[Click Here to Buy This Study Kit in Hard Copy: http://sscportal.in/community/study-kit/cgl ] 29

(b) ii

(c) i and ii

(d) i, ii and iii

5. Which of the following are correct?

i) The territory and the units of the Indian federation may be altered by Parliament by a single majority

ii) The consent of the state legislature is necessary before the parliament redistributes its territory

iii) The presidents recommendation is necessary for introducing in parliament a bill on redistributing territory of any state

(a) i and ii

b) ii and iii

(c) i and iii

d) None

6. Which of the following state/ Union territories has a legislative assembly consisting of only 30 members

(a) Puducherry

(b) Mizoram

(c) Goa

(d) All

ANSWERS: 1. (b) 2. (a) 3. (d) 4. (d) 5. (c) 6. (a)

Geography

ASIA

Introduction

Area: 43,608,000 million sq. km (30% of total land surface of the earth.)

Population : 3588.9 million

Latitudes : 10011'S to 81012'N

Only some of the Indonesian group of Islands is located to the south of equator in the Southern Hemisphere.

Longitude : 2602'E to 169040'W in the east crossing 1800 longitude.

North-South Extent : 6,440 km East-West Extent : 9,650 km

Page 31: Study Kit Brochure for SSC Combined Graduate Level ...static.upscportal.com/files/study-kit/ssc-cgl/SSC-CGL-2013... · Brochure for SSC Combined Graduate Level Examination - 2014

Click Here for SSC CGL Online Coaching: http://sscportal.in/community/courses/ssc-cgl-tier-1

[Click Here to Buy This Study Kit in Hard Copy: http://sscportal.in/community/study-kit/cgl ] 30

• Greatest extent, North-South : 6440 km

• Greatest extent, East-West : 9650 km

Size : Largest continent in the world, 13 times larger than India.

Situation : The continent of Asia is situated entirely in the northern hemisphere except some of the islands of Indonesia. To the north of it lies the Arctic Ocean, a frozen sea, to the east the Pacific Ocean, to the south the Indian Ocean and to the west lies Mediterranean Sea. It is separated from Europe by the Ural Mountains, the Caspian Sea, the Black Sea, the Caucasus Mountains and the Strait of Dardanelles in the west and from Africa by the Red Sea and the Isthmus of Suez while the Bering Strait separates it from North America.

Asia—Physical

Important Seas

Name Location Part of Ocean

Kara Sea, Laptev Sea, and East Siberian Sea North of Russia Arctic Ocean

Bering Sea Northerst of Russia Pacific Ocean

Sea of Okhotsk East of Russia Pacific Ocean

Sea of Japan West of Japan Pacific Ocean

Yellow Sea West of Korea Pacific Ocean

East China Sea East of China Pacific Ocean

South China Sea South of China Pacific Ocean

Sulu Sea West of the Philippines Island Pacific Ocean

Celebes Sea North of Celebes Island Pacific Ocean

Banda Sea East of Celebes Island Pacific Ocean

Flores Sea South of Celebes Sea Pacific Ocean

Page 32: Study Kit Brochure for SSC Combined Graduate Level ...static.upscportal.com/files/study-kit/ssc-cgl/SSC-CGL-2013... · Brochure for SSC Combined Graduate Level Examination - 2014

Click Here for SSC CGL Online Coaching: http://sscportal.in/community/courses/ssc-cgl-tier-1

[Click Here to Buy This Study Kit in Hard Copy: http://sscportal.in/community/study-kit/cgl ] 31

Molucca Sea East of Celebes Island Pacific Ocean

Java Sea North of Java Pacific Ocean

Timor Sea Northwest of Australia Pacific Ocean

Arafura Sea North of Australia South Pacific Ocean

Bay of Bengal East of the Indian Peninsula Indian Ocean

Arabian Sea West of the Indian Peninsula Indian Ocean

Red Sea Separates Asia from Africa Indian Ocean Important Gulfs Name Location Part of Ocean

Gulf of Ob Between Yamal Peninsula and Gyda Peninsula Arctic Ocean

Gulf of Chihli East of China Yellow Sea (Pacific Ocean)

Gulf of Tonkin Eastof Vietnam South China Sea (Pacific Ocean)

Gulf of Thailand South of Thailand South China Sea

Persian Gulf Separates Arabian Peninsula from the Plateau of Iran Indian Ocean

Gulf of Oman Between Iranian Plateau and Oman Indian Ocean

Gulf of Aden Between Somalia and Yemen Indian Ocean

Gulf of Aquaba Between Aquaba (Jordan) and Sinai Peninsula (Egypt). Red Sea

Severnaya Islands North of Russia Arctic Ocean

New Siberian Islands North of Russia Between Laptev Sea and East Siberian Sea, Arctic Ocean

Wrangel Islands North of Russia East Siberian Sea, Arctic Ocean

Name Location Part of Ocean

Kurile Islands Between Kamchatka Peninsula and Hokkaido Island Pacific Ocean

• An extension of the volcanic Kamchatka Peninsula

Ryukyu Island Between Kyushu and Taiwan Pasific Ocean

• An example of sub- merged island.

Bonin Island East of Ryuky Island Pasific Ocean

Babuyan Islands Between Taiwan and Luzon (Philippines) Pasific Ocean

Spartly and Parcel Islands A group of atolls in South China Sea Pasific Ocean

• A disputed group of islands claimed by China, Vietnam, Malaysia, Taiwan, Philippines, Brunei because of the vast

Page 33: Study Kit Brochure for SSC Combined Graduate Level ...static.upscportal.com/files/study-kit/ssc-cgl/SSC-CGL-2013... · Brochure for SSC Combined Graduate Level Examination - 2014

Click Here for SSC CGL Online Coaching: http://sscportal.in/community/courses/ssc-cgl-tier-1

[Click Here to Buy This Study Kit in Hard Copy: http://sscportal.in/community/study-kit/cgl ] 32

reserves of oil beneth these islands.

The Philippines Lying 1200 km east of Vietnam and 150 north of the Pasific Ocean archipelago equator.

Luzon.

• The largest and the important island of the Philippines is known as the ‘Rice Bowl of Philippines’.

• Mount Mayon, the only active volcano of the Philippines, is located in the Luzon.

• Good quality cigar tobaco is grown in the northern Luzon (mainly Cagayan Valley).

• Igorot tribe lives in the northern region of island. Mindano Island • Second largest island located in southern part of the

Philippines. • Mount Apo, a dormant volcano on Mindano, is the

country’s highest peak.

Important Straits

Name Separates Connects

Bering Strait Asia and North America East Siberian Sea with Bering Sea

La Parouse Strait Sakhalin Island and Hokkaido Island Sea of Okhotsk with Sea of Japan

Tata Strait Eastern Russia and Sakhalin Sea of Okhotsk with Sea of Japan

Korea Strait South Korea and Kyushu (Japan) Yellow Sea with Sea of Japan

Formosa Strait Taiwan and China East China Sea with South China (Taiwan Strait) Sea

Luzon Strait Taiwan and Luzon (Philippines) South China Sea with Pacific Ocean.

Name Separates Connects

Makassar Strait Borneo (Kalimantan) and Celebes Celebes Sea with Java Sea. Island

Sundra Strait Java and Sumatra Java Sea with India Ocean

Malacca Strait Malaya Peninsula and Sumatra Java Sea with Bay of Bengal • One of the great ship- ping corridors of the world

Strait of Jahore Singapore and Malaysia South China Sea with strait of Malacca

Strait of Hormuz UAE and Iran Persian Gulf with Gulf of Oman.

Strait of Bosporus Asia and Europe Black Sea with Sea of Marmara.

Strait of Dardanelles Asia and Europe Sea of Marmara with Mediterranean Sea

Important Mountains

The Himalayan Mountain Range • Lies to the north of Indo Gangetic Plain, is an example of mountain range which formed in the same age with same process.

• An example of fold mountain of Alpine orogeny.

Page 34: Study Kit Brochure for SSC Combined Graduate Level ...static.upscportal.com/files/study-kit/ssc-cgl/SSC-CGL-2013... · Brochure for SSC Combined Graduate Level Examination - 2014

Click Here for SSC CGL Online Coaching: http://sscportal.in/community/courses/ssc-cgl-tier-1

[Click Here to Buy This Study Kit in Hard Copy: http://sscportal.in/community/study-kit/cgl ] 33

• Mount Everest (8,848 m) is the highest peak, located in Nepal.

Karakoram Range • Lies in the north of the Himalaya. • Godwin Austen (K2) is the highest peak of Karakoram.

Kailash Range • East of Karakoram Range in Tibet.

Kunlun Shan Range • Lies to the north of Tibet Plateau and to the South of the desert basin of Tarim in China.

Tienshan • Lies to the north of Tarim basin.

Great Khingan Mountain • The Tien Shan extends to the north east and reaches the Amur River under the name of Great Khingan Mountains.

Altai Mountain • Lies to the north of Tienshan in succession in a more or less east-west direction.

Hangay Mountain Sayan Mountain

Yablonovy Range, • Lies to the north east of Atlai Range in the eastern Siberia. Stanovoy Range, Dzhugzur Range and Kolyama Range

Verkhoyansk Range • Lies to the east of the Lena River separates Central Siberian Plateau from the eastern Siberian regio.

Pegu Yoma • The Irrawaddy and Sittang are divided in their middle course by this mountain range.

Arakan Yoma • Running southwards, is the western range of Myanmar.

• A range of the Himalaya which further countinues through the Andaman and Nicobar Islands, Sumatra, Java and other Indonesian islands.

Page 35: Study Kit Brochure for SSC Combined Graduate Level ...static.upscportal.com/files/study-kit/ssc-cgl/SSC-CGL-2013... · Brochure for SSC Combined Graduate Level Examination - 2014

Click Here for SSC CGL Online Coaching: http://sscportal.in/community/courses/ssc-cgl-tier-1

[Click Here to Buy This Study Kit in Hard Copy: http://sscportal.in/community/study-kit/cgl ] 34

The Mountain Regions to the West of the Pamir

Hindukush Mountains • Running westwards from the Pamir Knot to the Elburz Mountains in Iran.

Elburz Mountains • Hindukush Mountain joins the Armenian Knot near the Caspian Sea under the name of Elburz Mountains in Iran.

Sulaiman Range • It proceeds southwards from the Pamir Knot along the border between Pakistan and Afganistan.

• The most famous pass across this mountain wall is the Khyber, linking Pakistan with Afghanistan.

Kirthar Range • Sulaiman Range takes the name of Kirthan in the south. Makran Range • Kirthan Range turns westward on the Arabian Sea coast and assumes

the name of Makran. Zagros • Running to the northwest of Makran, ultimately ends up in the

Armenian Knot. Pontic Mountain Range • Running to the northwest of Armenian Knot in Turkey.

Taurus Mountain Range • Running to the southwest of Armenian Knot along the Mediterranean coast in Turkey.

Important Plteaus and Basins

Page 36: Study Kit Brochure for SSC Combined Graduate Level ...static.upscportal.com/files/study-kit/ssc-cgl/SSC-CGL-2013... · Brochure for SSC Combined Graduate Level Examination - 2014

Click Here for SSC CGL Online Coaching: http://sscportal.in/community/courses/ssc-cgl-tier-1

[Click Here to Buy This Study Kit in Hard Copy: http://sscportal.in/community/study-kit/cgl ] 35

NAME INFORMATION

Ladakh Plateau (Inter- • Lies between the Karakoram in the north and the Himalayas in the montane Plateau) south.

Tibet Plateau (Inter- • Enclosed to the south by the Himalays, to the north by the Kunlun montane Plateau) Shan Range is the highest tableland (4,800 m) of Tibet.

• The largest plateau in the world with an average altitude of 4,250 m.

Yunan Plateau • Lies to the southeast of the Tibet Plateau is separated from the fertile Szechuan basin by the range of mountains.

Takla Makan Desert • An intermontane desert basin lies between the Tienshan in the north Plateau and Tarim Basin and Kunlun Shan in the south.

• Convectie and ‘Karaburan’ blows in the arid regions.

Pamir Plateau • Situated to the northwest of the Indian sub-continent.

• Meeting point for the central Asian mountain ranges which make up the skeleton of the continent.

• The highest plateau of the world with an altitude of 4,900 m is also known as ‘roof of the world’.

Armenian Plateau • Situated between the Caspian Sea and Black Sea.

• Pamir-like plateau from where many mountain ranges spread out, therefore called the Armenian Knot.

Iranian Plateau • The central desert plateau of Iran is enclosed by Zagros in the south and west and Elburz in the north.

Mongolian Plateau • A high plateau between 1,500 and 1,800 m above sea level, is situated between the Khinghan in the east, Altunshan and Tien Shan in the south, Altai in the west and Yablonovy in the north.

Urfan Basin • Lies to the west of Mongolian Plateau.

Aldan Plateau • Lies to the north west of Stanovoy Mountain.

Indo-China Plateau • Region of Southeast Asia, extends over Vietnam, Laos, Cambodia, Thailand and Myanmar.

Shan Plateau • Lies between Pegu Yoma and Arkan Yoma in the eastern part of Myanmar.

• The Salween River flows through the plateau.

• Region important for precious stone (Rubies), silver, lead, and tin and tungsten.

Deccan Plateau • Lies to the south of Indo Gangetic plain, between the Eastern Ghats and Western Ghats.

• North western part of Deccan Plateau is an example of lava Plateau or Plateau of accumulation.

Baluchistan Plateau • A desert plateau (900 m high) situated to the west of Kirthar Range.

NAME INFORMATION

Arabian Plateau • An example of titled block extending over the entire Arabian Peninsula.

Page 37: Study Kit Brochure for SSC Combined Graduate Level ...static.upscportal.com/files/study-kit/ssc-cgl/SSC-CGL-2013... · Brochure for SSC Combined Graduate Level Examination - 2014

Click Here for SSC CGL Online Coaching: http://sscportal.in/community/courses/ssc-cgl-tier-1

[Click Here to Buy This Study Kit in Hard Copy: http://sscportal.in/community/study-kit/cgl ] 36

Plateau of Antolia of Asia • Enclosed between Pontic Mountain Ranges in the north west and Minor or Turkey Tauras in the south west.

• A vast plateau with an average height of 1,200 m, broken by ridges and volcanic outcrops.

• The main rivers draining off the plateau are Tigris and Euphrates.

Loess Plateau • The Plateau region is surrounded by Great Plain in the east, Qining Mountains in the south and Ordos Plateau in the north.

• Loess is the wind borne fine dust, deposited beyond the deserts limits and is the fine loam, rich in lime.

• Shansi and Shensi region of loess plateau is known for its extensive coal reserves.

• The Hwang Ho flows through loess land.

Dzungarian Basin • Relatively low lying land between the Tien Shan and Altai Range in Mongolia.

• A sort of door between China and Mongolia in the east and Tarim lowland in the west.

Mesopotamian Plain • Formed by the Tigris and Euphrates River in Iraq.

• One of the major regions where wheat, barley, tobacco and cotton is grown.

Gobi Desert • Situated to the southeast of Mongolians Plateau and extends into China.

• Aridity due to interior location shows the features of undulating sand sea as well as barren sheets or rock and stone.

Important Land Regions

Manchurian Plain • Formed by the Amur River and its tributaries in the northern part of China.

• Important cities Anshan, Shenyag and Fushun of Mukden Triangle are located.

Great Plain of China • Formed by the Hwang-Ho and Yangtze Kiang Rivers in the eastern China.

Turanian Plain • Formed by the two principal Asian rivers, the Amu Darya and Syr Darya.

• It is an example of Basin plain which covers the region of Turkmenistan, Uzbekistan and Kazakhstan.

West Siberian Plain • It extends over 3,200 km eastwards from the Urals to the Yenisey River and is one of the largest lowlands in the world.

Taaidam Basin • Lies between Altun Shan and Kunlun Shan and to the north east of Central Tibetan Plateau.

• The major region of oil fields.

Szechuan Basin • Situated to the north of Yunan Plateau is the fertile basin where the Yangtze Kiang River is joined by three major tributaries known as the Red Basin because of its coloured sandstone.

• Red basin of Szechuan is the leading rice producing region and is called the ‘Rice Bowl of China’.

Page 38: Study Kit Brochure for SSC Combined Graduate Level ...static.upscportal.com/files/study-kit/ssc-cgl/SSC-CGL-2013... · Brochure for SSC Combined Graduate Level Examination - 2014

Click Here for SSC CGL Online Coaching: http://sscportal.in/community/courses/ssc-cgl-tier-1

[Click Here to Buy This Study Kit in Hard Copy: http://sscportal.in/community/study-kit/cgl ] 37

THE DESERT LANDSCAPE OF SAUDI ARABIA

Rub-al-Khali • Largest continuous sand desert in the southern part of Saudi Arabia.

• It is also called ‘Empty Quarter’.

Al Nafud Desert • Deserts land in the north eastern region of Saudi Arabia.

THE DESERT LANDSCAPE OF IRAN

Dash-I-Kavir Largest Salt desert of the world situated in the northern Iran.

Dasht-I-Lut Barren desert of Iran situated in the eastern part of country.

Important Lakes

NAME INFORMATION

Lake Baikal • Located in the southern part of eastern Siberia and to the west of Yablonovy Range in Russia.

• Source of the Lena River.

• It is the deepest continental body of water on Earth having a maximum depth of 1,620 m.

Lake Balkash • Located in Kazakhastan.

• North of the Lake is Karaganda coal basin.

Aral Sea • Located between Kazakhstan and Uzbekistan

• Two principal Asian rivers, the Amu Darya and Syr Darya flow through semi arid depression into the sea.

Lake Van Golu • The largest lake of Turkey.

• One of the saltiest lake in the world.

Lake Turnool • The northernmost extent of East African Rift Valley.

• Located in the Anatolia Peninsula of Turkey.

Lake Asad • Located in Syria.

Dead Sea • It is an example of rift valley lake which lies along the West Bank of Jordan.

• One of the deepest points of Asia is the world’s lowest lake.

• The Jordan river flows into the Dead Sea, which has no outlet, and evaportation balances the inflow.

Sea of Gallilee • Lava flow blocked the Jordan Valley and formed an elongated lake in Israel.

NAME INFORMATION

• The lave-blocked lake is linked to the Dead Sea through the Jordan River.

Lake Tonle Sap • Located in central lowland of Cambodia.

Lake Toba • Situated in Sumatra (Indonesia).

• An example of crater of caldera lake.

Lop Nor • Salt lake in China, site of numerous nuclear testing.

Page 39: Study Kit Brochure for SSC Combined Graduate Level ...static.upscportal.com/files/study-kit/ssc-cgl/SSC-CGL-2013... · Brochure for SSC Combined Graduate Level Examination - 2014

Click Here for SSC CGL Online Coaching: http://sscportal.in/community/courses/ssc-cgl-tier-1

[Click Here to Buy This Study Kit in Hard Copy: http://sscportal.in/community/study-kit/cgl ] 38

• Tarim river drains into the lake.

Caspian Sea • The largest lake in the world which is five times larger than the Lake Superior.

• It separates Europe from Asia.

Important Rivers

NAME SOURCE OUTFLOW

NORTH FLOWING RIVERS FROM WEST TO EAST IN RUSSIA

Ob Altai Mountain Gulf of Ob

Tributaries : Irtysh, Tobolsk

Yenisey Tannuala Mountain Arctic Ocean

Lena Lake Baikal Arctic Ocean

Kolyma Kolyma Range East Siberain Sea

EAST FLOWING RIVERS (FROM NORTH TO SOUTH)

Amur Confluence of Shika Tatar Strait Tributary : Sungari River. Argun River.

Yalu Korea Bay (Yellow Sea)

• Forms a well defined border between North Korea and the Eastern Russia.

Hwang Ho Tibetan Plateau The river’s course was diverted away into the Gulf of Pohai instead of the Yellow Sea.

Yangtse Kiang Tibetan Plateau East China Sea.

• It is the longest river of Asia.

• It flows through Szechuan basin.

• Shangai and Wuhan, are the important cities of China lying along the river.

Si Kiang Eastern Yunan South China Sea. (China) • More than half of the total silk production in China comes from the Yangtse Kiang and Si Kiang deltas.

EAST FLOWING RIVERS (FROM NORTH TO SOUTH)

• About 75% of the Chinese live in fertile river basin of Hwang Ho, Yangtse Kiang.

SOUTH FLOWING RIVERS

Mekong Tibetan highlands South China Sea

Page 40: Study Kit Brochure for SSC Combined Graduate Level ...static.upscportal.com/files/study-kit/ssc-cgl/SSC-CGL-2013... · Brochure for SSC Combined Graduate Level Examination - 2014

Click Here for SSC CGL Online Coaching: http://sscportal.in/community/courses/ssc-cgl-tier-1

[Click Here to Buy This Study Kit in Hard Copy: http://sscportal.in/community/study-kit/cgl ] 39

• It flows through China, Thailand-Laos border, Cambodia and Vietnam over 4,160 km to South China Sea.

• Longest river in South East Asia.

• In Laos, the Mekong forms the western boundary with Thailand.

• Phnom Penh and Ho Chi Minch cities are situated on the bank of river.

• Mekong valley of south Vietnam is devoted to rice cultivation.

• Delta: Arcuate, is (characterised by funnel shaped distributaries, stabled by growth of mangrove vegetation) lying in Vietnam.

Chao Phraya Gulf of Thailand

City located : Bangkok

• The principal river of Thailand.

• The river’s delta has rich alluvial soil which is used almost entirely for rice and has very high density of population.

Salween Tibetian highlands Gulf of Martaban

• It flows through China & Shan Plateau (Myanmar) over 2,090 km.

• Longest river of Myanmar.

Irrawaddy North Myanmar Bay of Bengal

Tributary : Chindwin

City located : Mandalay

• Major river of the country referred to as the ‘Life lije of Myanmar’

• Central basin between Irrawaddy and Chindwin around Mandalay is very important for wheat and cotton cultivation.

• Delta region of Irrawaddy and Sittang, known as the ‘Rice Bowl of Mayanmar’.

Brahmaputra Rises in galcier Bay of Bengal • Brahmputra is called Tsang-Po in Tibet before about 100 km south entering in India. east of Mansarovar Lake

SOUTH FLOWING RIVERS

• It enter Bangladesh near Bhubri.

Ganga Gangotri Glacier Bay of Bengal

• In the upper course Alaknanda and Bhagirathi River meet at Devprayag and from Ganga

Indus Mansarovar lake Arabian Sea

Page 41: Study Kit Brochure for SSC Combined Graduate Level ...static.upscportal.com/files/study-kit/ssc-cgl/SSC-CGL-2013... · Brochure for SSC Combined Graduate Level Examination - 2014

Click Here for SSC CGL Online Coaching: http://sscportal.in/community/courses/ssc-cgl-tier-1

[Click Here to Buy This Study Kit in Hard Copy: http://sscportal.in/community/study-kit/cgl ] 40

• One of the world’s largest rivers flow south-east- wards acros the country then drain into Arabian Sea through Attock plains of Pakistan.

Tigris and Eupharates Persian Gulf

• These two rivers dominate Iraq and flow south-east- wards across the country then drain into the Persian Gulf via a combined estuary.

• They meet at Shatt-al-Arab in Iraq.

Amu Darya and Syr Darya

• The principal rivers of Central Asia flow in to the Aral Sea through the semi and depression.

EXERCISE: 1. Match the following. Sea Part of Ocean i) Kara Sea a) Pacific Ocean ii) Bering Sea b) South Pacific iii) Arafura Sea c) Arctic iv) Red Sea d) Indian

a) i- a, ii-b, iii-c, iv-d b) i- c, ii-a, iii-b, iv-d c) i-c, ii-d, iii-b, iv-a

2. Match the following Gulfs Location i) Gulf of Ob a) East of China ii) Gulf of Chihli b) East Vietnam iii) Gulf of Tonkin c) Between Yamal and Gyda peninsula iv) Gulf of Thailand d) South of Thailand

a) i- c, ii- a, iii-b, iv-d b) i-d, ii-b, iii-c, iv-a c) i-a, ii-b, iii-c, iv-d

3. Match Gulf Location i) Persian Gulf a) Separates Arabian peninsula from plateau of Iran ii) Gulf of Oman b) between Iranian plateau and Oman iii) Gulf of Aden c) between Somalia and Yemen iv) Gulf of Aquaba d) between Jordan and Egypt

a) i- d, ii- b, iii-c, iv-a b) i-c, ii- d, iii-b, iv- a c) i-a, ii-b, iii-c, iv-d

4. Consider the following, which of these are correct? i) Kurile Islands is an extension of the volcanic Kamchatka Peninsula ii) Ryukyu Island is an example of submerged island

Page 42: Study Kit Brochure for SSC Combined Graduate Level ...static.upscportal.com/files/study-kit/ssc-cgl/SSC-CGL-2013... · Brochure for SSC Combined Graduate Level Examination - 2014

Click Here for SSC CGL Online Coaching: http://sscportal.in/community/courses/ssc-cgl-tier-1

[Click Here to Buy This Study Kit in Hard Copy: http://sscportal.in/community/study-kit/cgl ] 41

iii) Spartly and Parcel Islands are a group of atolls in the South China Sea a) i and ii b) ii and iii c) i, ii and iii d) None

5. Consider the following statements, which of the following are incorrect? i) Luzon is known as the Rice Bowl of Philippines ii) Mount Mayo the only active volcano of Philippines is located in Luzon iii) Igorot tribe lives in the northern region of Luzon

a) i only b) iii only c) i and ii d) none of the above

6. Consider the following statements, which of the following are correct? i) Mindano island is the second largest island located in the northern part of Philippines ii) Mount Apo a dormant volcano on Mindano is the country’s highest peak.

a) ii only b) i only c) Both i and ii d) None

7. Match the following Strait Connects i) Bering Strait a) Sea of Okhotsk with Sea of Japan ii) Tata Strait b) East China sea with South China Sea iii) Farmosa Strait c) Celebs Sea with Java Sea iv) Makassar Strait d) East Siberian sea with Bering Sea

a) i-b, ii-c, iii-d, iv-a b) i-d, ii-a, iii-b, iv-c c) i-d, ii-c, iii-a, iv-b

8. Consider the following, which are correct? i) The Malacca strait separates the Malaya Peninsula and Sumatra ii) The Makassar separates the Borneo and Celebes Islands. iii) The Bering strait separates Asia and North America

a) i and iii b) ii and iii c) All of the above d) iii only

9. Consider the following, which of the following are false? a) The Himalayan mountain range lies to the North of Indo Gangetic plain. b) It is an example of block mountains c) Mount Everest is the highest peak, located in Nepal

a) i b) ii and iii c) iii only d) none of the above

10. Consider the following, which of them are true? a) The Karakoram Range lies in the North of Himalayas b) Godwin Austen (K2) is the highest peak of Karakoram c) The Tien Shan extends to the north east and reaches the Amur river under the name of Great

Khingan Mountains a) ii and iii b) i, ii and iii

c) None of the above d) ii and iii

Page 43: Study Kit Brochure for SSC Combined Graduate Level ...static.upscportal.com/files/study-kit/ssc-cgl/SSC-CGL-2013... · Brochure for SSC Combined Graduate Level Examination - 2014

Click Here for SSC CGL Online Coaching: http://sscportal.in/community/courses/ssc-cgl-tier-1

[Click Here to Buy This Study Kit in Hard Copy: http://sscportal.in/community/study-kit/cgl ] 42

ANSWERS: 1. (b) 2. (a) 3. (c) 4. (c) 5. (d) 6. (a) 7. (b) 8. (c) 9. (d) 10. (b)

India Physical INTRODUCTON

IMPORTANT MOUNTAIN RANGES

1. Great Himalaya or Himadari • Northern most part of the Himalayan Range is the world’s highest part with an average altitude of 6100 m

above sea level. • Include the world’s highest peak, Mt. Everest (8,848 m) and other high peaks-Makalu (8,481 m), Mansalu

(8,156 m), Annapurna (8,,078 m) and also the Indian peak Manghenjunga (8,598 m) and Nanga Parbat (8,126 m).

• Includes some famous passes—Burzil and Zozi La in Kashmir, Shipki La and Bara Lapcha La in Himachal Pradesh, Thaga La, Niti Pass, and Lipu Lekh in U.P., Jelep La and Nathu La in Sikkim.

ZASKAR RANGE • Situated on the western part of the Greater Himlaya and to the south of Trans Himalaya. Nanaga Parbat (8,126 m) • Forms the north-west part of Zaskar Range but geographically confined to Kashmir-Himachal Pradesh-Garhwal

region.

KARKAKORAM RANGES • Extend from the Pamir, east of the Gilgit Rier, 600

km long and the average width - 120-140 km. • Ancient name was Krishnagiri. • Grans Himalaya, originally a part of Eurasian plate. • Abode of largest glaciers in India. • Siachen, Baltoro, Biafo, and Hisper glaciers. • Highest Peak (in India): K2 or Godwin Austen (8611

m). • Other Important Peaks: Gasherbrum I or Hidden

Peak, Broad Peak and Gasherbrum II. • In the northern limit of Karakoram Range lies the Pamir, the Aghil Mountains, and Yarkand River and in the southern limit lies the River Indus and its tributary Shyok.

LADDAKH RANGE • Situated to the north of the Indus

Tsangpo Suture Zone (ITSZ) and south of Karakoram, between River Indus and Shyok.

• Highest Peak: Mt. Rakaposhi (steepest peak in the world).

Page 44: Study Kit Brochure for SSC Combined Graduate Level ...static.upscportal.com/files/study-kit/ssc-cgl/SSC-CGL-2013... · Brochure for SSC Combined Graduate Level Examination - 2014

Click Here for SSC CGL Online Coaching: http://sscportal.in/community/courses/ssc-cgl-tier-1

[Click Here to Buy This Study Kit in Hard Copy: http://sscportal.in/community/study-kit/cgl ] 43

• Second highest peak of the Himalayan Range in India. Dhaulagiri (8,172 m)

•Eastern continuation of Nanga Parbat and is located in Nepal.

2. Lesser Himalaya or Himachal • Lying to the north of Siwalik Range they are separated from the Siwalik Range by Duns—Dehradun,

Kothridun and Patlidun.

(I) PIR PANJAL RANGE

• Located in Kashmir—Punjab and extends from the Jhelum River to the upper Beas River for over 300 km.

• Separated from the Zaskar Range by the valley of Kashmir.

(II) DHAULADHAR

• Southern most range of the Lessers Himalaya.

• Rarely attains elevations higher than 4,000 m.

•Continue eastward into Mahabharat Range.

3. Siwalik Range

• Estends from Jammu & Kashmir (150 km wide) to Arunchal Pradesh (8-15 km) over 2400 km.

• Also known as Sub-Himalaya or Outer Himalaya.

• Northern limit—Main Boundary Thrust which separates Outer Himalaya from the Lesser Himalaya. Its southern limit is Indo Gangetic Plain.

•Youngest part of mountain chain, stretching from the Brahmputra to the Indus.

Important Gulfs

NAME SEPARATES LOCATION INFORMATION

Gulf of Kachchh Kachchh and Kathiawar West of Gujarat Region with highest potential of tidal Peninsula energy generation.

Gulf of Cambay Kathiawar Peninsula Gujarat Tapi, Narmada, Mahi and Sabarmati and Gujarat river drain into the Gulf.

Gulf of Mannar Sri Lanka and Southern South east of Asia’s first marine biosphere India reserve.

Important Lagoons and Lakes

NAME STATE INFORMATION

Kayals Kerala Popularly called backwaters in Kerala, on the Malabar coast.

A chain of lakes which are connected with each other by canals.

(Peaty soils of backwaters are called Kari in Kerala).

Vembnad Lake Kerala Large sized backwater of Kerala, have fertile (Largest Kayal alluvial islands. It is 75 km long and 5-10 km wide

enclosed by a spit.

Chilka Lake Orissa Situated to the south west of the Mahanadi Delta.

Page 45: Study Kit Brochure for SSC Combined Graduate Level ...static.upscportal.com/files/study-kit/ssc-cgl/SSC-CGL-2013... · Brochure for SSC Combined Graduate Level Examination - 2014

Click Here for SSC CGL Online Coaching: http://sscportal.in/community/courses/ssc-cgl-tier-1

[Click Here to Buy This Study Kit in Hard Copy: http://sscportal.in/community/study-kit/cgl ] 44

Maximum length - 64 km Enclosed by the sand pit, has an opening which permits sea-connection.

Maximum breath - 20 km Largest brackish water lake in Asia.

Average width - 150 km

Fresh Water Lakes

Wullar Lake Jammu and Largest fresh water lake of India. Kashmir

Kolleru Lake Andhra Pradesh A part of the southern border of Andhra Pradesh.

60 km long and 60 km Lagoon formed due to enclosure by sand spit now in the widest part. called Sriharikota islands.

Jaisamand Lake Rajasthan Largest fresh water lake of Rajasthan.

Nakki Lake Rajasthan A small natural lake near Mt. Abu surrounded by hills, important as tourist place.

Loktak Lake Manipur Site of hydroelectricity power generation.

An example of centripetal drainage.

Saline Water Lakes

Sambhar Lake Rajasthan Largest Lake of Rajasthan lies on the border of Jaipur and Nagaur District.

Sodium chloride (common salt) and sodium sulphate are produced fainly by the Hindustan Salt Ltd.

Deedwana Lake Rajasthan Situated near Deedwana Town of Nagaur District.

EXERCISE: 1. Consider the following, which of these are correct?

a) Siachen, Baltoro, Biafo and Hisper are glaciers in the Karakoram Range

b) The highest peak of Ladhak range is Mt Rakaposhi which is the steepest peak in the world

a) i and ii b) ii and iii

c) i and iii d) all of the above

2. Consider the following, which of these are correct?

i) Nanga Parbat is the second highest peak of Himalayan Range in India

ii) Eastern continuation of the Nanga Parbat is located in Nepal

a) i and ii b) i

c) ii d) none

3. Match the following

Name Location

i) Punjab Himalaya a) Between Kali and Tista

ii) Kumaon Himalaya b) Between Satluj and Kali

iii) Nepal Himalaya c) Between Tista and Dihang

iv) Assam Himalaya d) Between Indus and Satluj

a) i- d, ii- b, iii-a, iv-c

Page 46: Study Kit Brochure for SSC Combined Graduate Level ...static.upscportal.com/files/study-kit/ssc-cgl/SSC-CGL-2013... · Brochure for SSC Combined Graduate Level Examination - 2014

Click Here for SSC CGL Online Coaching: http://sscportal.in/community/courses/ssc-cgl-tier-1

[Click Here to Buy This Study Kit in Hard Copy: http://sscportal.in/community/study-kit/cgl ] 45

b) i-a, ii-b, iii-c, iv-d

c) i-b, ii-c, iii-d, iv-a

4. Consider the following, which of these is incorrect?

i) Mishmi Hills is the highest range of Purvanchal Hills

ii) Mizo Hills were previously known as Lushai Hills

iii) Lohtak Lake is situated in the Manipur hills

a) i and ii b) ii and iii

c) none of the above d) i and iii

5. Yamuna, Ghagra, Gomti, Kosi and Son are tributaries of

a) Ganga b) Brahmaputra

c) Indus d) Yamuna

6. Consider the following are incorrect?

i) Vindhyan range is the example of Relict mountain

ii) The Aravalli range is an example of block mountain

iii) Guru Shikhar is the highest peak on the Abu hills

a) i and ii b) ii and iii

c) i and iii d) None

7. Match the following

Hill Highest Peak

i) Nilgiri Hill a) Amarkantak

ii) Maikal range b) Doda Beta

iii) Anamalai Hills c) Kalsubai

iv) Sahyadri d) Ana Mudi

a) i- a, ii-b, iii-d, iv-c

b) i- c, ii-d, iii-a, iv-b

c) i-b, ii-a, iii-d, iv-c

8. Consider the following, which of these are true

i) Chota Nagpur is described as the Ruhr of India

ii) Pat lands are a chief characteristic of Chota Nagpur plateau

iii) It is poor in mineral resources

a) i and ii b) ii and iii

c) i and iii d) iii

9. Match the following

i) Gulf of Kachchh a) Asia’s first Marine biosphere reserve

(ii) Gulf of Cambay b) Region with highest potential of tidal Energy

(iii) Gulf of Mannar c) Tapi, narmada, Mahi and Sabarmati drain into this gulf

a) i- b ,ii- a, iii-c

b) i-a, ii-b, iii-c

c) i-b, ii-c, iii-a

10. Match the following

Lake Location

i) Nakki a) Orissa

ii) Vembnad b) Rajasthan

Page 47: Study Kit Brochure for SSC Combined Graduate Level ...static.upscportal.com/files/study-kit/ssc-cgl/SSC-CGL-2013... · Brochure for SSC Combined Graduate Level Examination - 2014

Click Here for SSC CGL Online Coaching: http://sscportal.in/community/courses/ssc-cgl-tier-1

[Click Here to Buy This Study Kit in Hard Copy: http://sscportal.in/community/study-kit/cgl ] 46

iii) Chilka c) Kerala

a) i-b, iii-a, iv-c

b) i- a, ii-b, iii-c

c) i-b, ii-c, iii-a

ANSWERS: 1. (d) 2. (a) 3. (b) 4. (c) 5. (a) 6. (a) 7. (c) 8. (a) 9. (a) 10. (c)

Indian Economy

Planned economy is one in which the state owns(partly or wholly) and directs the economy. While such a role is assumed by the State in almost every economy, in planned economies, it is pronounced: for example in communist and socialist countries- former USSR and China till the 1970's. In such a case a planned economy is referred to as command economy or centrally planned economy or command and control economy. In command economies, state does the following

• Control all major sectors of the economy

• Legislate on their use and about the distribution of income

• State decides on what should be produced and how much; sold at what price

• Private property is not allowed

In a market economy, it is the opposite- state has a minimal role in the management of the economy- production, consumption and distribution decisions are predominantly left to the market. State plays certain role in redistribution. State is called the laissez faire state here. It is a French phrase literally meaning "Let do."

Indicative plan (see ahead) is one where there is a mixed economy with State and market playing significant roles to achieve targets for growth that they together set. It is operated under a planned economy but not command economy.

The difference between planned economy and command economy is that in the former there may be mixed economy and while in the latter Government owns and regulates economy to near monopolistic limit.

Command economies were set up in China and USSR, mainly for rapid economic growth and social and economic justice but have been dismantled in the last two decades as they do not create wealth sustainably and are not conducive for innovation and efficiency. Cuba and North Korea are still command economies.

History of Economic Planning in India: The beginnings

India being devastated economically after more than 2 centuries of colonial exploitation resulting in chronic poverty, eradication of poverty was the driving force for the formulation of various models of growth before Independence.

In 1944 leading businessmen and industrialists (including Sir Purshotamdas Thakurdas, JRD Tata, GD Birla and others) put forward "A Plan of Economic Development for India" -popularly known as the 'Bombay Plan". It saw India's future progress based on further expansion of the textile and consumer industries already flourishing in cities like Bombay and Ahmedabad. It saw an important role the State in post-Independent India: to provide infrastructure, invest in basic industries like steel, and protect Indian industry from foreign competition.

Visionary engineer Sir Mokshagundam Visvesvarayya. pointed to the success of Japan and insisted that 'industries and trade do not grow of themselves, but have to be willed, planned and systematically developed' - in his book titled "Planned Economy for India"(1934) Expert economists and businessmen were to do the planning. The goal was poverty eradication through growth.

Page 48: Study Kit Brochure for SSC Combined Graduate Level ...static.upscportal.com/files/study-kit/ssc-cgl/SSC-CGL-2013... · Brochure for SSC Combined Graduate Level Examination - 2014

Click Here for SSC CGL Online Coaching: http://sscportal.in/community/courses/ssc-cgl-tier-1

[Click Here to Buy This Study Kit in Hard Copy: http://sscportal.in/community/study-kit/cgl ] 47

The Indian National Congress established a National Planning Committee under the chairmanship of Jawaharlal Nehru. It (1938) stated the objective of planning for development "was to ensure an adequate standard of living for the masses, in other words, to get rid of the appalling poverty of the people". It advocated heavy industries that were essential both to build other industries, and for Indian self-defence; heavy industries had to be in public ownership, for both redistributive and security purposes; redistribution of land away from the big landlords would eliminate rural poverty.

During the 1940's, the Indian Federation of Labour published its People's Plan by MN Roy that stressed on employment and wage goods. SN Agarwala, follower of Mahatma Gandhi published Gandhian Plan that emphasized on decentralization; agricultural development; employment; cottage industries etc.

Planning Goals

After Independence in 1947, India launched the year plan for rapid growth. Planning has the following long term goals.

• Growth

• Modernization

• Self-reliance and

• Social justice

Economic growth is the value of the goods and services produced by ban economy. It is conventionally measured as the percent rate of increase in real gross domestic product, or real GDP- real means adjusted to inflation. Growth measures quantitative increase in goods and services.

Economic development refers to growth that includes redistributive aspects and social justice. GPD shows growth and welfare and human development aspects like education, access to basic amenities, environmental quality, freedom, or social justice. Economic growth is necessary for development but not sufficient.

Growth is expected to [spread to all sections and regions; raise resources for the Government to spend on socio-economic priorities etc. It takes a long time for growth to trickle down to all people and regions. Therefore, State plans for an expeditious process of inclusive growth.

Modernization is improvement in technology. It is driven by innovation and investment in R and D. Education is the foundation of modernization. The more modernized the economy, the greater the value created by it.

Self-reliance means relying on the resources of the country and not depending on other countries and the MNCs for investment and growth. India embarked on the goal partly due to the colonial experience and partly due to the goal of orienting growth to development and poverty eradication. Nehru-Mahalanobis model of growth that closed Indian economy and relied on basic industries is the main plank for self-reliance.

The term self-reliance should not be confused with self-sufficiency - the former means depending on resources of the country and avoid dependence on external flows; the latter means that the country has all the resources it needs. No country can be self-sufficient. Social justice means inclusive and equitable growth where inequalities are not steep and benefits of growth reach all- rural- urban , man-woman; caste divide and inter-regional divides are reduced.

While the above four are the long term goals of the planning process, each five year plan has specific objectives and priorities.

History of Planning

First Plan (1951-56)

The First Plan stressed more on agriculture, in view of large scale import of foodgrains and inflationary pressures on the economy. Other areas of emphasis were power and transport. The annual average growth rate during the First Plan was estimated as 3.61% as against a target of 2.1%. Renowned economist KN Raj, who died in 2010 was one of the main architects of India's first five-year plan.

Page 49: Study Kit Brochure for SSC Combined Graduate Level ...static.upscportal.com/files/study-kit/ssc-cgl/SSC-CGL-2013... · Brochure for SSC Combined Graduate Level Examination - 2014

Click Here for SSC CGL Online Coaching: http://sscportal.in/community/courses/ssc-cgl-tier-1

[Click Here to Buy This Study Kit in Hard Copy: http://sscportal.in/community/study-kit/cgl ] 48

Second PIan (1956-61)

With agricultural targets of previous plan achieved, major stress was on the establishment of heavy industries. Rate of investment was targeted to increase from 7% to 11%. The Plan achieved a more than targeted growth rate of 4.32%. This Plan envisaged to give a big push to the economy so that it enters the take off stage. It was based on Nehru-Mahalanobis model- self-reliance and basic-industry driven growth.

Third Plan (1961-66)

It tried to balance industry and agriculture. The aim of Third Plan was to establish a self sustaining economy. For the first time, India resorted to borrowing from IMF. Rupee was also devalued for the first time in 1996. India's conflict with Pakistan and repeated droughts also contributed in the failure of this Plan.

Annual Plan

As the Third Plan difficulties on the external front (war with China in 1962 and Pakistan in 1965); and the economic troubles mounted on the domestic front- inflation, floods, forex crisis- the Fourth Plan could not be started from 1966. There were three annual plans till 1969. This period is called plan holiday- that is when five year plans are not implemented. The Annual Plans were: 1966-67, 1967-68 and 1968-69.

Forth Plan (1969-74)

The main objective of this Plan was growth with stability. The Plan laid special emphasis on improving the condition of the under-privileged and weaker sections through provision of education and employment. Reducing the fluctuations in agricultural production was also a point of emphasis of this Plan. The Plan aimed at a target growth of 5.7% and the achievement against this was 3.21 %.

Fifth Plan (1974-79)

The main objective of the Plan was Growth for Social Justice. The targeted growth rate was 4.4% and we achieved 4.8%. It was cut short by the Janata Party that came to power in 1977.

Sixth Plan (1980-1985)

Removal of poverty was the foremost objective of Sixth Plan. Another area of emphasis was infrastructure, which was to be strengthened for development of both industry and agriculture. The achieved growth rate of 5.7% was more than the targeted one.

Direct attack on poverty was the main stress of the Plan.

Seventh Plan (1985-90)

This Plan stressed on rapid growth in food-grains production and increase in employment opportunities. The growth rate of 5.81% achieved in this Plan was more than targeted one. The plan was more than the targeted. The plan saw the beginnings of liberalization of Indian economy. The 8th Plan could not start in 1990 due to economic crisis and political instability. There were two annual plans- plan holiday.

Eighth Plan (1992-1997)

This Plan was formulated keeping in view the process of economic reforms and restructuring of the economy. The main emphasis of this Plan were

• to stabilize the adverse balance of payment scenario sustainably.

• improvement in trade and current account deficit.

• human development as main focus of planning.

It was indicative plan for the first time. The Plan was formulated in a way so as to manage the transition from a centrally planned economy to market led economy. The targeted annual average rate of growth of the economy during Eighth Plan was 5.6%. Against this, we achieved an average annual growth of 6.5%.

Page 50: Study Kit Brochure for SSC Combined Graduate Level ...static.upscportal.com/files/study-kit/ssc-cgl/SSC-CGL-2013... · Brochure for SSC Combined Graduate Level Examination - 2014

Click Here for SSC CGL Online Coaching: http://sscportal.in/community/courses/ssc-cgl-tier-1

[Click Here to Buy This Study Kit in Hard Copy: http://sscportal.in/community/study-kit/cgl ] 49

The Plan was based on Rao-Manmohan Singh model of liberalization.

Ninth Five Year Plan (1997-2002)

The salient features of the Ninth Five Year Plan are a target annual average growth rate of 6.5 per cent for the economy as a whole, and a growth rate of 3.9 per cent for agriculture sector, among others. The key strategies envisaged to realise this target rest on attaining a high investment rate of 28.2 per cent of GDP at market prices. The domestic saving rate, which determines the sustainable level of investment, is targeted at 26.1 per cent of the GDP. Care has been taken to ensure achievement of a sustainable growth path in terms of external indebtedness as well as fiscal stability, Rate of growth achieved was 5.4%

Tenth Plan (See ahead)

Growth Performance in the Five Year Plans (per cent per annum)

Target Actual

1. First Plan (1951-56) 2.1 3.61

2. Second Plan (1956-61) 4.5 4.32

3. Third Plan (1961-66) 5.6 3.21

4. Fourth Plan (1969-74) 5.7 4.80

5. Fifth Plan (1974-79) 4.4 5.69

6. Sixth Plan (1980-85) 5.2 5.81

7. Seventh Plan (1985-90) 5.0 6.7

8. Eighth Plan (1992-97) 5.6 5.35

9. Ninth Plan (1997-2002) 6.5 7.8%

10. Tenth Plan(2002-2007) 8%

11. Eleventh Plan( 2007-12) 8.1 (revised 2010)

The economy is expected to expand by 9% per cent in 2010-11- having achieved 8.9% real growth in

the first half of 2010-2011. It may rise to 10 per cent in the terminal year of the 11th Plan. Government set an average annual growth target of 9 per cent for the 11th Plan - beginning with 8.5 per cent in the first year and closing with 10 per cent In 2011-12. The MTA document said the economy exceeded expectations in 2007-08, with a growth rate of 9 per cent, but the momentum was interrupted in 2008-09 because of the global financial crisis. Following the global meltdown, the growth rate slipped to 6.7 per cent in 2008-09 from over 9 per cent in the preceding three years. In the year 2009-10. the growth rate was 7.6%.

Planning Commission

The Planning Commission was constituted in March, 1950 by a Resolution of the Government of India, and works under the overall guidance of the National Development Council. The Planning Commission consults the Central Ministries and the State Governments while formulating Five Year Plans and Annual Plans and also oversees their implementation. The Commission also functions as an advisory body at the apex level.

The 1950 resolution setting up the Planning Commission outlined its functions as to:

• Make an assessment of the material, capital and human resources of the country, including technical personnel, and investigate the possibilities of augmenting such of these resources as are found to be deficient in relation to the nation's requirement;

• Formulate a Plan for the most effective and balanced utilisation of country's resources;

• On a determination of priorities, define the stages in which the Plan should be carried out and propose the allocation of resources for the due completion of each stage;

Page 51: Study Kit Brochure for SSC Combined Graduate Level ...static.upscportal.com/files/study-kit/ssc-cgl/SSC-CGL-2013... · Brochure for SSC Combined Graduate Level Examination - 2014

Click Here for SSC CGL Online Coaching: http://sscportal.in/community/courses/ssc-cgl-tier-1

[Click Here to Buy This Study Kit in Hard Copy: http://sscportal.in/community/study-kit/cgl ] 50

• Indicate the factors which are tending to retard economic development, and determine the conditions which, in view of the current social and political situation, should be established for the successful execution of the Plan;

• Determine the nature of the machinery which will be necessary for securing the successful implementation of each stage of the Plan in all its aspects;

• Appraise from time to time the progress achieved in the execution of each stage of the Plan and recommend the adjustments of policy and measures that such appraisal may show to be necessary; and

• Make such interim or ancillary recommendations as appear to it to be appropriate either for facilitating the discharge of the duties assigned to it, or on a consideration of prevailing economic conditions, current policies, measures and development programmes or on an examination of such specific problems as may be referred to it for advice by Central or State Governments.

The Prime Minister is the ex officio Chairman of the Planning Commission. Deputy Chairperson enjoys the rank of a cabinet minister. A member of the Planning Commission enjoys the rank of a Minister of State in the Union Government. Cabinet Ministers with certain important portfolios act as part-time members.

The Deputy Chairman and the full time Members of the Planning Commission function as a composite body in the matter of detailed plan formulation. They provide advice and guidance to the subject Divisions of the Commission in the various exercises undertaken for the formulation of Approach to the Five Year Plans, and Annual Plans. Their expert guidance is also available to the subject Divisions for monitoring and evaluating the Plan programmes, projects and schemes.

The Planning Commission functions through several technical/subject Divisions. Each Division is headed by a Senior Officer designated as Pr. Adviser/Adviser/Addl. Adviser/Jt. Secretary/Jt. Adviser.

The various Divisions in the Commission fall under two broad categories:

• General Divisions which are concerned with aspects of the entire economy; and

• Subject Divisions which are concerned with specified fields of development.

The General Divisions functioning in the Planning Commission are:

• Development Policy Division,

• Financial Resources Division, .

• International Economics Division,

• Labour, Employment and Manpower Division;

• Perspective Planning Division,

• Plan Coordination Division,

• Project Appraisal and Management Division,

• Socio-Economic Research Unit,

• State Plan Division, including Multi Level Planning, Border Area Development Programme, Hill Area Development and North Eastern Region (NER), and

• Statistics and Surveys Division,

• Monitoring Cell.

The Subject Divisions are:

• Agriculture Division,

• Backward Classes Division,

• Communication & Information Division,

Page 52: Study Kit Brochure for SSC Combined Graduate Level ...static.upscportal.com/files/study-kit/ssc-cgl/SSC-CGL-2013... · Brochure for SSC Combined Graduate Level Examination - 2014

Click Here for SSC CGL Online Coaching: http://sscportal.in/community/courses/ssc-cgl-tier-1

[Click Here to Buy This Study Kit in Hard Copy: http://sscportal.in/community/study-kit/cgl ] 51

• Education Division,

• Environment and Forests Division,

• Health & Family Welfare Division,

• Housing, Urban Development & Water Supply Division,

• Industry & Minerals Division,

• Irrigation & Command Area Development Division,

• Power & Energy Division (including Rural Energy, Non-Conventional Energy Sources and Energy Policy Cell)

• Rural Development Division,

• Science & Technology Division,

• Social Welfare & Nutrition Division,

• Transport Division,

• Village & Small Industries Division, and

• Western Ghats Secretariat.

The Programme Evaluation Organisation undertakes evaluation studies to assess the impact of selected Plan Programmes / Schemes in order to provide useful feedback to planners and implementing agencies.

The Commission is a corner-stone of our federal structure, a think-tank ; helps to balance the priorities and expenditures of the Ministries of the Union Government ; throws up ideas on policies for structural and perspective changes ; and is a reservoir of research."

Relevance of Planning

There has been a national debate about the relevance of planning in the era of liberalization where the state controls and regulations are dismantled to a great extent and market forces are given larger role. The investment of the government for the five year plans is also on decline. The trend began in the 7th plan and strengthens into the Eleventh Plan.

It is true that the quantitative aspects of planning in terms of control over economy are being selectively phased out and the nature of planning process is undergoing a qualitative change. Planning is important for the following reasons in the era of liberalization

• In a federal democracy like ours, the principal task of planning is to evolve a shared vision among not only the federal units but also among other economic agents so that the efforts of all the actors become convergent towards the national priorities, the role of planning is to develop a common policy stance for center and states. Also, the task of federal policy coordination is central to Indian Planning. For example, the need to invite foreign investment in infrastructure areas like power need center - state coordination as the necessary legislation and administrative changes involve both.

• While the growth process can be made the responsibility of the corporate sector to a greater degree, its direction and distribution are to be steered by planned public intervention so that regional imbalances are reduced and socio economic inequities are set right. For example, `directing the growth of the large industry into the backward areas and technology intensive areas to realize national goals.

• The nature of instruments available to planners in the implementation has changed. Quantitative controls have yielded place to qualitative ones .The planning process has to focus on the need for planning for policy.

• Planning at the grass roots level that is participatory is very crucial for improving the delivery systems and proper use of the resources. The role of the government is thus to facilitate participatory planning.

Page 53: Study Kit Brochure for SSC Combined Graduate Level ...static.upscportal.com/files/study-kit/ssc-cgl/SSC-CGL-2013... · Brochure for SSC Combined Graduate Level Examination - 2014

Click Here for SSC CGL Online Coaching: http://sscportal.in/community/courses/ssc-cgl-tier-1

[Click Here to Buy This Study Kit in Hard Copy: http://sscportal.in/community/study-kit/cgl ] 52

• Environmental priorities are a major concern of planning

• planning is necessary for the sectors like energy, communication, transport and so on as private sector needs to be guided into the national plan.

• In the era of globalization where corporates are not expected to plan beyond the growth of a particular unit, the role of safeguarding national interest is that of planning by the State. For example, being subjected to various discriminative trade practices by EU, USA and so on, the Indian farmers, manufacturers and exporters have to fight sophisticated battles in the WTO for which the legal services and information and building up bargaining power are best provided by the State.

Thus, planning continues to be relevant and ever more so for the following reasons

• Federal cooperation and coordination

• Equitable growth

• Environment friendly development

• Defending national interest in the age of globalization

• Inter-sectoral balance in growth

Changing Role of Planning Commission

From a highly centralized planning system, the Indian economy is gradually moving onwards indicative planning where hard planning is no longer undertaken. The role of the Planning Commission accordingly changes. The Commission concerns itself with the building of a long term strategic vision of the future and decide on priorities of nation. It works out sectoral targets and provides promotional stimulus to the economy to grow in the desired direction.

Planning Commission plays an integrative role in evolving a national plan in critical areas of human and economic development. In the social sector, Planning Commission helps in schemes which require coordination and synergy like rural health, drinking water, rural energy needs, literacy and environment protection.

When planning in a vast federal country like India involves multiplicity of agencies, a high powered body like the PC can help in evolution of an integrated approach for better results at much lower costs.

In our transitional economy, Planning Commission attempts to play a systems change role and provide consultancy within the Government for developing better systems. It has to ensure smooth management of the change and help in creating a culture of high productivity and efficiency in the Government

In order to spread the gains of experience more widely, Planning Commission also plays an information dissemination role.

With the emergence of severe constraints on available budgetary resources, the resource allocation system between the States and Ministries of the Central Government is under strain. This requires the Planning Commission to play a mediatory and facilitating role, keeping in view the best interest of all concerned.

From Planning Commission to Systems Reforms Commission

There has been a significant change in the role of the PC since its inception in 1950. In the beginning, Planning Commission was all powerful and had the final say and the veto over every aspect - related to growth and socio-economic development- of the functioning of the Union Ministries and the State Governments: The manner of raising and utilising resources; specific allocations to particular schemes and programmes; location of enterprises; expansion and reduction of capacities; application of technologies; sources of supplies; modalities of implementation; priorities, phasing, pricing, targets and time-frames; nature of the instrumentalities; qualifications and strength of personnel of organisations; staff emoluments etc.

Page 54: Study Kit Brochure for SSC Combined Graduate Level ...static.upscportal.com/files/study-kit/ssc-cgl/SSC-CGL-2013... · Brochure for SSC Combined Graduate Level Examination - 2014

Click Here for SSC CGL Online Coaching: http://sscportal.in/community/courses/ssc-cgl-tier-1

[Click Here to Buy This Study Kit in Hard Copy: http://sscportal.in/community/study-kit/cgl ] 53

Since 1991, India adopted the indicative planning model, away from the kind of centralised planning on the Soviet model envisaged by Jawaharlal Nehru. Now Ministries and Departments, as well as the corporate entities in the private sector, enjoy a lot of functional, financial and operational autonomy.

In the era of liberalisation, the economic players should properly be left to decide for themselves what they consider to be the appropriate courses of action on the various issues coming up before them, whether they relate to policies, schemes or investments.

The government intends to convert the Planning Commission into a think-tank to generate original ideas in the very broad domain of economic policy for the government to then act on. It will also be the government agency responsible for acting as an interface with other independent think-tanks and NGOs. The PM would like the commission to engage more directly with the "polity", presumably with various ministries in the Central and state governments, and be able to persuade them to implement certain ideas or "plans" generated by the government's own think tank. That isn't radically different from its existing role — the Planning Commission has few direct powers of execution in any case and must rely on the power of persuasion to sell its ideas to the Centre and states.

Interestingly enough, the new role sought for the Planning Commission seems to be very similar to the role played by the National Advisory Council, which also generates ideas within, coordinates with NGOs and civil society and then tries to "persuade" the government to act. NAC's focus so far has been social sectors whereas a systems reforms commission can take on a broader gambit of issues, including public finances, infrastructure and so on.

The government's move to revamp and gradually transform the Planning Commission into a System Reforms Commission is & major step that can make the institution more relevant to a market economy. The idea is to metamorphose the plan panel from a reactive agency into a strategic thinking group, which maps out risks and opportunities by focusing on issues

The shrinking role of the government in mobilising and controlling investments has pushed the Planning Commission to focus more on issues related to enforcing fiscal discipline in the central and state governments, including in the various ministries, departments and public sector enterprises.

According to Arun Maira, PC member, the Planning Commission will gradually transform itself into a Systems Reforms Commission for resolving the systemic problems of the 21st Century over the next two-three years as desired by Prime Minister Marimohan Singh. It will restructure itself to serve three essential functions: build a larger network around its members with think tanks and opinion makers, produce thought papers at a faster pace and communicate more lucidly with polity.

After Montek Singh Ahluwalia, Planning Commission Deputy Chairman, first admitted last December that the Plan panel cannot work in a silo, Arun Maira, formerly with Boston Consulting Group, India, and appointed Plan panel member in the second UP A term, was assigned the task of revamping the Planning Commission.

National Development Council

The National Development Council is not a Constitutional body nor a statutory body (not set up by an Act of the Parliament). Union Cabinet set up the NDC in 1952 with the following functions

• To prescribe guidelines for the formulation of the national plan.

• To consider the national plans formulated by the Planning Commission.

• To assess the resources for the plan and recommend a strategy for mobilizing the resources.

• To consider important questions of socio-economic policy affecting development of the nation.

• To review the progress of the five year plan mid-course and suggest measures for achieving the original targets.

NDC is headed by the Prime Minister of India and comprising of all Union Cabinet Ministers, Chief Ministers of all the States and Administrators of Union Territories and Members of the Planning

Page 55: Study Kit Brochure for SSC Combined Graduate Level ...static.upscportal.com/files/study-kit/ssc-cgl/SSC-CGL-2013... · Brochure for SSC Combined Graduate Level Examination - 2014

Click Here for SSC CGL Online Coaching: http://sscportal.in/community/courses/ssc-cgl-tier-1

[Click Here to Buy This Study Kit in Hard Copy: http://sscportal.in/community/study-kit/cgl ] 54

Commission. Ministers of State with independent charge are also invited to the deliberations of the Council.

The National Development Council (NDC) has a special role in our federal polity. It is the apex body for decision making and deliberations on development matters. It has the explicit mandate to study and approve the Approach Plan to the Five year Plans and the Five Year Plan documents. The mid-term reviews of the Five year Plans are considered by the NDC. In fact,, without the NDC approving, the Five Year Plan does not come into effect.

55th meeting of NDC was held in July 2010 to approve the MTR of the 11th FYP.

The CMP of the UPA Government (2004) says that NDC will be activated. It will meet at least three times in a year and in different state capitals . It will be developed as an effective instrument of cooperative federalism.

Mixed economy

India is a mixed economy combining features of both capitalist market economies and socialist command economies. Thus, there is a regulated private sector (the regulations have decreased since liberalisation) and a public sector controlled almost entirely by the government. The public sector generally covers areas which are deemed too important; or not profitable enough for the private sector. Thus such services as railways and postal system are carried out by the government.

Since independence, various phases have seen nationalisation of such areas as banking, thus bringing them into the public sector, on one hand, and privatisation of some of the Public Sector Undertakings during the liberalisation period on the other

Financial resources for the Five year Plans

Theresources for the Plan come from

• Central budget

• State budgets PSEs

• Domestic private sector and

• FDI

A Note on Gross Budgetary Support

Resources of the Centre consist of both budgetary resources including external assistance routed through the budget and the Internal & Extra Budgetary Resources (IEBR) of Central Public Sector Enterprises (CPSEs). The quantum of budgetary resources of the Centre which is available for providing overall budgetary support to the plan is divided into two parts viz. budgetary support for Central Plan (including U.Ts without Legislature) and Central Assistance for States' Plans (including U.Ts with Legislature). A part of the budgetary resources allocated as budgetary support for the Central Plan is used for providing necessary support to CPSEs.

GBS is the amount from the central Budget that goes to fund the plan investments during the plan period.

Achievements of Planning

In the last about 60 years since India became a Republic, the National Income has increased many times. Today, India is the third largest economy in Asia with about $1.4 trillion GDP after China and Japan; is the 11th largest economy in the world. India is the fourth largest in the world as measured by purchasing power parity (PPP), with a gross domestic product (GDP) of about $4 trillion-USA, China, Japan, India.

In the face of global recession, India posted 6.7% rate of growth in 2008-09 and 7.6% in 2009-10 and is the second fastest growing major economy after China/The first half of 2010-11 saw the growth rate at 8.9%.

Page 56: Study Kit Brochure for SSC Combined Graduate Level ...static.upscportal.com/files/study-kit/ssc-cgl/SSC-CGL-2013... · Brochure for SSC Combined Graduate Level Examination - 2014

Click Here for SSC CGL Online Coaching: http://sscportal.in/community/courses/ssc-cgl-tier-1

[Click Here to Buy This Study Kit in Hard Copy: http://sscportal.in/community/study-kit/cgl ] 55

Poverty dropped to about 20% of the population- the criterion used is monthly consumption of goods valued less than Rs. 211.30 per capita for rural areas and Rs. 454.11 for urban areas (2006) Social indicators improved e though there is a long way to go- IMR, MMR, literacy, disease eradication etc. The industrial infrastructure is relatively strong - cement, steel .fertilizers, chemicals, etc Agricultural growth is also gaining momentum with food grains production at 233 mt in 2010.

Forex reserves are $300 b (January 2011) which is a dramatic turnaround from 1991

when we had a billion dollars.

More than 1.7 lakh MW of power capacity is installed by the end of 2010

India has emerged as a back office if the world and its prowess in software is growing.

India ranks fourteenth worldwide in factory output.

India ranks fifteenth worldwide in services output.

There has been considerable expansion of higher, education. At the time of Independence there were 20 universities and 591 colleges, while today, there are almost 500 universities and 21,000 colleges. Literacy levels are 75%(2010).

The failures of planning are equally clear

• Poverty still plagues about 360million (Tendulkar)

• Inflation on CPI and food inflation are rising relentlessly hurting the poor-. 14.4% is the food inflation I December 2010.

• Unemployment is high

• Regional imbalances are intensifying

• Malnutrition haunts about half the children in India.

Indicative planning Indicative planning was adopted since 8 five year plan (1992-97). It is characterized by an economy where the private sector is given a substantial role. State would turn its role into a facilitator from that of a controller and regulator.

It was decided that trade and industry would be increasingly freed from government control and that planning in India should become more and more indicative and supportive in nature. In other words, the remodeling of economic growth necessitated recasting the planning model from imperative and directive hard') to indicative (soft) planning. Since the Government did not contribute the majority of the financial resources, it had to indicate the policy direction to the corporate sector and encourage them to contribute to plan targets. Government should create the right policy climate- predictable, irreversible and transparent- to help the corporate sector contribute resources for the plan: fiscal, monetary, forex and other dimensions.

Indicative planning is to assist the private sector with information that is essential for its operations regarding priorities and plan targets. Here, the Government and the corporate sector are more or less equal partners and together are responsible for the accomplishment of planning goals. Government, unlike earlier, contributes

less than 50% of the financial resources. Government provides the right type of policies and creates the right type of milieu for the private sector-including the foreign sector to contribute to the results.

Indicative planning gives the Government an opportunity to give the private sector encouragement to achieve growth in areas where the country has inherent strengths. It is known to have brought Japan results in shifting towards microelectronics. In France, too indicative planning was in vogue.

Planning Commission would work on building a long-term strategic vision of the future. The concentration would be on anticipating future trends and evolving strategies for competitive international standards. Planning will largely be indicative and the public sector would be gradually withdrawn from areas where no public purpose is served by its presence. The new approach to

Page 57: Study Kit Brochure for SSC Combined Graduate Level ...static.upscportal.com/files/study-kit/ssc-cgl/SSC-CGL-2013... · Brochure for SSC Combined Graduate Level Examination - 2014

Click Here for SSC CGL Online Coaching: http://sscportal.in/community/courses/ssc-cgl-tier-1

[Click Here to Buy This Study Kit in Hard Copy: http://sscportal.in/community/study-kit/cgl ] 56

development will be based on "a re-examination and re-orientation of the role of the government". This point is particularly stressed in the development strategy of the Tenth Five Year Plan (2002-2007)

Indicative planning was not contemplated at the beginning of fifties as there was hardly any corporate sector in India and Government shouldered almost the entire responsibility of socio-economic planning.

Rolling Plan

It was adopted in India in 1962, in the aftermath of Chinese attack on India, in the Defence Ministry in India. Professor Gunnar Myrdal (author of famous book ‘Asian Drama') recommended it for developing countries in his book - Indian Economic Planning in Its Broader Setting.

In this type, every year three new plans are made and implemented- annual plan that includes annual budget; five year plan that is changed every year in response to the economic demands; and perspective plan for 10 or 15 years into which the other two plans are dovetailed annually. Rolling plan becomes necessary in circumstances that are fluid.

Financial Planning

Here, physical targets are set in line with the available financial resources. Mobilization and setting expenditure pattern of financial resources is the focus in this type of planning.

Physical planning

Here, the output targets are prioritized with inter-sect oral balance .Having set output targets, the finances are raised.

Nehru-Mahalanobis Model of Economic Growth

Indian economy at the time of independence was characterized by dependence on exports of primary commodities; negligible industrial base; unproductive agriculture etc.

Thus, the turning point in India's planning strategy came with the second five-year (1956-61) plan. The model adopted for the plan is known as the Nehru-Mahalanobis strategy of development as it articulated by Jawahar Lal Nehru's vision and P.C. Mahalanobis was its chief architect. The central idea underlying this strategy is well conveyed by recalling the following statement from the plan document. ‘If industrialization is to be rapid enough, the country must aim at developing basic industries and industries which make machines to make the machines needed for further development.'

The Mahalanobis model of growth is based on the predominance of the basic goods (capital goods or investment goods are goods that are used to make further goods; the goods that make up the industrial market like machines, tools factories, etc. It is based on the premise that it would attract all round investment and result in a higher rate of growth of output. That will develop small cale and ancillary industry to boost employment generation, poverty alleviation, exports etc. The emphasis was on expanding the productive ability of the system, through forging strong industrial linkages, as rapidly as possible.

Other elements of the model are

• Import substitution. Protective barriers against foreign competition to enable Indian companies to develop domestically produced alternatives for imported goods and to reduce India's reliance on foreign capital.

• A sizeable public sector active in vital areas of the economy including atomic energy and rail transport.

• A vibrant small-scale sector driving consumer goods production for dispersed and equitable growth and producing entrepreneurs.

In terms of the core objective of stepping up the rate of growth of industrial production, the strategy paid off. Rate of growth of overall industrial production picked up. The strategy laid the foundation for a well-diversified industrial structure within a reasonably short period and this was a major achievement. It • gave the base for self-reliance.

However, the strategy is criticized for the imbalances between the growth of the heavy industry sector and other spheres like agriculture and consumer goods etc that resulted. It is further criticized as it

Page 58: Study Kit Brochure for SSC Combined Graduate Level ...static.upscportal.com/files/study-kit/ssc-cgl/SSC-CGL-2013... · Brochure for SSC Combined Graduate Level Examination - 2014

Click Here for SSC CGL Online Coaching: http://sscportal.in/community/courses/ssc-cgl-tier-1

[Click Here to Buy This Study Kit in Hard Copy: http://sscportal.in/community/study-kit/cgl ] 57

relied on trickle down effect benefits of growth will flow to all sections in course of time. This approach to eradication of poverty is slow and incremental. It is believed that frontal attack on poverty is required.

The criticism is one sided as in the given context, the Mahalanobis model was correct for growth and self-reliance.

Rao-Man Mohan Singh Model of Growth

The launching of economic reforms by the government in 1991 is driven .by the Rao-Man Mohan model - Mr. Narasimha Rao, the PM in 1991 and Finance Minister Dr. Man Mohan Singh. Its essence is contained in the New Industrial • Policy 1991 and extends beyond it too. The model has the following contents

• Reorient the role of State in economic management. State should re focus on social and infrastructural development, primarily

• Dismantle, selectively controls and permits in order to permit private sector to invest liberally

• Open up the economy and create competition for PSEs- for better productivity and profitability

• External sector liberalization in order to integrate Indian economy with the global economy to benefit from the resource inflows and competition.

Its success is seen in the more than 6.5% average annual rate of growth of economy during the 8th Plan (1992-1997). Forex reserves accumulated leaving the BOP crisis in history; taming of inflation; and the foreign flows- FDI` and FII increased.

Economic Reforms

Since July 1991, India has been taking up economic reforms to achieve higher rates of economic growth so that socio-economic problems like unemployment, poverty, shortage of essential goods and services, regional economic imbalances and so on can be successfully solved. The force behind the reforms is?

• Indian economy reached a level of growth and strength to benefit from an open market economy.

• Private sector in India had come of age and was willing and capable of playing a major role

• Indian economy needed to integrate with the world with all the advantages like capital flows; technology; higher level of exports; state of art stock markets; Indian corporates can raise finances abroad and so on.

The country under the leadership of Dr. Manmohan Singh, Union Finance minister(l 991 -1996 and Prime Minister since 2004) converted the economic crisis - caused by , domestic cumulative problems of economy, political instability and gulf crisis-into an opportunity to initiate and institutionalise economic reforms to open up the economy. The deep crisis in 1991 could not be solved by superficial solutions. Therefore, structural reforms were taken up.

It was realized that by closing economy to global influences, the country was missing on technology developments and also gains from global trade. India needed exports, FDI and FII for stability on the balance of payments front and higher growth rates for social development. Worldwide, countries were embracing market model of growth, for example China, with proven results. So, India could make the historic shift from centralized planning to market-based model of growth.

Misgivings About Economic Reforms

Initially reforms were feared and resisted as there was scepticism and fear as the experience in Latin American countries in the 1980s was not a success in economic and social terms. The fears related to

• Inflation as there will be little left for domestic consumption as exports would be attractive

• Large scale unemployment due to capital intensity of growth process.

• Worsening of poverty as fiscal concerns will reduce social sector expenditure

• Flood of imports as customs duties will come down.

• food security will suffer as social sector expenditure will be reduced

Page 59: Study Kit Brochure for SSC Combined Graduate Level ...static.upscportal.com/files/study-kit/ssc-cgl/SSC-CGL-2013... · Brochure for SSC Combined Graduate Level Examination - 2014

Click Here for SSC CGL Online Coaching: http://sscportal.in/community/courses/ssc-cgl-tier-1

[Click Here to Buy This Study Kit in Hard Copy: http://sscportal.in/community/study-kit/cgl ] 58

• Pressures on labour sector due to domestic industry's inability to compete.

Some fears have indeed come true-jobless growth and uncertainty in farming. But by and large, reforms have done well.

Reforms mainly targeted the following areas :

• Dismantling the licence raj so that private sector and government were on a level playing field

•Drive public sector towards sustainable profitability and global play by dereservation; disinvestment; professionalization of management etc.

EXERCISE: 1. Match the following

(i) Planned Economy (a) characterized by an economy private sector is given substantial role and state turns its role into a facilitator.

(ii) Mixed Economy (b) is the one in which state owns and directs and eco-nomy.

(iii) Indicative (c) combines feature of Economy both capitalist mar-ket economics & socialist command economic

(a) (b) (c)

(i) 1 2 3

(ii) 3 1 2

(iii) 2 3 1

2. Consider the following statements?

(1) The state is called the Laissez faire state i the Market Economy. The French word Laissez faire means “Let do”.

(2) Command economy do not create wealth sustainably and are not conducive for innovation & efficiency.

(3) The Bombay plan was made by J.L. Nehru.

Which of the above is / are correct.

(a) none of the above (b) 1 & 2 only

(c) all of the above.

3. Which of the following countries is still command economy?

(a) USA (b) China

(c) Cuba (d) Japan.

4. Who wrote the Book titled “Planned Economy of India”?

(a) M.N. Roy

(b) Dr. Manmohan Singh

(c) Sir Mokshagundam Visvesvarayya

(d) Sir Purshotamdas Thakurdas.

5. Consider the following statements?

(1) The INC established National Planning Commission under the chairmanship of J.L. Nehru.

(2) A Plan of Economic Development of India was put forwad by leading Businessmen and Industrialists in 1944 was also called as the ‘Bombay Plan’.

(3) Indicative plan is operated under & planned economy but not command economy.

(a) all the statements are correct

(b) only 2 is correct

Page 60: Study Kit Brochure for SSC Combined Graduate Level ...static.upscportal.com/files/study-kit/ssc-cgl/SSC-CGL-2013... · Brochure for SSC Combined Graduate Level Examination - 2014

Click Here for SSC CGL Online Coaching: http://sscportal.in/community/courses/ssc-cgl-tier-1

[Click Here to Buy This Study Kit in Hard Copy: http://sscportal.in/community/study-kit/cgl ] 59

(c) only 1 & 3 are correct.

6. Match the following

(i) Bombay Plan (a) S.N. Aggarwal

(ii) People’s Plan (b) J.R.D. Tata, G.D. Birla etc.

(iii) Gandhian Plan (c) M.N. Roy

(a) (b) (c)

(i) 1 2 3

(ii) 2 3 1

(iii) 3 1 2

7. Consider the following statements?

(1) Economic Growth is measured as the percent rate of increas in real gross domestic Product.

(2) Economic Growth is not only necessary for the development but also sufficient.

(3) Growth measures quantitative increase in goods and services.

(a) all the above are correct

(b) only 1 and 3 are correct

(c) none of the above are correct.

8. Match the following

(i) First Plan (a) based on Nehru Mohalanobis Model.

(ii) Second Plan (b) Based on Rao-Manmohan Singh Model.

(iii) Eight Plan (c) K.N. Raj was one of the Main architects.

(a) (b) (c)

(i) 2 3 1

(ii) 1 2 3

(iii) 3 1 2

7. Consider the following statements?

(1) Modernization is improvement in technoloyg and is driven by innovation & investment in Research & Development.

(2) Self Reliance means relying on the resources of the country & the MNCs for investment & grwoth.

(3) Self sufficiency means that the country has all resources it needs.

(a) only 3 is correct.

(b) None of the is correct.

(c) All of the above are correct.

10. Consider the following statements?

(1) During the sixth plan (1980-1985), India for the first time resorted to borrow from IMF.

(2) With the seventh saw the beginnings of liberalization of Indian Economy.

(3) The Eight Plan was the indicative plan for the first time.

(a) only 2 and 3 are correct.

(b) all the above are correct.

(c) none of the above are correct.

Page 61: Study Kit Brochure for SSC Combined Graduate Level ...static.upscportal.com/files/study-kit/ssc-cgl/SSC-CGL-2013... · Brochure for SSC Combined Graduate Level Examination - 2014

Click Here for SSC CGL Online Coaching: http://sscportal.in/community/courses/ssc-cgl-tier-1

[Click Here to Buy This Study Kit in Hard Copy: http://sscportal.in/community/study-kit/cgl ] 60

ANSWERS: 1. (ii) 2. (b) 3. (c) 4. (c) 5. (a) 6. (iii) 7. (b) 8. (i) 9. (c) 10. (a)

Indian History

Harappan Civilization Salient Features

1. Urban Civilisation

2. Bronze Age Civilisation (Flourished during Circa 2500-2000 B.C.) 3. The largest Civilisation in geographical area of the ancient world.

4. Town-planning and well developed drainage system.

5. Gridiron layout and fortification.

Origin

1. Origin lies in various indigenous Pre-Harappan cultures.

2. Indus Civilisation was culmination of a long series of cultural evolution.

3. Emerged out of the farming communities of Sind and Baluchistan, Haryana Oujarat and Rajasthan.

4. Continuous cultural evolution from 6000 BC onwards in North West India which finally culminated in the rise of Indus Civilisation.

Phases of Development

1. Archaeological excavation & research have revealed phases of cultural development bursting up in the emergence of a full-fledged civilization at Kalibangan, Banavali and Rakhigarhi.

2. Began in Baluchistan & Sind are then extended into the plains.

3. These phases are Pre-Harappan, Early Harappan, Mature Harappan and Late Harappan

4. Extent of Harrappa:- From in North Manda (J & K) to 1600 k.m Daimabad (Maharastra.) in East Alamgirpur (U.P.) to 1100 K.m Sutkangedor (Bluchistan). Total Area was 12,99,600 Sq. k.m.

5. These phases of cultural evolution are represented by Mehargarh, Amri, Kalibangan and Lothal respectively.

Different Phases and Transformation

• Pre-Harappan - Mehargarh - nomadic herdsrnen to settled agriculture

• Early Harappan - Amri - growth of large villages and towns

• Mature Harappan - Kalibangan - rise of great cities

• Late Harappan - Lothal mature as well as decling phase.

Town planning

Facts on Town Planning

1. Features of Town Planning

Page 62: Study Kit Brochure for SSC Combined Graduate Level ...static.upscportal.com/files/study-kit/ssc-cgl/SSC-CGL-2013... · Brochure for SSC Combined Graduate Level Examination - 2014

Click Here for SSC CGL Online Coaching: http://sscportal.in/community/courses/ssc-cgl-tier-1

[Click Here to Buy This Study Kit in Hard Copy: http://sscportal.in/community/study-kit/cgl ] 61

• A great uniformity in town planning, the fundamental lay-out of prominent urban settlements exhibits apparent similarities.

• Based on ‘Grid Pattern’: streets and lanes cutting across one another at right angles dividing the city into a number of rectangular blocks. Main streets ran from north to south and were as wide as 30 feet. Streets and lanses were not paved.

2. Entire city complex was bifurcated into two distinct parts: the ‘CITADEL’ a fortified area which housed important civic and religious public buildings including granaries and residences of the ruling class and the ‘LOWER TOWN’, somewhat bigger in area and invariably located east to the former, meant. for commoners. Evidence of fortification of the lower towns as well from a few urban centres like Surkotada and Kalibangan and evidence of division of the city into three parts instead of two from Dhaulvira.

3. Use of standardized burnt bricks on massive scale in almost all types of constructions (an extraordinary feature of the contemporary civilizations), circular stones were used at Dholavira.

4. Elaborate and planned underground drainage system. Houses were connected to the main drain equipped with manholes. Mostly made up of bricks with mud mortar. Use of gypsum and lime to make it watertight. Cesspits were there inside the houses to deposit solid waste. Bricks culverts meant for carrying rain and storm water have also been found. Bricks were made in ratio of I 2 4. Size of Bricks - 7 C.m in Thick,

- 14 C.m Width

- 28 C.m Long.

5. Features of Houses

• Houses were plain and did not exhibit, any refinement and beauty. So far as the decorative value of the houses was concerned, they lacked it. In general they gave plain and un-decorative look.

• An average house comprised a courtyard and four to six rooms, a bedrooms, a kitchen, and a well presence of staircase gives indication of the second storey. Houses had side-entrances and windows were conspicuously absent. Except Lothal, where enterance were on main road and windows were found.

• Houses varied from a single-roomed tenements to houses with a number of rooms and having even a second storey. Floors were generally of beaten earth coated with cowdung. Fire-places were common in rooms. Walls were thick and square holes in them suggest of use of wooden beams. Every house was separated by another by a narrow space of ‘no-man’s land’. Staircases were usually wooden but some made up of burnt bricks have been found too. Roofs were flat. Doors were set in wooden frames and the average width of a door was one metre. Square and rectangular pillars of burnt bricks were used in larger rooms, round pillars were absent. Kitchen was small in size. A round oven meant for baking chappatis has been found.

Art and Craft

Pottery

1. Mainly two types Plain pottery and Red and Black Pottery with decoration, the majority being the former.

2. Widespread use of potter’s wheel made up of wood, use of firing technique, use of kiln.

3. Variety of Pleasing Design — Horizontal strips, Check, Chess-Board Pattern, Interesecting Circles (Pattern exclusively found), Leaves & Petals, Natural Motif — Birds, Fish, Animals, Plants, Human Figure — Rare (A Man & A Child found from Harappa), Triangles.

4. Pottery had plain bases. Few ring bases have been found.

5. Mainly famous colour of pot was pink. General design was on the red base horizontally black line on pots.

Seals

1. Seals are the greatest artistic creation of the Harappan people — cutting &polishing craftmanship is excellent.

Page 63: Study Kit Brochure for SSC Combined Graduate Level ...static.upscportal.com/files/study-kit/ssc-cgl/SSC-CGL-2013... · Brochure for SSC Combined Graduate Level Examination - 2014

Click Here for SSC CGL Online Coaching: http://sscportal.in/community/courses/ssc-cgl-tier-1

[Click Here to Buy This Study Kit in Hard Copy: http://sscportal.in/community/study-kit/cgl ] 62

2. No of seals discovered is approx. 2000

3. Made of steatite (Soft stone), Sometimes of Copper, Shell, Agate, Ivory, Faience, Terracotta.

4. Size— 4 inch to 2Y2 inch.

5. Shape — Square, Rectangular, Button, Cubical, Cylinder, Round

6. Two main types:-

• Square — carved animal & inscription, small boss at the back.

• Rectangular— inscription only, hold on the back to take a cord.

7. Colour — White appearance. Famous colour of seal was green.

8. Displays symbols - Circles, Crosses, Dots, Swastiks, Leaves of the Pipal tree.

9. Most frequently depicted animal - Unicorn

10. Other animals : Elephant, Tiger, Rhino, Antelope, Crocodile.

11. No bird were depicted on Harappan seal.

12. Purpose: Marked ownership of property. Used in applying to bales of merchandise. (Discovery of such seals beside the dockyard of Lothal).

13. Pashupati Seal has been found from Mohanjodaro. It depicts Siva seated on a stool flanked by an elephant, a tiger, a. rhinocerous, a buffalo and two antelopes / goats. Marshall identified it with Proto — Siva.

14. ‘Persian Gulf Seals’ have been discovered from Lothal.

TECHNICAL ACHIEVEMENTS

• Lost-Wax technique, used for making bronze images.

• English Bond method-Bonding system for bricks.

• Flemish Bond method-used for making staircases.

• Kiln Bricks — Evidence of Kiln has been found at Rakhigarhi

• Water Harvesting System—Dholaveera

• For small measurement binary system and for big measurement decimal system were used in Harappa.

FINDINGS AND EVIDENCES

1. Cemeteries ‘H’ & ‘R 37’ Harappa

2. Furrow mark Kalibangan

3. City divided into three parts Dholvira

4. Fragment of Woven cloth Mohanjodaro

5. City without a citadal Chanhudaro

6. Small pot (Probably an ink-pot) Chanhudaro

7. Cities where lower towns Kalibangan & were fortified Surkotada

8. Cities having both proto— Kalibangan & Harappan and mature Banwali Harappan evidences and Harappan Cultural Phases

9. Dockyard Lothal

10. Evidence of Rice Rangpur & Lothal

11. Evidence of Coffin Burial Lothal

12. Remains of Bones of Horse Surkotada

Page 64: Study Kit Brochure for SSC Combined Graduate Level ...static.upscportal.com/files/study-kit/ssc-cgl/SSC-CGL-2013... · Brochure for SSC Combined Graduate Level Examination - 2014

Click Here for SSC CGL Online Coaching: http://sscportal.in/community/courses/ssc-cgl-tier-1

[Click Here to Buy This Study Kit in Hard Copy: http://sscportal.in/community/study-kit/cgl ] 63

13. Fire altars Kalibangan, Banarvali, Lothal and Rakhigarhi

14. Terracotta Models of ships/ boats Lothal

15. Temple— like structure Mohanjodaro

16. Human skeletons huddled together indicating violent death/massacre Mohanjodaro

17. Houses having front entrances Lothal

18. Bronze rod/stick with measure marks Lothal

19. Single — roomed barracks Harappa

20. Bronze models of’ikkas’ Harappa, and ‘bullock-carts’ Chandudaro

21. Pasupati Mahadev seal (As said by John Marshall) Mohanjodaro

22. Medical Beliefs, surgery of Lothal & skull Kalibangan

23. Seal depicting Mother Mohanjodaro Goddess with a plant growing from her womb

24. Painting on a jar resembling the story of the cunning fox of panchtantra Lothal

25. Harappan game similar to chess Lothal

26. Copper rhinocerous Daimabad

27. Copper chariot Daimabad

28. Copper Elephant Daimabad

29. Devastation by flood Dhanhudaro, Mohanjodaro & Lothal

30. Collegiate building Mohanjodaro

31. Assembly Hall Mohanjodaro

32. Granaries Mohanjodaro & Harappa

33. Steatite figure of a bearded man Mohanjodaro

34. Bronze dancing girl (11.5 cm) Mohanjodaro

35. Cylindrical seals of Gigamesh and Ekindu Mesopotamian type, three in number Mohanjodaro

36. Warehouse Lothal

37. Granary outside citadel Harappa

Page 65: Study Kit Brochure for SSC Combined Graduate Level ...static.upscportal.com/files/study-kit/ssc-cgl/SSC-CGL-2013... · Brochure for SSC Combined Graduate Level Examination - 2014

Click Here for SSC CGL Online Coaching: http://sscportal.in/community/courses/ssc-cgl-tier-1

[Click Here to Buy This Study Kit in Hard Copy: http://sscportal.in/community/study-kit/cgl ] 64

38. Working platcform Harappa

39. Sandstone male dancer Harappa

40. Terracotta figurine of a horse Mohanjodaro

41. Absence of mother goddess figurines Rangpur

42. Bead-making factory Chanhudaro & Lothal

43. Absence of seals Alamgirpur

44. Shell-ornament makers Chanhudaro, factory Balakot and Lothal

45. Metal workers factory Chanhudaro & Lothal

46. Persian Gulf seals Lothal

47. City having a middle town apart from the citadel and the lower town Dholvira

48. A merchant house Lothal

49. Impressions of cloth on sealing Lothal

50. Six types of pottery Kalibangan

51. Evidence of double burial Lothal

52. Evidence of pot-burial Surkotada

53. Furnace Rakhigarhi

54. Kiln Rakhigarhi

55. Boustro phedon Kalibangan

56. Camel bones Kalibangan

57. Horse skeleton Surkotada

58. Horse teeth Rana Ghundai

59. Regalia Kunal

60. Stone cut water reservoir Dholavira

Indus Civilisatlon-General Aspects

CUSTOMS AND AMUSEMENTS

• A good number of toys — Clay Cart, Rattles, Bulls with mobile heads, Monkey with movable arms, Dices, Chessboard.

• Hunting, Fishing, Cock-Fighting.

COSMETICS

Tweezers, Ear-scoop, Piercer, Antimony rods, Ivory comb, Mirrors, Hair pins, Round buttons, Minor razors, Kohl pots and sticks

DRESS AND ORNAMENTS

1. Females were scantily dressed, they wore a short skirt especially figurines of mother Goddess.

2. The male wore a robe, sometimes embroidered.

3. Cotton was used, no evidence of Linen and Silk.

4. No evidence of footwear.

Page 66: Study Kit Brochure for SSC Combined Graduate Level ...static.upscportal.com/files/study-kit/ssc-cgl/SSC-CGL-2013... · Brochure for SSC Combined Graduate Level Examination - 2014

Click Here for SSC CGL Online Coaching: http://sscportal.in/community/courses/ssc-cgl-tier-1

[Click Here to Buy This Study Kit in Hard Copy: http://sscportal.in/community/study-kit/cgl ] 65

5. Special care of hair, reference of pony-tail, bun and braided hair.

6. Curley hair (clay figure from Mohanjodaro).

7. Beards (not very long), shaven upper lips in the figure of Yogi.

8. Necklaces, Beads, Girdle, Bracelet, Fillets, Finger rings, Bangles, Nose Ornaments, Anklets.

RELIGION

1. Predominance of Mother Goddess, denotes people’s faith in fertility cult.

2. Male deity — Pasupati Shiva.

3. Animal worship (200l atry)— Unicorn bull and humped bull.

4. Tree worship — Pipal.

5. Fertility Cult — Phallus worship.

6. Nature worship.

7. Amulets and Talisman (Mohanjodaro)

8. Sacred bath — Water Cosmology

BURIALS

1. Bodies were extended in north-South direction.

2. Cemeteries of Harappa, Mohanjodaro, Lothal, Kalibangan, Rakhigarhi and Ropar located around the outskirts.

3. Three forms of burials — complete burial, Fractional burial and post-cremation burial.

4. General practice was body lying on its back and head to the north.

5. Coffin burial from Harappa.

6. Pot burial from Surkotada (Sacrophagus tradition).

7. Double burial from Lothal.

8. Pit burial from KaIibangan.

9. Cemetety —R-37 (Harappa).

10. Cemetery — (Post Harappan)

WEIGHTS & MEASURE

1. Standard weights and measures, uniformity and accuracy of denominations.

2. Weights were made of a variety of material — state, jasper, chert, alabaster, limestone and quartzile, but main1 polished chert.

3. For larger weights decimal system was used and for smaller ones binary system was followed.

4. The unit weight had the calculated value of 0.8 750 gms, the largest weights was 10970 gms.

5. A few specimens of scales used with the weights appear to be a very ordinary patterns comprising a bronze bar with suspended copper pens.

6. Broken Ivory were used as a scale in Chanhudaro.

7. Bronze scale was used in Lothal.

WEAPONS

1. Mainly offensive weapons were found in Harrappa.

2. Spears, Axes, Arrow-heads.

3. No defensive weapons were found in Harrappa.

CROPS

1. Main crops: Wheat & Barley.

2. Others Peas, Rai, Linseed, Mustard, Cotton, Dates.

3. No evidence of sugarcane.

Page 67: Study Kit Brochure for SSC Combined Graduate Level ...static.upscportal.com/files/study-kit/ssc-cgl/SSC-CGL-2013... · Brochure for SSC Combined Graduate Level Examination - 2014

Click Here for SSC CGL Online Coaching: http://sscportal.in/community/courses/ssc-cgl-tier-1

[Click Here to Buy This Study Kit in Hard Copy: http://sscportal.in/community/study-kit/cgl ] 66

4. Rice was produced region (Rice husk has been found from Lothal & Rangpur).

ANIMALS & BIRDS

Elephants, Cats, Dogs, Camels, Asses, Buffalo, Dear, Rhino, Goat, Unicorn, Pigs, Tortoise, Fowls, Ox; Fish, Stag, Antelope, Patridge.

METALS

Copper, Silver, Gold, Jade, Lapis Lazulli, Lead. Tin, Ingot (Lead+Silver), Electrum (Silver & Gold)

TOOLS

Needles, Razors, Sickles, Fish-hooks, Saws, Chappers, Spades, Knoves, Chisels, Spoons

PROFESSIONS

Potters, Copper and Bronze workers, Stone workers, Builders, Brick-makers, Priests, Faience workers, Farmers, Traders.

RARITIES

Rice, Horse, Round Cylindrical seals, Plough, Whetstones.

ABSENCE

Iron, Winows, Tp1e, Fork, Hair dyes, Round columns, Silk fabrics, Swords, Sheilds, Metalic Money, Water closets Brick-Lamps, Footwares, Linen, Wool, Golden finger rings, Indigenous, Gradual decline.

LANGUAGE & SCRIPT

Yet not deciphered, pictographic or ideographic approximately 450 signs have been listed, written from right from left and left to in alternate lines style known as Boustrophedan, inscriptions are short.

TRADE

Evidences

• Sumerian text make a mention of Meluha which is identified with Indus Civilization and two intermediate trading stations Dilmun (Bahrain) and Makan (Makaran Coast).

• Appearance of Indus seals in the Mesopotamian cities of Ur, Kish,Susa, Tell Asmar, Lagash.

• Cylindrical seals of Mesopotamian type have been found in Indus valley (Mohanjodaro).

• “Persian Gulf Seals” of intermediate trading stations have been discovered from Indus region (Lothal).

• Discovery of”Reserved slip ware” of Mesopotamian type from Harappa & Lothal.

Internal Trade

With Saurashtra, Maharastra, South India, Rajasthan, ports of western UP.

Ports

Port was found in Lothal. This is the first man-made port in the world.

External Trade

With Mesopotamia (Modern Iraq), Persian Gulf region and probably Egypt.

Item of Imports

Gold, Silver, Copper, Tin, Lapis Lazuli,-Amethyst, Agate, Jade, Shells.

Items of Exports

• Agricultural products Wheat, Barley, Peas, Oil Seeds

• Finished Products Cotton goods, Pottery1 Beads, Shells, Terrocotta Products, Ivory Products.

Main Imports And Areas

Agate Saurashtra

Amethyst Maharashtra

Carnelians Saurashtra, West India

Page 68: Study Kit Brochure for SSC Combined Graduate Level ...static.upscportal.com/files/study-kit/ssc-cgl/SSC-CGL-2013... · Brochure for SSC Combined Graduate Level Examination - 2014

Click Here for SSC CGL Online Coaching: http://sscportal.in/community/courses/ssc-cgl-tier-1

[Click Here to Buy This Study Kit in Hard Copy: http://sscportal.in/community/study-kit/cgl ] 67

Chalcedonies Saurashtra, West India

Copper Rajasthan (Khetri), South India, Arabia, Baluchistan

Gold North Karnataka, Afganistan, Persia

Jade Central Asia

Lapis Lazuli Afghanistan

Lead East India/South India

Shells Saurashtra, Deccan

Silver Afghanistan of Iran

Tin Afghanistan, Jharkhand (Hazaribagh)

Turquoise Persia

Important Sites & Archaeologist

Sites Year Archaeologist

Harappa 1921 Daya Ram Sahni

Mohanjodaro 1922 R. D. Banerjee

Aniri 1929 M. G. Majumdar

Chanhu-Daro 1931 M. G. Majumdar

Suktagendor 1927 Aurel Stein

Kot Diji 1935 Ghurey

Ropar 1953 Y.D. Sharma

Banawali 1973 R.S. Bist

Kalibangan 1953 A. Ghosh

Lothal 1957 S.R. Rao

Alamgirpur 1958

Rangpur 1931 M.S. Vats

Surkodata 1964 Jagatpati Joshi

Dholvira Dr. J.P. Joshi and R.S. Bist

Area-Wise Distribution of Settlements

1. Sind -Mohenjodaro, Amri, Kot Diji, Pandiwahi, Tarkai Quila, Othmaujo Butha, Sukkur, Allahdino, Chanhudaro, Rohri, Lohumjo darn, All Murad, Jhukar.

2. Baluchistan-Mehargarh, Damb Sadaat, Killi Ghul Mohmmed, Rana Ghundai, Siah Damb, Moghul Ghundai, Anjira, Na!, Kulli, Leval, Dabar KOt, Balakot.

3. Afghanistan-Mundigak, Shortu Ghai

4. West Punjab-E-Iarappa, Jalilpur, Ganeriwala, Jhang, Saidhanwala, Derawar.

5. Gujarat-Dholvira, Lothal, Surkotada, Bhagatrav, Rangpur, Rojadi, Desalpur.

6. Rajasthan-Ka!ibangan, Siswal, Bara, Bhagwanpura, Hulas.

7. Uttar Pradesh-Alamgirpur, Manpur, Bargaon, Hulas and Mandi

Page 69: Study Kit Brochure for SSC Combined Graduate Level ...static.upscportal.com/files/study-kit/ssc-cgl/SSC-CGL-2013... · Brochure for SSC Combined Graduate Level Examination - 2014

Click Here for SSC CGL Online Coaching: http://sscportal.in/community/courses/ssc-cgl-tier-1

[Click Here to Buy This Study Kit in Hard Copy: http://sscportal.in/community/study-kit/cgl ] 68

8. Haryana-Banwali, Rakhigarhi and Kunal.

9. Punjab-Ropar, Sarai Khola, Kotla Nihang Khan.

Settlements and their Riverine Locations

Bhadar Desalpur

Bhadar Rojdi

Bhadar Rangpur

Bhogava and Sabarmati Lothal

Chenab Manda

Ghaggar (Left bank) Kalibangan

Hindan Alamgirpur

Indus Chanhudaro

Indus Gumala

Indus (Left bank) Kot Diji

Indus (Right bank) Mohenjodaro

Pravara Daimabad

Rangoi (Saraswati) Banwali

Ravi (Left bank) Harappa

Sutlaj Kotla Nihang Khan

Other Related Facts

1. Mandu (J & K) Dalimábad (Maharashtra) Sutkagendor (PAK) and Alamgirpur (western U.P.) are the northernmost, southernmost, westernmost and eastemmost points respectively.

2. The four Harappan sites in descending order (the first being jhe largest) are:

• Mohenjodaro in Sind (Pak)

• Ganeriwala in Bahwalpur (Pak)

• Harappa in West Punjab (Pak)

• Dholvira in Gujarat (India).

2. The largest Harappan settlement in India is Rakhigarhi in Haryana.

3. Mepotamian literature stopped mentioning Meluha by the end of 1990 BC — Indicates closure of trade links.

4. Crossed furrows, widely spaced in one direction and closely spaced in other, gram or seasmum sown at the former and mustard at the later have been found at Kalibangan.

5. The ‘great bath’ ofMohenjodaro measures l2 x 7 x 3 metres.

6. Harappan wheels were solid without spokes.

7. Harappan Weight & Measures systems were as follows:

• Of lower denomination — followed Binary system — 1, 2, 4, 8, 64 . . .160

• Of higher denomination — followed Decimal system — 16, 320, 640, 1600, 3200

8. The largest number of settlements are in Ghaggar — Hakar valleys.

9. Principal Harappan Crops and the areas of their distribution are as follows:

• Barley — Rajasthan

• Wheat & Barley — Sind & Punjab

Page 70: Study Kit Brochure for SSC Combined Graduate Level ...static.upscportal.com/files/study-kit/ssc-cgl/SSC-CGL-2013... · Brochure for SSC Combined Graduate Level Examination - 2014

Click Here for SSC CGL Online Coaching: http://sscportal.in/community/courses/ssc-cgl-tier-1

[Click Here to Buy This Study Kit in Hard Copy: http://sscportal.in/community/study-kit/cgl ] 69

• Rice & Millet — Gujarat

10 Sinage on wall — Dholavira

11. Double fortification — Kuntasi

12. Thirteen roomed house from the overlap period-Bhagvanpura

13. Double spiral headed copper pin-Manda

Exercise: 1. Which was the most frequently depicted aminal.

(a) Elephant (b) Unicorn

(c) Rhino (d) Tiger.

2. Consider the following statements and mark the option which is correct.

(i) Pashupati Seal has been forund from Lothal.

(ii) The Harappan bricks were made in ratio of 1:2:4.

(iii) For small measurement binary system and for big measurement decimal system were used in harappa.

(iv) Persian Gulf seals have been found in Lothal.

(a) i, ii & iii (b) ii & iv

(c) ii, iii & iv (d) all of the above.

3. Match the following

(i) Cemetrics ‘H’ & R 37’ (a) Daimabad

(ii) Dockyard (b) Mohanjodaro

(iii) Pashupati Mahadev Seal (c) Harappa

(iv) Copper Chariot (d) Lothal

(a) (b) (c) (d)

(1) (iv) (iii) (i) (ii)

(2) (iv) (iii) (ii) (i)

(3) (i) (ii) (iii) (iv)

(4) (i) (iii) (ii) (iv)

4. Remains of Horse bones have been found from.

(a) Lothal (b) Surkotada

(c) Kalibangan (d) Dholavira

5. A Seal depicting Mother Goddess with a plant growing form her womb has been found from:—

(a) Harappa (b) Kalibangan

(c) Daimabad (d) Mohanjodara

6. There has been an absence of seal in

(a) Alamgirpur (b) Mohanjodaro

(c) Harappa (d) Lothal

7. A stone cut water Reservoir has been discovered from

(a) Surkotada (b) Kalibangan

(c) Dholavira (d) Harappa.

Page 71: Study Kit Brochure for SSC Combined Graduate Level ...static.upscportal.com/files/study-kit/ssc-cgl/SSC-CGL-2013... · Brochure for SSC Combined Graduate Level Examination - 2014

Click Here for SSC CGL Online Coaching: http://sscportal.in/community/courses/ssc-cgl-tier-1

[Click Here to Buy This Study Kit in Hard Copy: http://sscportal.in/community/study-kit/cgl ] 70

8. Consider the following statements and mark the option which is correct.

(i) The predominence of Mother Goodess denotes people’s faith in fertility cult.

(ii) Mainly offensive weapons were found in Harappa.

(iii) Six types of pottery have been discovered from Kalibangan.

(iv) Painting on a jar resembling the story of the cunning for the Panchtantra has been found from Lothal.

(a) i, & iv (b) all of the above.

(c) none of the above (d) iii only.

9. Match the following

(i) Harappa (a) copper elephant

(ii) Daimabad (b) Bronze Dancing girl

(iii) Mohanjodaro (c) Granary outside citadel

(iv) Furrow Marks (d) Kalibangan

(a) (b) (c) (d)

(1) (ii) (i) (iii) (iv)

(2) (i) (ii) (iii) (iv)

(3) (iv) (iii) (ii) (i)

(4) (ii) (iii) (i) (iv)

10. Consider the following statements and mark the option which is correct.

(i) The first man made port was found in Harappa.

(ii) The main crops of Indus Valley civilization were wheat & Barley.

(iii) The Largest Harappan Settlemtn in India is Rakhigarhi in Haryana.

(iv) The Largest number of settlements are in Ghaggar-Hakar Valleys.

(a) i & ii (b) all of the above

(c) ii, iii & iv (d) none of the above.

11. Match the following

(i) Mohanjodaro (a) Daya Ram Sahni

(ii) Chanhudaro (b) S.R. Rao

(iii) Harappa (c) R.D. Banerjee

(iv) Lothal (d) M.G. Majumdar

(a) (b) (c) (d)

(1) (iii) (i) (ii) (iv)

(2) (iii) (iv) (i) (ii)

(3) (i) (ii) (iii) (iv)

(4) (i) (ii) (iv) (iii)

12. Match the following

(i) Ghaggat (a) Mohanjodaro

(ii) Ravi (b) Kalibangan

(iii) Indus (c) Lothal

(iv) Bhogava (d) Harappa

(a) (b) (c) (d)

Page 72: Study Kit Brochure for SSC Combined Graduate Level ...static.upscportal.com/files/study-kit/ssc-cgl/SSC-CGL-2013... · Brochure for SSC Combined Graduate Level Examination - 2014

Click Here for SSC CGL Online Coaching: http://sscportal.in/community/courses/ssc-cgl-tier-1

[Click Here to Buy This Study Kit in Hard Copy: http://sscportal.in/community/study-kit/cgl ] 71

(1) (i) (ii) (iii) (iv)

(2) (iii) (iv) (i) (ii)

(3) (iii) (i) (iv) (ii)

(4) (ii) (iii) (iv) (i)

13. Consider the following statements and mark the option which is correct.

(i) Rice Husk has been found from Lothal & Rangpur.

(ii) Evidence of signage on wall has been discovered from Dholavira.

(iii) A thirteen roomed house from the overlap period has been discovered from Bhagvanpura.

(iv) The steatite figure of a bearded man has been recovered from Mohanjodaro.

(a) All of the above (b) ii & iii

(c) none of the above (d) i & iv

14. Match the following

(i) Mohanjodaro (a) Absence of Mother Goddess figurines

(ii) Chanhudaro (b) Assembly Hall & Collegiate Building

(iii) Rangpur (c) Bead Making factory

(iv) Harappa (d) Sandstone Make Dancer

(a) (b) (c) (d)

(1) (i) (iii) (ii) (iv)

(2) (iii) (i) (ii) (iv)

(3) (ii) (iii) (iv) (i)

(4) (ii) (iv) (i) (iii)

15. Consider the following statements and mark the option which is correct.

(i) Broken Ivory were used as a scale in Chanhudaro.

(ii) During Burial bodies were extended in North-South Direction.

(iii) Ganerinkla site is situated in Bahwalpur, Pakistan.

(iv) The ‘Great Bath’ of Mohanjodaro Measures 12 × 7 × 3 mts.

(a) i, ii & iii (b) ii & iii

(c) ii & iv (d) all of the above.

Answers: 1. (b) 2. (c) 3. (1) 4. (b) 5. (d) 6. (a) 7. (c) 8. (b) 9. (4) 10. (c) 11. (2) 12.

(3) 13. (a) 14. (2) 15. (d)

Modern India

Revolts & Rebellions PARTICIPANTS IN THE REVOLT

1. Bahadur Shah II Delhi

2. Laxmi Bai Jhansi

3. Kunwar Singh Jagdishpur

4. Hazarat Mahal Lucknow

5. Maulavi Ahmadullah Faizabad

Page 73: Study Kit Brochure for SSC Combined Graduate Level ...static.upscportal.com/files/study-kit/ssc-cgl/SSC-CGL-2013... · Brochure for SSC Combined Graduate Level Examination - 2014

Click Here for SSC CGL Online Coaching: http://sscportal.in/community/courses/ssc-cgl-tier-1

[Click Here to Buy This Study Kit in Hard Copy: http://sscportal.in/community/study-kit/cgl ] 72

6. Tantia Tope Gwalior Kanpur

7. Nana Saheb Kanpur

8. Mangal Pandey Barrackpore

9. Hakim Ahsanullah Chief Advisor to Bahadur Shah II Delhi.

10. Firuz Shah Relative of Bahadur Shah II Delhi

11. General Bakht Khan Delhi

12. Rao Sahib Kanpur

13. Azimulah Khan Kanpur

14. Ahmad Ullah Lucknow

15. Khan Bahadur Khan Bareilly

16. Amar Singh Jagdishpur

CENTRES OF REVOLT & THEIR LEADERS

Delhi Bahadur Shah was the nominal leader the real command was under General Bakht Khan.

Kanpur The revolt was led by Nana Sahib. Battle was mainly led by Tantia Tope. Azimullah Khan was another loyal servant of Nana Sahib.

Lucknow The revolt was led by Hazarat Mahal the begum of Avadh, Maulvi Ahmedullah of faizabad was also one of the great leaders.

Jhansi Rani Laxmi bai assumed the leadership of the Sepoys. Later she captured Gwalior with the help of Tantia Tope and Afghan guards.

Bareilli Khan Bahadur Khan proclaimed himself as Nawab Nazim.

Arrah Kunwar Singh, the ruined Zamindar of Jagdishpur was the main leader.

FATE OF THE LEADERS OF THE REVOLT

Bahadur Shah II He was arrested and deported to Rangoon where he died in 1862.

Nana Sahib After being defeated he refused to surrender and escaped to Nepal in early 1859, never to be heared again.

Bengum Hazrat The Begum of Avadh was compelled to hide in Nepal after the capture of Lucknow by campbel.

Rani Laxmi Bai The Rani of Jhansi died in the battle field in June 1858.

Tantia Tope Tantia Tope escaped to the jungles of central India in April 1859. He was betrayed by a zamindar friend and captured and put to death after a hurried trial on 15th April 1859.

Kunwar Singh The leader of the revolt in Bihar died on 9th May 1859 after successfully carrying on the revolt.

SUPPRESSION OF THE REVOLT

1. Gen. John Nicholsan Delhi

• Henry Benard and Brig. Wilson besieged Delhi

• Accession of Kashmiri gate by J.Nicholsan and captured city embrance and Delhi.

• Bahadurshah and killed his sons and grandsons.

2. Maj. Hudson Killed bhadur Shah’s sons and grand sons in Delhi

3. Sir Hugh Wheeler Defence against Nana Saheb’s forces in the beginning

4. Gen. Neil Kanpur (killed Indians as a revenge against the killing of English by Nana Shahib’s forces), Banaras, Allahabad.

Page 74: Study Kit Brochure for SSC Combined Graduate Level ...static.upscportal.com/files/study-kit/ssc-cgl/SSC-CGL-2013... · Brochure for SSC Combined Graduate Level Examination - 2014

Click Here for SSC CGL Online Coaching: http://sscportal.in/community/courses/ssc-cgl-tier-1

[Click Here to Buy This Study Kit in Hard Copy: http://sscportal.in/community/study-kit/cgl ] 73

5. Sir Colin Compbell Final recovery of Kanpur on 6th December 1857 captured Lucknow on 21st March 1858.

6. Henry Lawrence Chief Commissioner of Awadh (died)

7. Maj. Gen. Havelok Dfeated the rebels (Nana Saheb’s force) on 17th July 1857.

8. William Taylor Suppressed the revolt at Arrah in August 1857.

9. Hugh Rose Suppressed the revolt at Jhansi

OPINION OF THE HISTORIANS ABOUT THE NATURE OF THE REVOLT

Disraeli A national revolt

Dr. Tarachand Specially medieval but efforts to powerless class get their lost power.

J.Outaram & W.Taylor A Hindu-Muslim cons-piracy

Jawaharlal Nehru Not only mutiny, formed the shape of Civil Rebelion the real form was of feudalistic though some national-isitc elements were also present

Maulana Abdul Kalam Indian national charac-ter had sank very low.

R.C. Majumdar Neither first nor national nor war of Independence.

S.N. Sen Inherited in the constitution of British Rule an effort by the conservative elements to turn back the clock.

T.R. Holmes Conflict between civilization and Barbarism

V.D. Savarkar First war of Indian Independence.

BOOKS AND AUTHORS ON 1857

S.N.Sen 1857

P.C. Joshi Rebellion, 1857

H.P. Chattopdhyaya The Sepoy Mutiny, 1857

A.T. Embree 1857 in India

B.C. Majumdar The Sepoy Mutiny and the Revolt of 1857

S.B. Choudhury Civil Rebellion in the Indian Mutinies, 1857-59

Theories of the Indian Mutiny

Etic Stokes The Peasent and the Raj

Maulan Abdul Kalam Azad Eighteen Fifty Seven

V.D. Savarkar The Indian war of Independence 1857

Ashok Mehta 1857 a great Revolt

John Caye History of Sepoy war

Holms Sepoy war

TRIBAL MOVEMENTS

Chuar (Midnapur, Bengal 1766-1772 & 1795-1816)

Hos (Singhbhum 1820, 1822 & 1832)

Kol (Chottanagpur 1831-1832) led by Buddhu Bhagat

Ahom (Assam 1828-33) Led by Gomadhar Konwar

Khasi (Khasi Hills-Assam and Meghalaya 1829-32) Led by Triat Singh and Bar Manik.

Bhills Khandesh 1817-19, 1825, 1831, 1847) Led by Sewaram agaist the company owing to agrarian hardship.

Page 75: Study Kit Brochure for SSC Combined Graduate Level ...static.upscportal.com/files/study-kit/ssc-cgl/SSC-CGL-2013... · Brochure for SSC Combined Graduate Level Examination - 2014

Click Here for SSC CGL Online Coaching: http://sscportal.in/community/courses/ssc-cgl-tier-1

[Click Here to Buy This Study Kit in Hard Copy: http://sscportal.in/community/study-kit/cgl ] 74

Kolis (Sahyadri Hills-Gujrat-Maharashtra 1824, 1829, 1839, 1844-48)

Koyas Rampa Godavari region of Andhra Pradesh 1840, 1845, 1858, 1861-62, 1879, 1880, 1896, 1916, 1922-24). In 1922-24 Led by Alluri Sitarmaraju.

Santhais (Rajmahal Hills-Santhal Pargana, Bihar 1855-56) led by Sindhu and Kanhu.

Naikda (Panch Mahals-Gujarat 1858-59 and 1868) led by Rup Singh and Joria Bhagat

Kacha Naga (Cachar-Assam 1882) led by Sambhudam

Mundas Chotanagpur 1899-90. Known as ulgulan led by Birsa Munda against the erosion of their Kuntkatti land system, recruitment of forced labour (Beth-Begar) and against the activities of the Christian missionary. They attacked churches and police station. Birsa Munda was taken prisoner by the British and the movement was suppressed.

Bhils (Banswara, Suthi Dungarpur-Rajasthan 1913) led by Govind Guru

Oraons (Chottanagpur 1914-15) led by Jatra Bhagat and Tana Bhagat.

Kukis (Manipur 1917-19) led by Jadonang

Ramoshi In 1822-29 they rose against deposition of Raja Pratap Singh of Satara in 1839

Chanchus (1921-22) in Andhra Pradesh.

Sapha Har Among the Santhals in 1870's, earlier in 1855 by Sidhu and Kanhu against Dikus.

Pahariyas The British expansion on their territory led to an uprising by the martial Pahariyas of the Raj Mahal Hills in 1778

Kharwar The Kharwars of Bihar in the 1870s.

Bhuyan and Juang The first uprising of 1867-68 was under the leadership of Ratna Nayak

Singphos Assam in early 1830

Zeliangsong The Zeliangsong Movement of the Manipur Nagar was launched by the Zemi, Liangmei and Rongmei

Naga Under Jodanang (1905-31). The other leader was Gaidinliu who fed the Heraka cult.

NON-TRIBAL MOVEMENTS

Farazi Movement Led by Haji Shariatullah and his son Dadu Mian degeneration of Islamic Faridpur, East Bengal society and loss of power to the British. It supported the cause of tenants against the Zamindara Dadu Mian was finally arrested and confined to Alipore jail.

Godkari Rebllion Assumption of direct administration of Kolhapur by British and (Kalhapru 1844-45) resentment of Gadkris against revenue policy. Final suppression of the movement by the British.

Pagal Panthis Led by Karan Shah and his son. Tipu. A semi-religious sect. Rose against the oppression of the Zamindars.

Sambhalpur Led by Srendra Sai, inter- Uprisings. -ference of British in the internal affairs of (Orissa 1840-41) Sambalpur, Surendra

Sai was finally arrested and imporiosned by the British (1840)

Sanyasi Revolt Led by religious monks and dispossed zamindars. Against restrictions (Bengal 1760-1800) imposed by the English company on visit of holy place and ruin of peasent and zamindars.

Page 76: Study Kit Brochure for SSC Combined Graduate Level ...static.upscportal.com/files/study-kit/ssc-cgl/SSC-CGL-2013... · Brochure for SSC Combined Graduate Level Examination - 2014

Click Here for SSC CGL Online Coaching: http://sscportal.in/community/courses/ssc-cgl-tier-1

[Click Here to Buy This Study Kit in Hard Copy: http://sscportal.in/community/study-kit/cgl ] 75

The Kuka Revolt The Kuka movement started as a religious movement though it later began to acquire political overtone. The movement was began in 1840 by Bhagat Jawahar Mal or Sian Saheb in West Punjab. The movement aimed to purge Sikhism of its by preaching abolition of castes and similar discriminations and discouraging the eatheg of meat and taking of drugs. Woman were encouraged to step out of seclusion. After the British conquered the Punjab, the movement began to focus on achieving Sikh sovereignty. The British followed extremely repressive measures from 1862 to 1872 to suppress the movement.

PEASANT MOVEMENTS

All India Kissan Sabha Founded at Lucknow in 1936 with Shajananda Saras-wati as its first Chairman. It main demands included 50% cut in revenue rents, full occupancy right to tenants abolition of begar and restoration of customary forest rights.

Bijolia Movement Leaders Sitaram Das, Vijay Pathak Singh. Manik Lal Verma & Haribabu (Rajasthan 1905, 1913, Upadhya. The movement arose due to the imposition of 86 different types 1916, 1927) of cesses on the Kisans. The peasants refused to pay cesses and cultivate their land tried to migrate to neighbouring areas. In 1927, peasant adopted satyagrah methods to fight fresh cessed & begar.

Borsad Satyagraha Led by Vallabhai Patel. Against the Bombay govern-ments decision to hike (Gujarat 1928) The revenue on cotton by 22 percent while its prices declined in the international market. The pea-sant organised on revenue move-ment. Ultimately the government gave up their plans to revise the rate.

Champaran Satyagraha It was the movement of indigo peasants (working under tinnkathiya system) (Bihar 1917) against the oppression of the planters. Gandhi asked the peasants to resort to satyagraha

Deccan Riots Led by traditional headmen (Patel). The fall in the prices of cotton and (Maharashtra and Six hike in land revenue forced the peasants to take loans on high rates from Taluks of Poona & Gujarat and Marwari money lenders. The move-ment took the form of Ahmednagar District) forceful seizure & buming of debt bonds by peasants.

Eka Movement Founded by Madari Pasi. The main demands of the movement was (Avadh (1921) conver-sion of pro-duce (batai) into cash.

Forest Satyagrah Led by N.V. Rama Naidu and N.G. Ranga. It was star-ted against the (South India 1931) oppressive zamin-dars.

Indigo Revolt Led by Bishnu Biswas and Diagambar Biswas. It was the revolt of indigo (Bengal 1859-60) peasants against the British painters and their oppression refusal to take advances and enter into contract.

Kaira (Kheda) Satyagraha Leader Vallabhai Patel. The peasants arose against the demand of land raha (Gujarat 1918) revenue inspite of crop failure. The peasant collectively refused to pay lan revenue Govern-ment was forced to offer terms accep-table to the peasants.

Moplah Uprising Leader Sayyed Alwi & Sayyid Fazi. The movement arose against the (Malabar 1836-54) vastly enhanced rights of Hindu Namboodri & Nair Jenmis which had 1896, 1921) worse-ned the condition of Muslim lease hol-ders and cultivators the Moplahs. the Moplahs attacked the Jenmis property & temples it was purely agrarian in nature.

Oudh Kisaan Sabha Founded by Gauri Shankar Mishra, Main leader Baba Ramchandra. The (Uttar Pradesh 1920) organization demanded abolition of begar, reduction of rents and social boycott of oppressive zamindars.

Page 77: Study Kit Brochure for SSC Combined Graduate Level ...static.upscportal.com/files/study-kit/ssc-cgl/SSC-CGL-2013... · Brochure for SSC Combined Graduate Level Examination - 2014

Click Here for SSC CGL Online Coaching: http://sscportal.in/community/courses/ssc-cgl-tier-1

[Click Here to Buy This Study Kit in Hard Copy: http://sscportal.in/community/study-kit/cgl ] 76

Pabna Movement Bangal Led by Ishwar (1870-80) Chandra Roy, Shambhu Pal and Khoodi Mollah. The movement rose as a

result of high increase in zamin-dari rent. The peasant demanded changes in mea-surement standard, abolition of abwabs and re-duction in rent.

Ramosi Movement Led by Vasudev Balwant Phadke. Owing to hardship caused to the (Maha-rashtra 1879) pea-sants by the Deccan famine of 1876-77 Phadke organised the Ramosi peasants & revolted against the British and thought of establish-ing a Hindu Raj.

Tabhaga Movement To miplement the Flaud Commission recommendations of Tebhaga that is (Bengal 1964-47) 2/3 of crop to the bargadars working on the land rented from the jotadars.

Telegana Movement Biggest peasants guirella war in the indian history. It rose against the (Andhra Pradesh exploitation of peasant by Desmukh and Hagir-dars. The movement retained 1946-48) dimensions of national liberation struggle against the Nizams

U.P. Kisan Sabha Founded by Indra Narayan Dwivedi & Gowi Shankar Mishra. The (1918) organization was against the threat of bedakhli eviction and the Jajmani system.

EXERCISE: 1. Match the following

(i) Laxmi Bai (a) Faizabad

(ii) Mangal Pandey (b) Jhansi

(iii) Maulvi Ahmadullah (c) Barrackpore

(a) (b) (c)

(1) (iii) (i) (ii)

(2) (i) (ii) (iii)

(3) (i) (iii) (ii)

2. Consider the following statements and mark the option which is correct.

(i) In Delhi, Bahadur Shah was the nominal leader and the real command was under General Bakht Khan.

(ii) In Arrah, Rani Laxmi Bai assumed the leadership of the Sepoys.

(iii) In Lucknow, the revolt was led by Begum Hazrat Mahal.

(a) i & iii (b) all of the above

(c) none of the above (d) ii

3. Who among the following was the author of Civil Rebellion in the Indian Mutinies 1857-89.

(a) B.C. Majumdar (b) P.C. Joshi

(c) S.B. Chaudhary (d) V.D. Savarkar

4. Match the following

(i) Ahom (a) Savaram

(ii) Khasi (b) Buddhu Bhagat

(iii) Bhils (c) Gomdhar Kunwar

(a) (b) (c)

(1) (i) (ii) (iii)

(2) (iii) (ii) (i)

Page 78: Study Kit Brochure for SSC Combined Graduate Level ...static.upscportal.com/files/study-kit/ssc-cgl/SSC-CGL-2013... · Brochure for SSC Combined Graduate Level Examination - 2014

Click Here for SSC CGL Online Coaching: http://sscportal.in/community/courses/ssc-cgl-tier-1

[Click Here to Buy This Study Kit in Hard Copy: http://sscportal.in/community/study-kit/cgl ] 77

(3) (ii) (iii) (i)

5. Consider the following statements and mark the option which is correct.

(i) R.C. Majumdar described the 1857 Revolt as neither first nor national war of Independence.

(ii) T.R. Holmes described the 1857 Revolt as conflict between civilization & Barbarism.

(iii) Jawaharlal Nehru said that 1857 Revolt was nothing but a Hindu-Muslim conspiracy.

(a) ii (b) i & ii

(c) all of the above (d) none of the above

6. Match the following

(i) Kukis (a) Lhota Nagpur

(ii) Mundas (b) Assam

(iii) Singhpos (c) Manipur

(a) (b) (c)

(1) (ii) (iii) (i)

(2) (i) (ii) (iii)

(3) (ii) (i) (iii)

7. Consider the following statements and mark the option which is correct.

(i) Pagal Panthis (a) rose against the oppression of Zamindars

(ii) Farazi Movement (b) Led by Religious Monks

(iii) Sanyasi Revolt (c) Against degenera-tion of Islamic Faridpur and Loss of power to the British.

(a) (b) (c)

(1) (ii) (i) (iii)

(2) (iii) (i) (ii)

(3) (i) (iii) (ii)

8. Consider the following statements and mark the option which is correct.

(i) The Kuka revolt in Punjab was aimed to purge. Sikhism of its ills & discriminations by preaching Abolitions of castes etc.

(ii) One of the important leaders of Naga was Gadiniliu who fed the Heraka cult.

(iii) Gadkari Rebellion was the outcome of the assumption of direct administration of Kalhapur by British.

(a) i & iii (b) ii

(c) all of the above (d) none of the above

9. Consider the following statements and mark the option which is correct.

(i) All India Kissan Sabha’s Main demands included 50% cut in Revenue Revolts; abolition of Begar.

(ii) Champaran Satyagrah was the Movement of Indigo peasants against the oppression of the planters.

(iii) Eka movement was led by Bishnu Biswas. It was the revolt by indigo peasants.

(a) none of the above (b) i & ii

(c) iii (d) none of the above

10. Match the following

(i) Tehangana Movement (a) against the threat of Bedakhli eviction and the Jajmani System.

(ii) Moplah Uprising (b) Biggest peasants guerilla war in the Indian History.

Page 79: Study Kit Brochure for SSC Combined Graduate Level ...static.upscportal.com/files/study-kit/ssc-cgl/SSC-CGL-2013... · Brochure for SSC Combined Graduate Level Examination - 2014

Click Here for SSC CGL Online Coaching: http://sscportal.in/community/courses/ssc-cgl-tier-1

[Click Here to Buy This Study Kit in Hard Copy: http://sscportal.in/community/study-kit/cgl ] 78

(iii) Telangana Movement (c) arose against vastly enhanced Rights of Hindu Namboodr & Nair Jenmis.

(a) (b) (c)

(1) (iii) (i) (ii)

(2) (i) (ii) (iii)

(3) (ii) (iii) (i)

ANSWERS: 1. (1) 2. (a) 3. (c) 4. (2) 5. (b) 6. (1) 7. (3) 8. (c) 9. (b) 10. (1)

Section – 3 : Numerical Aptitude

Percentage

‘Per cent’ means ‘per hundred’. It is given by % symbol. Here x% means x per hundred or Thus, any percentage can be converted into an equivalent fraction by dividing it by 100.

eg 20% =

20 1 ;100 5

= 150% =

150 3100 2

=

Also, any fraction or decimal can be converted into its equivalent percentage by multiplying with 100.

eg

1 1 1005 5

= × = 20%;

3 3 1002 2

= ×= 150%.

Important Formulae

1. Percentage increase =

Increase 100Original value

×

2. Percentage decrease =

Decrease 100Original value

×

3. If the price of the commodity increases by r% then the reduction in consumption so as not to increase

the expenditure is 100 %

100r

r × +

4. If the price of the commodity decreases by r% then the reduction in consumption so as not to increase

the expenditure is 100 %

100r

r × −

5. If A’s income is r% more than B’s income then B’s income is less than A’s income by .

100 %100

rr

× + 6. If A’s income is r% less than B’s income then B’s income is more than A’s income by .

100 %100

rr

× −

Page 80: Study Kit Brochure for SSC Combined Graduate Level ...static.upscportal.com/files/study-kit/ssc-cgl/SSC-CGL-2013... · Brochure for SSC Combined Graduate Level Examination - 2014

Click Here for SSC CGL Online Coaching: http://sscportal.in/community/courses/ssc-cgl-tier-1

[Click Here to Buy This Study Kit in Hard Copy: http://sscportal.in/community/study-kit/cgl ] 79

7. Let the population of a town be P and it increases at the rate of r% per annum, then

(a) Population after n years = 1

100

nrP +

(b) Population n years ago = 1

100

nPr +

8. Let the present value of the machine be P and if it depreciates at the rate of r% per annum.

(a) Value of machine after n years = 1

100

nrP −

(b) Value of machine n years ago = 1

100

nn

rP −

Example 1: Express 3/2 as rate per cent.

Solution.

32 =

3 100 %2

× = 150%

Example 2: Find 25% of 1000.

Solution. 25% of 1000 =

25 1000100

× = 250

Example 3. What per cent of 6 is 144?

Solution. Required percentage =

144 100 %6

× = 2400%

Example 4: What per cent of 2.5 kg is 15 g?

Solution. Required percentage =

15 100 %2.5 1000

× × = 0.6% Example 5. If the price of tea falls by 12%, by how much pr cent must a house holder increase its

consumption, so as not to decrease its expenditure on tea? Solution. (Short cut method)

Increase % in consumption = 100 %

100r

r × − =

12 100 %100 12

× −

=

12 100 %88

× =

150 %11 =

713 %11

Example 6: The value of a machine depreciates at the rate of 10% per annum. If its present value is ` 162000, what was the value of the machine 2 year ago?

Solution. Value of machine 2 year ago = `

2162000.

101100

Rs

− = `

10 10. 1620009 9

Rs × ×

Page 81: Study Kit Brochure for SSC Combined Graduate Level ...static.upscportal.com/files/study-kit/ssc-cgl/SSC-CGL-2013... · Brochure for SSC Combined Graduate Level Examination - 2014

Click Here for SSC CGL Online Coaching: http://sscportal.in/community/courses/ssc-cgl-tier-1

[Click Here to Buy This Study Kit in Hard Copy: http://sscportal.in/community/study-kit/cgl ] 80

= ` 200000 Example 7: Due to a reduction of 5% in prices of sugar, a man is able to buy 1 kg more for ` 95. Find

the original and reduced rate of sugar. Solution. Let the original rate be ` x per kg.

Reduced rate = `

1(100 5)100

x− ×= `

95100

x ∴

95 95 195100

x x− =

5 1x

= ⇒ x = 5

∴ Original rate = ` 5 per kg

Reduced rate = `

19 5.20 1

Rs × per kg = `

19.4

Rs = 4.75 er kg

Example 8: If the price of 1 kg cornflakes is increased by 25%, the increase is ` 10. Find the new price of cornflakes per kg.

Solution. Original price =

Difference in price 100Difference in per cent

×=

10 10025

×= 400

∴ New price =

12540100

×= ` 50

EXERCISE: 1.The difference of two numbers is 20% of the larger number. If the smaller number is 20, then the larger

number is: (a) 25 (b) 46 (c) 27 (d) 82 2. When any number is divided by 12, then dividend becomes 1/4th of the other number. By how much

percent first number is greater than the second number? (a) 165 (b) 200 (c) 300 (d) 400 3. If one number is 80% of the other and 4 times the sum of their squares is 656, then the numbers are: (a) 6,8 (b) 8, 10 (c) 16, 20 (d) 10, 15 4. Two numbers A and B are such that the sum of 5% of A and 4% of B is two-third of the sum of 6% of A

and 8% of B. Find the ratio of A : B. (a) 1 : 2 (b) 3 : 1 (c) 3 : 4 (d) 4 : 3 5. Three candidates contested an election and received 1136, 7636 and 11628 votes respectively. What

percentage of the total votes did the winning candidate get? (a) 57% (b) 77% (c) 80% (d) 90% 6. The population of a town increased from 1,75,000 to 2,62,500 in a decade. The average percent

increase of population per year is: (a) 7% (b) 5% (c) 9% (d) 8.75% 7. A student multiplied a number by 3 instead of 5/3. What is the percentage error in the calculation? (a) 36% (b) 44% (c) 55% (d) 35% 8. A tempo is insured to the extent of 4 5 of its original value. If the premium on it at the rate of 1.3

percent amounts to ` 910, the original value of the tempo is: (a) ` 78,000 (b) ` 78,500

Page 82: Study Kit Brochure for SSC Combined Graduate Level ...static.upscportal.com/files/study-kit/ssc-cgl/SSC-CGL-2013... · Brochure for SSC Combined Graduate Level Examination - 2014

Click Here for SSC CGL Online Coaching: http://sscportal.in/community/courses/ssc-cgl-tier-1

[Click Here to Buy This Study Kit in Hard Copy: http://sscportal.in/community/study-kit/cgl ] 81

(c) ` 80,000 (d) ` 87,500 9. When 15% is lost in grinding wheat, a country can export 30 lakh tons of wheat. On the other hand, if

10% is lost in grinding, it can export 40 lakh tons of wheat. The production of wheat in the country is: (a) 40 lakh tons (b) 400 lakh tons (c) 200 lakh tons (d) 900 lakh tons 10. In a competitive examination in State A, 6% candidates got selected from the total appeared

candidates. State B had an equal number of candidates appeared and 7% candidates got selected with 80 more candidates got selected than A. What was the number of candidates appeared from each State?

(a) 7600 (b) 8000 (c) 8300 (d) 4000

ANSWERS: 1. (a) 2. (b) 3. (b) 4. (d) 5. (a) 6. (b) 7. (d) 8. (d) 9. (c) 10. (b)

EXPLAINATIONS: 1.Let the larger number be x.

Then, x – 20 =

20 1 20100 5

x x x⇔ − =

45

x= 20 ⇔ x =

5204

× = 25%

2. Let the numbers be x and y. Then,

3 .

12 4x y x y= ⇔ =

Required percentage =

100 %x y

y −

× =

2 100 %yy

×

= 200%

3. Let one number = x. Then, other number = 80% of x =

45

22 44

5x x

+ = 656

⇔ x = 2 216

25x x+

= 164

⇔ 241

25x

= 164

⇔ x2 =

164 2541×

= 100

⇔ x = 100 So, the numbers are 10 and 8. 4. 5% of A + 4% of B =(6% of A + 8% of B)

Page 83: Study Kit Brochure for SSC Combined Graduate Level ...static.upscportal.com/files/study-kit/ssc-cgl/SSC-CGL-2013... · Brochure for SSC Combined Graduate Level Examination - 2014

Click Here for SSC CGL Online Coaching: http://sscportal.in/community/courses/ssc-cgl-tier-1

[Click Here to Buy This Study Kit in Hard Copy: http://sscportal.in/community/study-kit/cgl ] 82

5 4 2 6 8100 100 3 100 100

A B A B ⇔ + = +

.

1 1 1 420 25 25 75

A B A B⇔ + = +

1 1 100 4100 75 75 3

AA BB

⇔ + ⇔ = =

5. Total number of votes polled = (1136 + 7636 + 11628) = 20400.

Required percentage =

11628 100 %20400

× = 57%.

6. Increase in 10 years = (262500 – 175000) = 87500.

Increases % =

87500 100 %175000

× = 50% Required average =

50 %10

= 5% 7. Let the number be x. Then, error = Error %

=

5 3 163 5 15

x x x− = = Error %

=

16 3 100 %15 5

xx

× × = 64%

8. Let the original value of the tempo be ` x. Then,

1.3%

45 of x= 910 ⇔

13 1 410 100 5

x× × ×= 910

⇔ x =

910 10 100 513 4

× × × × = 87500.

9. Let the total production be x lakh tons. Then, 15% of x – 10% of x = (40 – 30) lakh tons ⇔ 5% of x = 10 lakh tons

⇔ x =

10 1005

×

= 200 lakh tons. 10. Let the number of candidates appeared from each state be x. Then, 7% of x – 6% of x = 80 ⇔ 1% of x = 80 ⇔ x = 80 × 100 = 8000.

Profit & Loss

Cost Price The price at which an article is purchased is called the cost price or CP.

Page 84: Study Kit Brochure for SSC Combined Graduate Level ...static.upscportal.com/files/study-kit/ssc-cgl/SSC-CGL-2013... · Brochure for SSC Combined Graduate Level Examination - 2014

Click Here for SSC CGL Online Coaching: http://sscportal.in/community/courses/ssc-cgl-tier-1

[Click Here to Buy This Study Kit in Hard Copy: http://sscportal.in/community/study-kit/cgl ] 83

Selling Price The price at which an article is sold is called the selling price or SP.

Formulae Gain or Profit = SP – CP

Gain per cent or Profit per cent =

Gain Profit100 or 100CP CP

× ×

SP =

100 Profit% CP100

+ ×

Similarly, Loss = CP – SP

Loss per cent =

Loss 100CP

× ; SP =

(100 Loss%) CP100−

×

• The Profit and Loss per cent is always calculated on the cost price. • If a trader professes to sell his goods at CP but uses false weight, then Gain per cent or Profit per cent

=

Error 100 %TrueValue Error

× −

Marked Price or List Price Price that is indicated or marked on the article is called marked price or MP.

Discount It is reduction given on the Marked Price or List Price of an article.

d per cent =

100 discount ;MP

×

Selling Price =

(100 %)100

d MP−×

If a trade gets x% profit and x% loss in selling two different articles, then in over all transaction, there

is always a loss which is given by

Loss % =

2

10x

Example 1: A chair is bought for ` 1950 and sold at ` 2340. Find the gain per cent. Solution. CP = ` 1950 and SP = ` 2340 Gain = ` (2340 – 1950) = ` 390

Gain % =

390 100 %1950

× = 20%

Example 2: A radio is bought for ` 780 and sold at ` 650. Find the loss per cent. Solution. CP = ` 780 and Sp = ` 650 Loss = CP – SP = ` (780 – 650) = ` 130

Loss % =

13 100780

× =

Example 3: A book is bought for ` 80 and sold at the gain of 5%. Find the selling price.

Page 85: Study Kit Brochure for SSC Combined Graduate Level ...static.upscportal.com/files/study-kit/ssc-cgl/SSC-CGL-2013... · Brochure for SSC Combined Graduate Level Examination - 2014

Click Here for SSC CGL Online Coaching: http://sscportal.in/community/courses/ssc-cgl-tier-1

[Click Here to Buy This Study Kit in Hard Copy: http://sscportal.in/community/study-kit/cgl ] 84

Solution. CP = ` 80, Gain = 5%

SP = 105% of ` 80 = `

105 80100

× = ` 84

Example 4: If cost price of 15 articles is equal to the selling price of 12 articles, then find the gain per cent.

Solution. Let cost price of each article = ` 1 Then, Cost price of 15 articles = ` 15 ∴ Selling price of 12 articles = ` 15 But Cost price of 12 articles = ` 12 ∴ Profit = ` (15 – 12) = ` 3

∴ Profit % =

3 10012

× = 25%

Example 5: What is the equivalent discount of three consecutive discount 30%, 20% and 5%. Solution. Let MP = ` 100

∴ SP = 95% of 80% of 70% of 100 =

95 80 70 100100 100 100

× × × = ` 53.20

∴ Required equivalent discount = ` (100 – 53.20) = ` 46.80 Example 6: By selling 66 m of cloth a person gains the cost price of 22 m. Find the gain per cent. Solution. Let CP of 1 m cloth = ` 1

Then, Gain % =

gain 100CP

×=

CP of 22 m cloth 100CP of 66 m cloth

× =

22 10066

×=

133 %3 Example 7: A radio is listed at ` 500 with a discount of 10%. What additional discount must be offered

to the customer to bring the net price to ` 423? Solution. List price = ` 500, Discount = 10%

SP = 90% of ` 500 = `

90 500100

× = ` 450

Sale price = ` 423

Additional discount =

27 100450

× = 6%

EXERCISE: 1.A man purchased a box full of pencils at the rate of 7 for ` 9 and sold all of them at the rate of 8 for ` 11.

In this transaction, he gained ` 10. How many pencils did the box contain? (a) 111 (b) 112 (c) 114 (d) 116 2. A man bought a number of clips at 3 for a rupee and an equal number at 2 for a rupee. At what price

per dozen should he sell them to make a profit of 20%? (a) ` 9 (b) ` 10 (c) ` 6 (d) ` 7 3. A man buys eggs at 2 for Re. 1 and an equal number at 3 for ` 2 and sells the whole at 5 for ` 3. His

gain or loss percent is:

Page 86: Study Kit Brochure for SSC Combined Graduate Level ...static.upscportal.com/files/study-kit/ssc-cgl/SSC-CGL-2013... · Brochure for SSC Combined Graduate Level Examination - 2014

Click Here for SSC CGL Online Coaching: http://sscportal.in/community/courses/ssc-cgl-tier-1

[Click Here to Buy This Study Kit in Hard Copy: http://sscportal.in/community/study-kit/cgl ] 85

(a) % loss (b) % gain (c) % loss (d) % loss 4. A man bought some oranges at ` 10 per dozen and bought the same number of oranges at ` 8 per

dozen. He sold these oranges at ` 11 per dozen and gained `120. The total number of oranges bought by him was:

(a) 55 dozens (b) 80 dozens (c) 90 dozens (d) 60 dozens 5. A vendor bought toffees at 6 for a rupee. How many for a rupee must he sell to gain 20%? (a) 8 (b) 9 (c) 5 (d) 2 6. By selling 12 toffees for a rupee, a man loses 20%. How many for a rupee should he sell to get a gain

of 20%? (a) 6 (b) 8 (c) 10 (d) 9 7. By selling 45 lemons for ` 40, a man loses 20%. How many should he sell for ` 24 to gain 20% in the

transaction? (a) 19 (b) 18 (c) 24 (d) 22 8. A trader mixes 26 kg of rice at `20 per kg with 30 kg of rice of other variety at `36 per kg and sells the

mixture at `30 per kg. His profit percent is: (a) 6% (b) 5% (c) 11% (d) 10% 9. Arun purchased 30 kg of wheat at the rate of ` 11.50 per kg and 20 kg of wheat at the rate of ` 14.25

per kg. He mixed the two and sold the mixture. Approximately what price per kg should he sell the mixture to make 30% profit?

(a) ` 15.80 (b) ` 16.40 (c) ` 15.60 (d) ` 16.30 10. Padam purchased 30 kg of rice at the rate of `17.50 per kg and another 30 kg rice at a certain rate.

He mixed the two and sold the entire quantity at the rate of `18.60 per kg and made 20% overall profit. At what price per kg did he purchase the lot of another 30 kg rice?

(a) ` 14.50 (b) ` 13.50 (c) ` 17.50 (d) ` 16.50

ANSWERS: 1. (b) 2. (c) 3. (d) 4. (d) 5. (c) 6. (b) 7. (b) 8. (b) 9. (d) 10. (b)

EXPLAINATONS: 1.Suppose, number of pencils bought = L.C.M. of 7 and 8 = 56.

C.P. of 56 pencils = `

9 567

× = ` 72.

S.P. of 56 pencils = `

11 568

× = ` 77.

Now, ` 5 are gained on 56 pencils.

So, ` 10 are gained on

56 105

× = 112 pencils.

Page 87: Study Kit Brochure for SSC Combined Graduate Level ...static.upscportal.com/files/study-kit/ssc-cgl/SSC-CGL-2013... · Brochure for SSC Combined Graduate Level Examination - 2014

Click Here for SSC CGL Online Coaching: http://sscportal.in/community/courses/ssc-cgl-tier-1

[Click Here to Buy This Study Kit in Hard Copy: http://sscportal.in/community/study-kit/cgl ] 86

2. Suppose he bought 1 dozen clips of each kind.

C.P. of 2 dozens = `

1 112 123 2

× + ×

= ` 10 ∴ S.P. of 2 dozen = 120% of ` 10

= ` = `

120 10100

× 12

Hence, S.P. per dozen = ` 6. 3. Suppose he buy 6 eggs of each kind.

C.P. of 12 eggs = `

1 26 62 3

× + × = ` 7.

S.P. of 12 eggs = `

3 125

× = ` 7.20. ∴ Gain =

0.20 100 %7

× =

4. C.P. of 2 dozen oranges = ` (10 + 8) = ` 18. S.P. of 2 dozen oranges = ` 22. If profit is ` 4, orange bought = 2 dozen. If profit is ` 120, oranges bought

=

2 1204

× dozens = 60 dozens.

5. C.P. of 6 toffees = Re. 1 S.P. of 6 toffees = 120% of Re. 1 = `

65 .

For `

65 , toffees sold = 6. For Re. 1, toffees sold =

665

× = 5.

6. Let S.P. of 12 toffees be ` x. Then, 80 : 1 = 120 : x or x =

120 380 2

= .

For `

32 , toffees sold = 12.

For ` 1, toffees sold =

2128

× = 8.

7. Let S.P. of 45 lemons be ` x. Then, 80 : 40 = 120 : x

or x =

120 4080×

= 60.

For ` 60, lemons sold = 45. For ` 24, lemons sold =

45 2460

× = 18.

8. C.P. of 56 kg rice = ` (26 × 20 + 30 × 36) = ` (520 + 1080) = ` 1600. S.P. of 56 kg rice = ` (56 × 30) = ` 1680.

Page 88: Study Kit Brochure for SSC Combined Graduate Level ...static.upscportal.com/files/study-kit/ssc-cgl/SSC-CGL-2013... · Brochure for SSC Combined Graduate Level Examination - 2014

Click Here for SSC CGL Online Coaching: http://sscportal.in/community/courses/ssc-cgl-tier-1

[Click Here to Buy This Study Kit in Hard Copy: http://sscportal.in/community/study-kit/cgl ] 87

∴ Gain =

80 1001600

× % = 5%.

9. C.P. of 50 kg wheat = ` (30 × 11.50 + 20 × 12.25) = ` (345 + 285) = ` 630. S.P. of 50 kg wheat = 130% of ` 630

= ` = `

130 630100

× 819.

∴ S.P. per kg = `

81950

= ` 16.38 ≈ ` 16.30. 10. Let the required price per kg be ` x. Then, C.P. of 60 kg rice = ` (30 × 17.50 + 30 × x) = ` (525 + 30x) =

` (525 + 30x). S.P. of 60 kg rice = ` (60 × 18.60) = ` 1116

1116 (525 30 ) 100525 30

xx

− +×

+ = 20

519 30525 30

xx

−+ =

15 .

⇔ 2955 – 150x = 525 + 30x ⇔ 180x = 2430

⇔ x =

2430180

=

272

= 13.50. So, the C.P. of second lot is ` 13.50 per kg.

Section – 4 : English language & Comprehension

Comprehension

Directions (Q.1–11): Read the following passage carefully and answer the questions given below it. Certain words are printed in bold in the passage to help you locate them while answering some of the questions.

What is immediately needed today is the establishement of a Wrold Government or an International Federation of mankind. It is the utmost necessity of the world today, and all those persons who wish to see all human beings happy and prosperous naturally feel it keenly.

Of course, at times we feel that many of the problem of our political, social, linguistic and cultural life would come to an end if there were one Govenment all ovet the world. Travellers, businessmen, seekers of knowledge and teachers of righteousness know very well that great impediments and obstructions are faced by them when they pass from one country to another, exchange goods, get information, and make an efforts

Page 89: Study Kit Brochure for SSC Combined Graduate Level ...static.upscportal.com/files/study-kit/ssc-cgl/SSC-CGL-2013... · Brochure for SSC Combined Graduate Level Examination - 2014

Click Here for SSC CGL Online Coaching: http://sscportal.in/community/courses/ssc-cgl-tier-1

[Click Here to Buy This Study Kit in Hard Copy: http://sscportal.in/community/study-kit/cgl ] 88

to spread their good gospel among their fellow-men. In the past, religious sects divided one set of people against another, colour of skin or shape of the body set one against the other.

But today when philosophical light has exploded the darkness that was created by religious differences, and when scientific knowledge has flasified the superstitions, they have enabled human beings of all religious views and of all races and colours to come in frequent contact with one another . It is the governments of various countries that keep poeple of one country apart from, those of another. They create artificial barriers, unnatural distinctions, unhealthy isolation, unnecessary fears and dangers in the minds of common men who by their nature want to live in friendship with their fellow-men. But all these evils would cease to exist if there were one Government all over the world. 1. What divides people of a country against another? (a) Different religions (b) Different language (c) Different social and political systems of different people (d) Government of various countries 2. What is the urgent heed of the world today? (a) The establishment of an international economic order. (b) The establishment of a world government. (c) The creation of a cultural international social order. (d) The raising of an international spiritual army. 3. What will the world Government be expected to do? (a) it will arrange for interplanetary contacts (b) it will end all wars for all time to come (c) it will bring about a moral regeneration of mankind (d) it will kill the evil spirit in man 4. Choose the word which is SIMILAR in meaning as the word "righteousness" as used in the passage. (a) rectitude (b) religiosity (c) requirement (d) scrupulousness 5. Which of the following problems has not been mentioned in the passage as likely to be solved with the

establishment of world Government? (a) Social Problems (b) Political Problems (c) Cultural Problems (d) Economic Problems 6. Choose the word which is most OPPOSITE in meaning of the word 'implediments' as used in the

passage. (a) handicaps (b) furtherance (c) providence (d) hindrances 7. The most appropriate title of the above passage may be ........... (a) The evils of the world order. (b) The man can make his destiny. (c) The need of world Government. (d) The role of Religion in the Modern Times. 8. What was the factor, according to the passage, that set one man against another? (a) Material prosperity of certain people in the midst of grinding poverty. (b) Superior physical strength of some persons. (c) Colour of skin or shape of the body. (d) Some people being educated and other illiterate. 9. The theory of racial superiority stands falsified today by ........ (a) knowledge derived from scientific advances. (b) the ascendancy of people who were here to fore considered of inferior racial stock. (c) the achievements of the so called backward countries in every field of life. (d) the precedence given to the physical powers of different races. 10. In the part religious sects ............. (a) united the people with one another.

Page 90: Study Kit Brochure for SSC Combined Graduate Level ...static.upscportal.com/files/study-kit/ssc-cgl/SSC-CGL-2013... · Brochure for SSC Combined Graduate Level Examination - 2014

Click Here for SSC CGL Online Coaching: http://sscportal.in/community/courses/ssc-cgl-tier-1

[Click Here to Buy This Study Kit in Hard Copy: http://sscportal.in/community/study-kit/cgl ] 89

(b) Interfered in political affairs. (c) did a good job by way of spreading message of love and peace. (d) divided one set of people from another. Directions (Q. 12-18): Read the following passage carefully and answer the question given below it. Certain words are printed in bold to help you to locate them while answering some of the questions.

The window offered a view of the house opposite. The two families did not speak to each other because of a property dispute. One day, Ruchira's textbooks lay untouched as the young girl's gaze was on the happenings in the house opposite. There were two new faces in the neighbouring household – that of an elderly widow and a girls aged sixteen. Some times the elderly lady would sit by the window, doing the young girl's hair. On other days she was absent.

The new young neighbour's daily routine could be seen through the window – she cleaned the rice paddy; split nuts, put the cushions in the sun to air them. In the afternoons while the men were all at world some of the women slept and others played cards. The girl sat on the terrace and read. Sometimes she wrote. One day there was hindrance. She was writing when the elderly woman snatched the unfinished letter from her hands. Thereafter the girl was not to be seen on the terrace. Sometimes during the day sounds came from the house indicating that a massive argument was going on inside.

A few days passed. One evening Ruchira noticed the girl standing on the terrace in tears. The evening prayer was in progress. As she did daily, the girl bowed several times in prayer. Then she went downstairs. That night Ruchira wrote a letter. She went out and posted it that very instant. But as she lay in bed that night, she prayed fervently that her offer of friendship wouldn't reach its destination. Ruchira then left for Madhupur and returned when it was time for college to start. She found the house opposite in darkness, locked. They had left.

When she stepped into her room she found the desk piled with letters – one had a local stamp on it with her name and address in unfamiliar handwriting. She quickly read it. They continued to write to each other for the next twenty years. 11. Why did Ruchira write a letter to her new neighbour? (a) She wanted to offer her, her help. (b) She wanted to be friends with her. (c) To apologize for her family's behaviour towards her family. (d) To encourage her to continue learning to read and write. 12. Which of the following can be said about Ruchira? A. She used to spy on her neighbours because she didn't trust them. B. She was at home because she was studying. C. She did not speak to her neighbours because they did not own property. (a) None (b) Only B (c) Both A & B (d) Only C 13. How did the new young neighbour spend her days? (a) She was busy writing letters to Ruchira. (b) She used to daydream about her past experiences. (c) She would attend to the needs of the widow. (d) She spent her time learning to read and write. 14. Why was the young neighbour prevented from sitting on the terrace? (a) She used to while away her time instead of working (b) The old woman could no longer keep an eye on her. (c) She had not finished writing the letter she was asked to. (d) She had been writing a letter which she wasn't supposed to. 15. What was the major argument in the house about? (a) There were too many people living there, which resulted in arguments. (b) The young girl was insisting on attending college. (c) The young girl had been wasting her time instead of working. (d) The old woman did not guard the young girl closely.

Page 91: Study Kit Brochure for SSC Combined Graduate Level ...static.upscportal.com/files/study-kit/ssc-cgl/SSC-CGL-2013... · Brochure for SSC Combined Graduate Level Examination - 2014

Click Here for SSC CGL Online Coaching: http://sscportal.in/community/courses/ssc-cgl-tier-1

[Click Here to Buy This Study Kit in Hard Copy: http://sscportal.in/community/study-kit/cgl ] 90

16. Which of the following is TRUE in the context of the passage? (a) The young girl was very devout and prayed everyday. (b) Only two letters were exchanged between the two girls. (c) The new young neighbour was a servant. (d) The afternoon was a time to relax for everyone. 17. Why did the young girl wish that the letter would not reach its destination? A. She was going away and would not be able to see if her neighbour was glad to receive it. B. She was afraid that it would lead to a quarrel between the two families. C. She was afraid that her neighbour would be angry when she received her letter. (a) None (b) Only A (c) Only C (d) Both B & C Directions (Q.18-20): Choose the word which is most nearly the SAME in meaning as the world printed in bold as used in the passage. 18. Hindrance (a) handicapped (b) delay (c) interruption (d) difficult 19. Offered (a) forward (b) willing (c) volunteered (d) provided 20. Choose the word which is most OPPOSITE in meaning of the word piled as used in the passage. (a) low (b) empty (c) blank (d) nothing

ANSWERS: 1. (d) 2. (d) 3. (c) 4. (a) 5. (d) 6. (b) 7. (c) 8. (c) 9. (a) 10. (d) 11. (b) 12. (b) 13. (d) 14. (d) 15. (c) 16. (a) 17. (b) 18. (c) 19. (d) 20. (b)

Idioms and Phrases

A dog in the manager: A person who prevents others from enjoying something that is useless to him. A sitting duck: A person or thing that is an easy target to attack. A Bird in the Hand is Worth Two in the Bush: Having something that is certain is much better than

taking a risk for more, because chances are you might lose everything. A Blessing in Disguise: Something good that isn’t recognized at first. A Chip On Your Shoulder: Being upset for something that happened in the past. A Dime a Dozen: Anything that is common and easy to get. A Doubting Thomas: A skeptic who needs physical or personal evidence in order to believe

something. A Drop in the Bucket: A very small part of something big or whole. A Fool and his Money are Easily Parted: It’s easy for a foolish person to lose his/her money. A House Divided against itself cannot Stand: Everyone involved must unify and function together or

it will not work out. A Leopard Can’t Change his Spots: You cannot change who you are. A Penny Saved is a Penny Earned: By not spending money, you are saving money (little by little). A Picture Paints a Thousand Words: A visual presentation is far more descriptive than words.

Page 92: Study Kit Brochure for SSC Combined Graduate Level ...static.upscportal.com/files/study-kit/ssc-cgl/SSC-CGL-2013... · Brochure for SSC Combined Graduate Level Examination - 2014

Click Here for SSC CGL Online Coaching: http://sscportal.in/community/courses/ssc-cgl-tier-1

[Click Here to Buy This Study Kit in Hard Copy: http://sscportal.in/community/study-kit/cgl ] 91

A Piece of Cake: A task that can be accomplished very easily. A Slap on the Wrist: A very mild punishment. A Taste of Your Own Medicine: When you are mistreated the same way you mistreat others. A Toss-Up: A result that is still unclear and can go either way. Actions Speak Louder Than Words: It’s better to actually do something than just talk about it. Add Fuel to the Fire: Whenever something is done to make a bad situation even worse than it is. Against the Clock: Rushed and short on time. All Bark and no Bite: When someone is threatening and/or aggressive but not willing to engage in a

fight. All Greek to me: Meaningless and incomprehensible like someone who cannot read, speak, or

understand any of the Greek language would be. All in the Same Boat: When everyone is facing the same challenges. An Arm and a Leg: Very expensive. A large amount of money. An Axe to Grind: To have a dispute with someone. Apple of My Eye: Someone who is cherished above all others. As High as a Kite: Anything that is high up in the sky. At the Drop of a Hat: Willing to do something immediately. Black and Blue: To beat mercilessly. Back Seat Driver: People who criticize from the sidelines, much like someone giving unwanted advice

from the back seat of a vehicle to the driver. Back to Square One: Having to start all over again. Back to the Drawing Board: When an attempt fails and it’s time to start all over. Baker’s Dozen: Thirteen. Barking up the Wrong Tree: A mistake made in something you are trying to achieve. Beat a Dead Horse: To force an issue that has already ended. Beating around the Bush: Avoiding the main topic. Not speaking directly about the issue. Bend Over Backwards: Do whatever it takes to help. Willing to do anything. Between a Rock and a Hard Place: Stuck between two very bad options. Bite Off More Than You Can Chew: To take on a task that is way to big. Bite Your Tongue: To avoid talking. Blood is Thicker Than Water: The family bond is closer than anything else. Blue Moon: A rare event or occurrence. Break a Leg: A superstitious way to say ‘good luck’ without saying ‘good luck’, but rather the opposite. Buy a Lemon: To purchase a vehicle that constantly gives problems or stops runnings after you drive it

away. Close shave: A narrow escape. Can’t Cut the Mustard: Someone who isn’t adequate enough to compete or participate. Cast Iron Stomach: Someone who has no problems, complications or ill effects with eating anything or

drinking anything. Charley Horse: Stiffness in the leg/A leg cramp. Chew someone out: Verbally scold someone.

Page 93: Study Kit Brochure for SSC Combined Graduate Level ...static.upscportal.com/files/study-kit/ssc-cgl/SSC-CGL-2013... · Brochure for SSC Combined Graduate Level Examination - 2014

Click Here for SSC CGL Online Coaching: http://sscportal.in/community/courses/ssc-cgl-tier-1

[Click Here to Buy This Study Kit in Hard Copy: http://sscportal.in/community/study-kit/cgl ] 92

Chip on his Shoulder: Angry today about something that occurred in the past. Chow Down: To eat. Close but no Cigar: To be very near and almost accomplish a goal, but fall short. Cock and Bull Story: An unbelievable tale. Come Hell or High Water: Any difficult situation or obstacle. Crack Someone Up: To make someone laugh. Cross Your Fingers: To hope that something happens the way you want it to. Cry Over Spilt Milk: When you complain about a loss from the past. Cry Wolf: Intentionally raise a false alarm. Cup of Joe: A cup of coffee. Curiosity Killed the Cat: Being Inquisitive can lead you into a dangerous situation. Cut to the Chase: Leave out all the unnecessary details and just get to the point. Dark Horse: One who was previously unknown and is now prominent. Dead Ringer: 100% identical. A duplicate. Devil’s Advocate: Someone who takes a position for the sake of argument without believing in that

particular side of the argument. It can also mean one who presents a counter argument for a position they do believe in, to another debater.

Dog Days of Summer: The hottest days of the summer season. Don’t count your chickens before they hatch: Don’t rely on it until your sure of it. Don’t Look a Gift Horse in the Mouth: When someone gives you a gift, don’t be ungrateful. Don’t Put all Your Eggs in One Basket: Do not put all your resources in one possibility. Doozy: Something outstanding. Down to the Wire: Something that ends at the last minute or last few seconds. Drastic Times Call for Drastic Measures: When your are extremely desperate you need to take

extremely desperate actions. Drink like a fish: To drink very heavily. Drive someone up the wall: To irritate and/or annoy very much. Dropping Like Flies: A large number of people either falling ill or dying. Dry Run: Rehearsal. Eighty Six: A certain item is no longer available. Or this idiom can also mean, to throw away. Elvis has left the building: The show has come to an end. It’s all over. Ethnic Cleansing: Killing of a certain ethnic or religious group on a massive scale. Every Cloud Has a Silver Lining: Be optimistic, even difficult times will lead to better days. Everything But The Kitchen Sink: Almost everything and anything has been included. Excuse my French: Please forgive me for cussing. French Leave: Absence without permission. Feeding Frenzy: An aggressive attack on someone by a group. Field Day: An enjoyable day or circumstance. Finding Your Feet: To become more comfortable in whatever you are doing. Finger lickin’ good: A very tasty food or meal. Fixed in Your Ways: Not willing or wanting to change from your normal way of doing something.

Page 94: Study Kit Brochure for SSC Combined Graduate Level ...static.upscportal.com/files/study-kit/ssc-cgl/SSC-CGL-2013... · Brochure for SSC Combined Graduate Level Examination - 2014

Click Here for SSC CGL Online Coaching: http://sscportal.in/community/courses/ssc-cgl-tier-1

[Click Here to Buy This Study Kit in Hard Copy: http://sscportal.in/community/study-kit/cgl ] 93

Flash in the Pan: Something that shows potential or looks promising in the beginning but fails to deliver anything in the end.

Flea Market: A swap meet. A place where people gather to buy and sell inexpensive goods. Flesh and Blood: This idiom can mean living material of which people are made of, or it can refer to

someone’s family. Flip the Bird: To raise your middle finger at someone. Foam at the Mouth: To be enraged and show it. Fools’ Gold: Iron pyrites, a worthless rock that resembles real gold. French Kiss: An open mouth kiss where tongues touch. From Rags to Riches: To go from being very poor to being very wealthy. Fuddy-duddy: An old-fashioned and foolish type of person. Full Monty: This idiom can mean either, “the whole thing” or “completely nude.” Funny Farm: A mental institutional facility. Get Down to Brass Tacks: To become serious about something. Get Over it: To move beyond something that is bothering you. Get Up on the Wrong Side of the Bed: Someone who is having a horrible day. Get Your Walking Papers: Get fired from a job. Give him the Slip: To get away from. To escape. Go Down Like a Lead Ballon: To be received badly by an audience. Go for Broke: To gamble everything you have. Go Out on a Limb: Put yourself in a tough position in order to support someone/something. Go the Extra Mile: Going above and beyond whatever is required for the task at hand. Good Samaritan: Someone who helps others when they are in need, with no discussion for

compensation, and no thought of a reward. Graveyard Shift: Working hours from about 12:00 am to 8:00 am. The time of the day when most other

people are sleeping. Great Minds Think Alike: Intelligent people think like each other. Green Room: The waiting room, especially for those who are about to go on a tv or radio show. Gut Feeling: A personal intuition you get, especially when feel something may not be right. Haste Makes Waste: Quickly doing things results in a poor ending. Hat Trick: When one player scores three goals in the same hockey game. This idiom can also mean three

scores in any other sport, such as 3 homeruns, 3 touchdowns, 3 soccer goals, etc. Have an Axe to Grind: To have a dispute with someone. He Lost His Head: Angry and overcome by emotions. Head Over Heels: Very excited and/or joyful, especially when in love. Hell in a Handbasket: Deteriorating and headed for complete disaster. High Five: Slapping palms above each others heads as celebration gesture. High on the Hog: Living in Luxury. Hit the Books: To study, especially for a test or exam. Hit the Hay: Go to bed or go to sleep. Hit the Nail on the Head: Do something exactly right or say something exactly right.

Page 95: Study Kit Brochure for SSC Combined Graduate Level ...static.upscportal.com/files/study-kit/ssc-cgl/SSC-CGL-2013... · Brochure for SSC Combined Graduate Level Examination - 2014

Click Here for SSC CGL Online Coaching: http://sscportal.in/community/courses/ssc-cgl-tier-1

[Click Here to Buy This Study Kit in Hard Copy: http://sscportal.in/community/study-kit/cgl ] 94

Hit the Sack: Go to bed or go to sleep. Hocus Pocus: In general, a term used in magic or trickery. Hold Your Horses: Be patient. Icing on the Cake: When you already have it good and get something on top of what you already have. Idle Hands are the Devil’s Tools: You are more likely to get in trouble if you have nothing to do. If it’s not one Thing, it’s another: When one thing goes wrong, then another, and another... In Like Flynn: To be easily successful, especially when sexual or romantic. In the Bag: To have something secured. In the Buff: Nude. In the Heat of the Moment: Overwhelmed by what is happening in the moment. In Your Face: An aggressive and bold confrontation. It Takes Two to Tango: A two person conflict where both people are at fault. It’s a Small World: You frequently see the same people in different places. Its Anyone’s Call: A competition where the outcome is difficult to judge or predict. Ivy League: Since 1954 the Ivy League has been the following universities: Columbia, Brown, Cornell,

Dartmouth, Yale, Pennsylvania, Princeton, and Harvard. Jaywalk: Crossing the street (from the middle) without using the crosswalk. Joshing Me: Tricking me. Keep an Eye on Him: You should carefully watch him. Keep body and soul together: To earn a sufficient amount of money in order to keep yourself alive. Keep your chin up: To remain joyful in a tough situation. Kick the Bucket: Die. Kitty-corner: Diagonally across. Sometimes called Catty-Corner as well. Knee Jerk Reaction: A quick and automatic response. Knock on Wood: Knuckle tapping on wood in order to avoid some bad luck. Know the Ropes: To understand the details. Last but not least: An introduction phrase to let the audience know that the last person mentioned is no

less important than those introduced before him/her. Lend Me Your Ear: To politely ask for someone’s full attention. Let Bygones be Bygones: To forget about a disagreement or argument. Let Sleeping Dogs Lie: To avoid restarting a conflict. Let the Cat out of the Bag: To share a secret that wasn’t suppose to be shared. Level playing field: A fair competition where no side has an advantage. Like a chicken with its head cut off: To act in a frenzied manner. Liquor someone up: To get someone drunk. Long in the Tooth: Old people (or horses). Loose Cannon: Someone who is unpredictable and can cause damage if not kept in check. Make no Bones About: To state a fact so there are no doubts or objections. Method to My Madness: Strange or crazy actions that appear meaningless but in the end are done for a

good reason.

Page 96: Study Kit Brochure for SSC Combined Graduate Level ...static.upscportal.com/files/study-kit/ssc-cgl/SSC-CGL-2013... · Brochure for SSC Combined Graduate Level Examination - 2014

Click Here for SSC CGL Online Coaching: http://sscportal.in/community/courses/ssc-cgl-tier-1

[Click Here to Buy This Study Kit in Hard Copy: http://sscportal.in/community/study-kit/cgl ] 95

Mumbo Jumbo: Nonsense or meaningless speech. Mum’s the word: To keep quiet. To say nothing. Nest Egg: Savings set aside for future use. Never Bite the Hand that Feeds You: Don’t hurt anyone that helps you. New kid on the block: Someone new to the group or area. New York Minute: A minute that seems to go by quickly, especially in a fast paced environment. No Dice: To not agree. To not accept a proposition. No Room to Swing a Cat: An unusually small or confined space. Not Playing with a Full Deck: Someone who lacks intelligence. Off on the Wrong Foot: Getting a bad start on a relationship or task. Off the Hook: No longer have to deal with a tough situation. Off the Record: Something said in confidence that the one speaking doesn’t want attributed to him/her. On Pins and Needles: Anxious or nervous, especially in anticipation of something. On the Fence: Undecided. On the Same Page: When multiple people all agree on the same thing. Out of the Blue: Something that suddenly and unexpectedly occurs. Out on a Limb: When someone puts themself in a risky situation. Out on the Town: To enjoy yourself by going out. Over my Dead Body: When you absolutely will not allow something to happen. Over the Top: Very excessive. Pass the Buck: Avoid responsibility by giving it to someone else. Pedal to the metal: To go full speed, especially while driving a vehicle. Peeping Tom: Someone who observes people in the nude or sexually active people, mainly for his own

gratification. Pick up your ears: To listen very carefully. Pig in a Poke: A deal that is made without first examining it. Pig out: To eat alot and eat it quickly. Pipe Down: To shut-up or be quiet. Practice Makes Perfect: By constantly practicing, you will become better. Pull the plug: To stop something. To bring something to an end. Pulling Your Leg: Tricking someone as a joke. Put a sock in it: To tell noisy person or a group to be quiet. Queer the pitch: Destroy or ruin a plan. Raincheck: An offer or deal that is declined right now but willing to accept later. Raining Cats and Dogs: A very loud and noisy rain storm. Ring Fencing: Separated usual judgement to guarantee protection, especially project funds. Rise and Shine: Time to get out of bed and get ready for work/school. Rome was not Built in one Day: If you want something to be completely properly, then its going to take

time. Rule of Thumb: A rough estimate.

Page 97: Study Kit Brochure for SSC Combined Graduate Level ...static.upscportal.com/files/study-kit/ssc-cgl/SSC-CGL-2013... · Brochure for SSC Combined Graduate Level Examination - 2014

Click Here for SSC CGL Online Coaching: http://sscportal.in/community/courses/ssc-cgl-tier-1

[Click Here to Buy This Study Kit in Hard Copy: http://sscportal.in/community/study-kit/cgl ] 96

Run out of steam: To be completely out of energy. Saved by the Bell: Saved at the last possible moment. Scapegoat: Someone else who takes the blame. Scot-free: To escape and not have to pay. Sick as a Dog: To be very sick (with the flu or a cold). Sitting Shotgun: Riding in the front passenger seat of a car. Sixth Sense: A paranormal sense that allows you to communicate with the dead. Skid Row: The rundown area of a city where the homeless and drug users live. Smell a Rat: To detect someone in the group is betraying the others. Smell Something Fishy: Detecting that something isn’t right and there might be a reason for it. Son of a Gun: A scamp. Southpaw: Someone who is left-handed. Spitting Image: The exact likeness or kind. Start from Scratch: To do it all over again from the beginning. The Ball is in Your Court: It is your decision this time. The Best of Both Worlds: There are two choices and you have them both. The Bigger they are the Harder they Fall: While the bigger and stronger opponent might be alot more

difficult to beat, when you do they suffer a much bigger loss. The Last Straw: When one small burden after another creates an unbearable situation, the last straw is

the last small burden that one can take. The Whole Nine Yards: Everything. All of it. Third times a charm: After no success the first two times, the third try is a lucky one. Tie the knot: To get married. Till the cows come home: A long time. To Make a Long Story Short: Something someone would say during a long and boring story in order to

keep his/her audience from losing attention. Usually the story isn’t shortened. To Steal Someone’s Thunder: To take the credit for something someone else did. Tongue and Cheek: Humour, not to be taken serious. Turn a Blind Eye: Refuse to acknowledge something you know is real or legit. Twenty three skidoo: To be turned away. Under the weather: Feeling ill or sick. Up a blind alley: Going down a course of action that leads to a bad outcome. Use Your Loaf: Use your head. Think smart. Van Gogh’s ear for music: Tone deaf. Variety is the Spice of Life: The more experiences you try the more exciting life can be. Wag the Dog: A diversion away from something of greater importance. Water Under the Bridge: Anything from the past that isn’t significant or important anymore. Wear Your Heart on Your Sleeve: To openly and freely express your emotions. When it Rains, it Pours: Since it rarely rains, when it does it will be a huge storm. When Pigs Fly: Something that will never ever happen.

Page 98: Study Kit Brochure for SSC Combined Graduate Level ...static.upscportal.com/files/study-kit/ssc-cgl/SSC-CGL-2013... · Brochure for SSC Combined Graduate Level Examination - 2014

Click Here for SSC CGL Online Coaching: http://sscportal.in/community/courses/ssc-cgl-tier-1

[Click Here to Buy This Study Kit in Hard Copy: http://sscportal.in/community/study-kit/cgl ] 97

Wild and Woolly: Uncultured and without laws. Wine and Dine: When somebody is treated to an expensive meal. Without a Doubt: For certain. X marks the spot: A phrase that is said when someone finds something he/she has been looking for. You are What You Eat: In order to stay healthy you must eat healthy foods. You can’t Judge a Book by its Cover: Decisions shouldn’t be made primarily on appearance. You can’t Take it With You: Enjoy what you have and not what you don’t have, since when you die you

cannot take things (such as money) with you. Your Guess is as Good as Mine: I have no idea. Zero Tolerance: No crime or law breaking big or small will be overlooked.

French Phrases Here’s a list of French phrases and sayings that are used in English often enough to have become part of the language. Many of these relate to those French preoccupations, fashion and food.

À la carte On the menu, with each dish priced. À la mode Fashionable; also, in the USA, with ice cream. Agent provocateur ‘Provocative agent’—a spy employed to induce or incite a suspected

person or group to commit an incriminating act. Aide-de-camp An officer who assists a general in his military duties. Aide-memoire An aid to memory. Après-ski Socializing after a skiing session. Also a name of a type of footwear worn

after removing ski boots. Art déco ‘Decorative art’ —a style of art originating in Paris in the early 20th century.

An eclectic and glamourous artform, taking in aspects of Cubism and geometric industrial design.

Art nouveau ‘New art’—a style of art developed towards the end of the 19th century. It is characterized by ornamentation based on organic or foliate forms and by its asymmetric and curvaceous lines.

Au contraire To the contrary. Often used with an arch or rather camp form of delivery. Au fait To be conversant with; familiar with. Au gratin ‘With gratings’—in French, anything that is grated onto a food dish. In

English, specifically ‘with cheese’. Au naturel Undressed or ‘in a natural state. Au pair A young foreigner, usually female, who undertakes domestic tasks in

exchange for accommodation. Au revoir Farewell for the time being. Sometimes given in English in the jokey au

reservoir version. Avant garde The pioneers or innovators in art in a particular period. Also, a military

term, meaning vanguard oradvance guard. Belle époque ‘Beautiful era’—the golden age of art and culture in France in the early

20th century.

Page 99: Study Kit Brochure for SSC Combined Graduate Level ...static.upscportal.com/files/study-kit/ssc-cgl/SSC-CGL-2013... · Brochure for SSC Combined Graduate Level Examination - 2014

Click Here for SSC CGL Online Coaching: http://sscportal.in/community/courses/ssc-cgl-tier-1

[Click Here to Buy This Study Kit in Hard Copy: http://sscportal.in/community/study-kit/cgl ] 98

Bête noire A pet peeve. A thing or person found particularly unwelcome and to be avoided.

Billet doux A short love letter or note. Bon appétit ‘Good appetite’ — “Enjoy your food”. Bon mot Clever, witty remark. Bon vivant ‘Good liver’ — a person who enjoys life, especially ‘wine, women and song’. Bon voyage Have a good trip. Café au lait Coffee with milk. Carte blanche Having free rein to choose whatever course of action you want. Cause célèbre An issue arousing widespread controversy or debate. An English invention,

rarely used in France. C’est la vie That’s life or such is life. Often used in disappointed resignation following

some bad fortune. Chaise longue ‘Long chair’ - a form of sofa with an elongated seat long enough to support

the legs. Often erroneously called a chaise lounge in the USA. This isn’t the derivation of either the noun or verb lounge, which both long pre- date the invention of chaise longues.

Chargé d’affaires A diplomat, temporarily in charge of business. Cherchez la femme Literally, “look for the woman.” Cinéma vérité A form of filmmaking that combines documentary-style techniques to tell

a story. Cordon bleu High quality, especially of cooking. Cordon sanitaire A political or medical buffer zone. Coup d’état An abrupt overthrow of a government through unconstitutional means, for

example, by force, or by occupation of government structures during the leader’s absence. Coup de grâce Originally a blow by which one condemned or mortally wounded is ‘put out

of his misery’. Figuratively, a finishing stroke, one that settles or puts an end to something. Crème brûlée ‘Burnt cream’ —baked custard with a carmelized crust. Crème caramel A flan. A custard dessert with a layer or caramel on top. Crème de la crème The best of the best. Literally the cream of the cream. Cri de coeur ‘Cry of the heart’—a heartfelt cry of anguish. Cul-de-sac A thoroughfare that is closed at one end—a blind alley. Also, figuratively, a

venture leading to no successful outcome. Déjà vu The feeling of having seen or experienced something before. Literally

‘already seen’. De rigueur Obligatory or expected, especially with reference to fashion. Double entendre A word or phrase that has a double meaning—one of which is often

vulgar or sexual in nature. A staple form of British toilet humour— Carry On films would be virtually silent without it. For example, see ‘gone for a P’ in wee-wee.

Do jour ‘Of the day’ —as in ‘soup du jour’ (‘soup of the day’).

Page 100: Study Kit Brochure for SSC Combined Graduate Level ...static.upscportal.com/files/study-kit/ssc-cgl/SSC-CGL-2013... · Brochure for SSC Combined Graduate Level Examination - 2014

Click Here for SSC CGL Online Coaching: http://sscportal.in/community/courses/ssc-cgl-tier-1

[Click Here to Buy This Study Kit in Hard Copy: http://sscportal.in/community/study-kit/cgl ] 99

Éminence grise A powerful adviser or decision-maker who operates secretly or unofficially. Literally ‘grey eminence’.

Enfant terrible Literally, a “terrible child”. It is sometimes used to describe unruly children. More commonly, it is used in relation to adults who cause trouble by unorthodox or ill-considered speech or behaviour—especially those who have habitually done this from an early age.

En masse In a group; all together. En passant In passing. En route On the way. En suite Part of a set, especially a series of rooms that adjoin each other forming a suite. Esprit de corps The regard entertained by the members of a group, especially a military

unit, for the honour and interests of the group as a whole. Literally, ‘spirit of the corps’.

Fait accompli An irreversible action that has happened before those affected by it knew of its existence.

Faux pas A social blunder, causing embarrassment or loss of reputation. Literally, a ‘false step’.

Film noir A bleak cinematographic style, usually dark visually and in storyline terms. Femme fatale A dangerously attractive woman. Fleur de Lis The heraldic lily; a device supposed by some to have originally represented

an iris, by others the top of a sceptre, of a battle-axe or other weapon. Force majeure Irresistible force or overwhelming power. Grand prix The premier events of several sports, especially the races in the Formula I

motor racing championship. Literally, ‘grand prize’. Haute couture ‘High sewing’—trend-setting high fashion. Also, the collective name for the

leading dressmakers and designers. Haute cuisine High class cooking. Literally, ‘upper kitchen’. Hors de combat ‘Out of combat’—unable to fight. Hors d’oeuvres An extra dish served as a relish to whet the appetite, normally at the start of a

meal. Je ne sais quoi An indescribable or inexpressible something. Literally, ‘I know not what’. Joie de vivre A feeling of healthy enjoyment of life; exuberance, high spirits. Laissez-faire The principle that government should not interfere with the action of

individuals. Also, more generally, a policy of indulgence towards the actions of others. Literally, ‘let (people) do (as they think best)’.

L’esprit de l’escalier This isn’t actually widely adopted into English. I include it here in the hope that it might become so. It means—thinking of a suitable retort or remark after the opportunity to make it has passed. Literally, ‘the wit of the staircase’.

Mal de mer Seasickness. Mardi gras The last day of the Carnival or pre-Lenten season. Literally, ‘Fat Tuesday’,

called Shrove Tuesday in the UK.

Page 101: Study Kit Brochure for SSC Combined Graduate Level ...static.upscportal.com/files/study-kit/ssc-cgl/SSC-CGL-2013... · Brochure for SSC Combined Graduate Level Examination - 2014

Click Here for SSC CGL Online Coaching: http://sscportal.in/community/courses/ssc-cgl-tier-1

[Click Here to Buy This Study Kit in Hard Copy: http://sscportal.in/community/study-kit/cgl ] 100

Ménage à trois ‘Household of three’ —three people in a sexual relationship. Merci beaucoup Thank you very much. Mot juste Exactly the right word or expression. N’est-ce pas? Is it not so? Noblesse oblige The responsibility conferred by rank. Literally, ‘noble rank entails responsibility’. Nom de guerre A name assumed by individuals engaged in a military enterprise or

espionage, usually in order to conceal their true identity. Literally, ‘war name’. Nom de plume An assumed name under which a person writes or publishes. Literally,

‘pen name’. Nouveau riche “Newly rich’ - a snobbish term for a person who has come into money

but has not developed the manners expected of wealthy people. Nouvelle cuisine A form of cooking of the mid 20th century that emphasized lightness and

decorative form. Objet d’art An ‘art object’. Papier mâché A material used for sculptural artwork and craftwork. Literally ‘mashed

paper’. Par excellence Pre-eminently supreme—above all others. Pas de deux Impossible to avoid the corny father of twins’ joke here. The real meaning is a

dance (typically a ballet), and in extended use a partnership, between two people. Petit four A small dessert—usually a dainty cake. Petit mal ‘Small illness’ —a mild epilepsy. Petit point Small stitching, used in needlepoint. Piece de résistance The best part or feature of something, especially of a meal. Pied-à-terre A second home, typically an apartment in the city. Prêt-à-porter Ready-to-wear clothing. Pot-pourri A mixture of dried petals of different flowers mixed with spices, kept in a

jar for its perfume. Also, a stew made from a variety of meats cooked together. By extension, any collection of miscellaneous items.

Quelle horreur What a horrible thing? This is frequently used sardonically, when the ‘horror’ is trivial.

Qu’est-ce que c’est? What is this? Raison d’être The thing that is central to our existence. Literally, ‘reason for being’. RSVP Please respond (to my message). Literally the abbreviation of ‘Répondez, s’il

vous plaît’. Sacré bleu This general mild exclamation of shock is the archetypal French phrase, as

viewed by the English. No portrayal of a stage Frenchman in an English farce could be complete without a character in a beret and striped jumper, shrugging his shoulders and muttering ‘Sacre bleu!’. Literally, ‘holy blue’, which refers to the colour associated with the Virgin Mary.

Sang-froid Coolness, indifference. Literally, ‘cold blood’. Savoir-faire Social grace; means know-how in French. S’il vous plaît Please. Literally, ‘if it pleases you’. Soupe du jour ‘Soup of the day’ —the soup offered by a restaurant that day. Table d’hôte A full-course meal offering a limited number of choices and served at a

fixed price in a restaurant or hotel.

Page 102: Study Kit Brochure for SSC Combined Graduate Level ...static.upscportal.com/files/study-kit/ssc-cgl/SSC-CGL-2013... · Brochure for SSC Combined Graduate Level Examination - 2014

Click Here for SSC CGL Online Coaching: http://sscportal.in/community/courses/ssc-cgl-tier-1

[Click Here to Buy This Study Kit in Hard Copy: http://sscportal.in/community/study-kit/cgl ] 101

Tête-à-tête A private meeting between two people. Literally, ‘head-to-head’. Tout de suite At once. Tour de force A masterly stroke or feat of strength or skill. Literally, ‘feat of strength’. Trompe l’œil An art technique involving high levels of realism in order to create the

illusion that the depicted objects are real rather than paintings. Literally, ‘trick the eye’.

Vis-à-vis In a position facing another. Literally ‘face to face’. Often now used in the sense of ‘in relation to’.

Vive la différence Long live the difference (between male and female). Vol-auvent A small, light savoury pastry. Literally ‘flight of the wind’. Zut alors A general exclamation. Like Sacré bleu, this is more likely to be spoken by

pretend Frenchmen than by real ones.

Latin Phrases Here’s a list of Latin phrases and sayings that are used in English often enough to have become part of the language.

A cappella [Music] in church [style], i.e., unaccompanied voices. A priori From what comes before. Ad absurdum To the point of absurdity. Ad hoc For this purpose. Ad infinitum Without limit-endlessly. Ad nauseam To a sickening extent. Alma Mater One’s old school (literally ‘bounteous mother’). Alter ego Other (alternative) self. Annus horribilis A horrible year. Agnus Dei Lamb of God. Aqua pura Pure water. Aqua vitae Alcoholic spirit, e.g., brandy/whisky. Literally ‘water of life’. Ars longa, vita brevis Skill takes time to acquire, but life is short. Ave Maria Hail Mary. Carpe diem Seize the day (More literally translated as ‘enjoy the day, pluck it when it is

ripe’). Caveat emptor Let the buyer beware. Cogito ergo sum I think, therefore I am. Corpus Christi The body of Christ. De facto In fact—in reality. Dei Gratia By the grace of God. E Pluribus Unum One from many. Et cetera (etc.) And the rest. Et tu, Brute And you, Brutus.

Page 103: Study Kit Brochure for SSC Combined Graduate Level ...static.upscportal.com/files/study-kit/ssc-cgl/SSC-CGL-2013... · Brochure for SSC Combined Graduate Level Examination - 2014

Click Here for SSC CGL Online Coaching: http://sscportal.in/community/courses/ssc-cgl-tier-1

[Click Here to Buy This Study Kit in Hard Copy: http://sscportal.in/community/study-kit/cgl ] 102

Ex libris ‘Out of the books’, i.e., from the library. Gloria in excelsis deo Glory to God in the highest. Habeas corpus You must have the body (in court). In absentia In one’s absence. In camera In private chamber. In flagrante delicto In the act of committing an offence. In loco parentis In the place of a parent. In vitro In a test tube (literally ‘in glass’). Ipso facto By that very fact. Magnum opus A great work. Mea culpa My fault. Modus operandi Mode of operating. (m.o.) Nolens volens Willingly or unwillingly (see also, willy-nilly). Post partum After childbirth. Praemonitus, Forewarned is forearmed. praemunitus Prima facie At first sight; on the face of it. Pro bono Without charge—for the public good. Opus Dei The work of God. Quid pro quo Something for something, i.e., a favour for a favour. Quo vadis? Where are you going? Rigor mortis The rigidity of death. Semper fidelis Always faithful. Sine qua non Indispensable. Status quo The current state of affairs. Sub judice Before a court. Tempus fugit Time flees. Terra firma Solid ground. Urbi et orbi To the city and to the globe. Veni vidi vici I came, I saw, I conquered. Vice versa The other way around. Vivat Regina Long live the queen.

Vox populi The voice of the people.

EXERCISE:

Directions (Q. No. 1-30): Select the meaning of the bold Idiom or phrase from the given alternatives. 1. In the organised society of today no individual or nation can plough a lonely furrow.

Page 104: Study Kit Brochure for SSC Combined Graduate Level ...static.upscportal.com/files/study-kit/ssc-cgl/SSC-CGL-2013... · Brochure for SSC Combined Graduate Level Examination - 2014

Click Here for SSC CGL Online Coaching: http://sscportal.in/community/courses/ssc-cgl-tier-1

[Click Here to Buy This Study Kit in Hard Copy: http://sscportal.in/community/study-kit/cgl ] 103

(a) remain unaffected (b) do without the help of others (c) survive in isolation (d) remain non-aligned 2. To flog a dead horse (a) to do interesting things (b) to try to take work from a weak horse (c) to beat a horse that is dead (d) to revive interest in a subject which is out of date 3. To bait the hook to suit the fish (a) to look at things from other person's point of veiw (b) to catch fish by providing suitable food (c) to do things to please others (d) to prepare a box to pack the fish 4. Bed of roses (a) very soft bed (b) dull life (c) belong to (d) full of joys 5. Black and blue (a) painting (b) severely (c) together (d) intermix 6. Smell a rat (a) to act unfairly (b) to talk boastfully (c) to have reason to suspect (d) to discourage 7. Under duress (a) under compulsion (b) willingly (c) to elicit information (d) to demand 8. In the throes of (a) out of the grip (b) in the grip of (c) uncontrolled (d) in the power of 9. There followed an awkward silence after Salim declared his love to her. (a) Uncomfortable silence (b) Embarrassing silence (c) Clumsy silence (d) A period of embarrassment 10. Looking at them now, who can imagine that they were a couple of babes in the wood in school! (a) Children or babies in the forest (b) Babies made of wood (c) Babies lost in the wood (d) Innocent and inexperienced people 11. The ship was about to weigh anchor when the storm came. (a) Weigh the anchor (b) Leave the docking berth (c) Raise the anchor as a preparation to sail again (d) Drop the anchor

Page 105: Study Kit Brochure for SSC Combined Graduate Level ...static.upscportal.com/files/study-kit/ssc-cgl/SSC-CGL-2013... · Brochure for SSC Combined Graduate Level Examination - 2014

Click Here for SSC CGL Online Coaching: http://sscportal.in/community/courses/ssc-cgl-tier-1

[Click Here to Buy This Study Kit in Hard Copy: http://sscportal.in/community/study-kit/cgl ] 104

12. I did not know that he was pulling my leg all the time. (a) complimenting me (b) defaming me (c) degrading me (d) befooling me 13. Only the cowards hit below the belt. (a) give a crippling blow (b) hit in the thighs (c) act unfairly (d) act brutally 14. Sow the wind and reap the whirlwind (a) A good done to others does not go unnoticed (b) Do or say a thing which may bring severe punishment (c) Prevention is better than cure (d) See no evil and do no evil 15. Anurag has made a laughing stock of the braggart: (a) Foolish (b) Laugh (c) Laughing store-house (d) Look ridiculous

ANSWERS: 1. (b) 2. (d) 3. (a) 4. (d) 5. (b) 6. (c) 7. (a) 8. (b) 9. (b) 10. (d) 11. (c) 12. (d) 13. (c) 14. (b) 15. (d)